PDA

نسخه کامل مشاهده نسخه کامل : اتاق ریاضیات(طرح سؤالات)



صفحه ها : 1 2 3 4 5 6 7 8 9 10 11 12 [13] 14 15 16 17 18 19 20

msm43njn
28-08-2010, 18:31
سایت انگلیسی زبان سراغ دارید که مسائل ریاضی دانشگاهی رو که دانشجویان پرسیدن و به اونها جواب داده شده رو مرتب برای ما ایمیل کنه؟

davy jones
28-08-2010, 20:33
لطفا حد زیر رو برام حل کنید.
روش تعریف حد
روش هوپیتال

با شرط P>1

[ برای مشاهده لینک ، لطفا با نام کاربری خود وارد شوید یا ثبت نام کنید ] %5Cleft&space;%28&space;lnn&space;%5Cright&space;%29%5Ep%7D%7Bn%7D


سلام.

حل از طریق هوپیتال (n بار هوپیتال میگیریم):


[ برای مشاهده لینک ، لطفا با نام کاربری خود وارد شوید یا ثبت نام کنید ] 7B%28ln%28n%29%29%5E%7Bp%7D%7D%7Bn%7D%5Coverset%7B hop%7D%7B%5Crightarrow%7DI=%5Clim_%7Bn&space;%5Cto&space;%5Cin fty&space;%7D%28%5Cfrac%7B%7B%5B%28ln%28n%29%29%5E%7Bp%7 D%5D%7D%27%7D%7B%7Bn%7D%27%7D%29=%5Clim_%7Bn&space;%5Cto &space;%5Cinfty&space;%7D%5Cfrac%7B%5Cfrac%7B1%7D%7Bn%7Dp%28ln %28n%29%29%5E%7Bp-1%7D%7D%7B1%7D=p%5Ctimes&space;%5Clim_%7Bn&space;%5Cto&space;%5Cinft y&space;%7D%5Cfrac%7B%28ln%28n%29%29%5E%7Bp-1%7D%7D%7Bn%7D=p%28p-1%29%5Ctimes&space;%5Clim_%7Bn&space;%5Cto&space;%5Cinfty&space;%7D%5Cfrac %7B%28ln%28n%29%29%5E%7Bp-2%7D%7D%7Bn%7D=...=p%28p-1%29%28p-2%29...%5Ctimes&space;2%5Ctimes&space;1%5Ctimes&space;%5Clim_%7Bn&space;%5 Cto&space;%5Cinfty&space;%7D%5Cfrac%7B1%7D%7Bn%7D=p%21%5Ctimes &space;%5Clim_%7Bn&space;%5Cto&space;%5Cinfty&space;%7D%5Cfrac%7B1%7D%7Bn% 7D=0



از روش تعریف حد نمیدونم چطوری بایر ثابت کنم. فقط اینکه به طور شهودی واضحه که رشد توابع چندجمله ای در بینهایت بیشتر از توابع لگاریتمی هستش و بنابراین مخرج بیشتر به سمت بینهایت میره و در نتیجه حاصل حد برابر با صفره.

شدت رشد توابع در بینهایت به این ترتیبه:

[ برای مشاهده لینک ، لطفا با نام کاربری خود وارد شوید یا ثبت نام کنید ] 8a%5E%7Bn%7D%29%3EO%28n%5E%7Ba%7D%29%3EO%28ln%28n% 29%5E%7Ba%7D%29%5C;&space;%5C;&space;%5C;%5C;&space;%5C;&space;%5C;&space;%5C;&space;a ,b%3E1
(منظور از O میزان رشد در بینهایته)

موفق باشین.
89/6/6

Daiereh
29-08-2010, 20:41
15جعبه داريم كه در هر جعبه 15 سكه وجود دارد وزن هر سكه تقلبي 4gو وزن هر سكه اصلي 5g‏ است اكر سكه هاي 14 جعبه از 15 جعبه اصلي باشد و يك جعبه سكه هاي تقلبي داشته باشد جطور ميتوان با يك بار وزن كردن جعبه تقلبي را بدست آورد?

panizir
30-08-2010, 03:36
از جعبه اول يك سكه جعبه دوم دو سكه و ... برميداريم وزن ميكنيم. اگه مجموعش از 120 گرم 1 گرم كم بود يعني جعبه اول 2 گرم كم بود يعني جعبه دوم و ... سكه هاي تقلبي دارن.

Daiereh
30-08-2010, 08:58
آفرين panizir‏ و ممنون.

panizir
30-08-2010, 11:04
البته از 120 گرم نبايد ميگفتم بايد ميگفتم از 600 گرم. خواهش ميكنم.

iranch
02-09-2010, 13:35
تایع خطی را بیابید.
که از نقطه ی (1.5) بییابید و وارون آن تایع نباشد.\

در تابع خطی
f={(x,y)lx,y E R,y=ax+b{
a,b را چنان بیابیئ که تایع f با وارون آن برابر باشد.
ممنون پاسخ بدید.

lebesgue
07-09-2010, 18:05
لطفا حذف شود.

ali_hp
12-09-2010, 13:50
سلام،برای حل این مساله کافیه که دقت کنید نمودار وارون یک تابع از قرینه کردن نمودار تابع نسبت به نیمساز ربع اول و سوم بدست می آید.و یک خط وقتی و فقط وقتی نمودار یک تابع نیست که موازی محور Y ها باشد.

تایع خطی را بیابید.
که از نقطه ی (1.5) بییابید و وارون آن تایع نباشد.
پس باید خطی را بیابیم که از نقطه (1.5) بگذرد،و وقتی نسبت به y=x قرینه می شود،موازی محور Y ها شود،پس این خط باید موازی محور x ها باشد.و معادله خط می شود: y=5 .
البته در اینجا منظور سوال از وارون،تابع وارون نیست!


در تابع خطی
f={(x,y)lx,y E R,y=ax+b{
a,b را چنان بیابیئ که تایع f با وارون آن برابر باشد.
ممنون پاسخ بدید.یعنی باید خطهایی را در صفحه بیابیم که قرینه شان نسبت به y=x بر خود خط منطبق شود،یک جواب خود استy=x و دسته دیگر خطهایی که بر y=x عمودند.که شیب این خطها منفی یک است و عرض از مبدا آنها هرمقداری می تواند باشد.پس a=1 و b=0 یک جواب است و a=-1 و b دلخواه یک دسته جواب دیگر است.

lebesgue
22-09-2010, 20:24
لطفا حذف شود.

karajboy
24-09-2010, 15:28
سلام، خسته نباشید. اگه میشه این دوتا خاصیت اعداد مختلط رو از روش استقرا اثبات کنید:

[ برای مشاهده لینک ، لطفا با نام کاربری خود وارد شوید یا ثبت نام کنید ]

attractive_girl
28-09-2010, 23:46
سلام.جواب این سوال رو تا روز یکشنبه میخواستم .
ممنون
چرا بعضی اعداد اعشاری را وقتی میخواهیم از مبنای 10 به مبنای 2 ببریم قسمت اعشاری بعد از چند مرحله به صفر رسیده اما برخی اعداد به صفر نمیرسند؟
مثال : دو عدد 1101/1011 و 8/8888 که عدد 8/8888 به صفر نمیرسد

lebesgue
29-09-2010, 11:18
سلام.جواب اين سوال رو تا روز يکشنبه ميخواستم .
ممنون
چرا بعضي اعداد اعشاري را وقتي ميخواهيم از مبناي 10 به مبناي 2 ببريم قسمت اعشاري بعد از چند مرحله به صفر رسيده اما برخي اعداد به صفر نميرسند؟
مثال : دو عدد 1101/1011 و 8/8888 که عدد 8/8888 به صفر نميرسد

اعداد گنگ در هر مبنايي تعداد رقم بعد از مميز نامحدود دارند (اين به سادگي قابل اثبات است.)
پس بحث ما محدود مي شود به اعداد گويا.

چه اعدادي در مبناي 10، تعداد رقم بعد از مميز محدود دارند؟
عدد گوياي m/n که در آن m و n صحيح و نسبت به هم اول هستند را در نظر بگيريد.
اگر (و فقط اگر) بتوان اين عدد را با تعداد محدودي رقم بعد از مميز مانند p نشان داد، خواهيم داشت:

[ برای مشاهده لینک ، لطفا با نام کاربری خود وارد شوید یا ثبت نام کنید ]{m}{n}=%5Cfrac{x}{10^ {p}}%5Crightarrow%20x=%5Cfrac{m}{n}10^{p}
که در آن x عددي صحيح است.
اين حاصل تنها در صورتي مي تواند صحيح باشد که 10 به توان p، بر n
بخش پذير باشد و در نتيجه عوامل اول n تنها مي تواند 2 و 5 باشد.

چه اعدادي در مبناي 2، تعداد رقم بعد از مميز محدود دارند؟
به طور مشابه مي توان نشان داد n بايد عامل اولي جز 2 نداشته باشد.

نتيجه:
آن دسته از m/n ها که n فقط عامل 2 را داشته باشد، هم در مبناي 2 و هم در مبناي 10 تعداد محدودي رقم بعد از مميز دارند،
اما آن دسته از m/n ها که n هم عامل 2 و هم 5 را داشته باشد، فقط در مبناي 10 تعدادي محدود رقم بعد از مميز دارند.

توجه داشته باشيد که کلمه "اعشار" از عشر به معناي يکدهم مياد، بنابراين استفاده از کلمه اعشار براي مبناهاي غير از 10
زياد جالب نيست، براي همين من به جاي اون از عبارت "رقم بعد از مميز" استفاده کردم.

CATALONIA
30-09-2010, 20:59
سلام و خسته نباشيد به تمام زحمت كشان اين تاپيك ! 
  چند تا سوال داشتم كه تا شنبه بهشون نياز دارم .. متشكر 




پاره خطي به طول a مفروض است . پاره خطي به طول راديكال a رسم كنيد . ( در توضيح اين سوال بايد بگم كه منظور نشان دادن اعداد گنگ روي محور اعداد است . يعني مثلا راديكال 19 را چطوري ميشه فقط در يك حركت روي محور نشان داد . يعني به صورتي نباشه كه چندين مثلت بكشيم تا بتونيم اين نقطه رو نشون بديم ، فقط با يه حركت اين نقطه مشخص بشه ) 
مي دانيم راديكال 2 گنگ است . ثابت كنيد راديكال 3 + راديكال 5 + راديكال 2 گنگ است . ( در توضيح اين سوال بگم كه راديكال 3 روي تمام راديكال هاي ديگر است و راديكال 5 هم روي راديكال 2 ؛ يعني راديكال 2 و 5 زيره راديكال 3 هستند و راديكال 2 زيره راديكال 5 . و همچنين اگر امكانش هست اين سوال رو از روش برهان خلف حل كنيد خيلي بهتره .. اگر با اين روش حل بشه  ) 
فرض كنيد راديكال 2 گنگ است . ثابت كنيد راديكال 3 - راديكال 2 گنگ است . ( توضيح خاصي نداره ولي بهتره از روش برهان خلف حل بشه . ) 

يه چند تا سوال ديگه هم بود كه سخته نشون دادنشون ... برا همين از خيرشون گذشتيم :دي
دوستان اين سوالات در محدوده ي كلاس اول دبيرستان و كتاب مبتكران هست و اگر ممكنه زياد از سطح هاي بالا حلش نكنيد بهتره ... خواهشا تا شنبه هم جواب ها رو بديد كه خيلي خيلي متشكر مي شم ... 
ممنون ! 

lebesgue
30-09-2010, 22:20
1. اگر بتوانید پاره خطی با طول واحد رسم کنید:

[ برای مشاهده لینک ، لطفا با نام کاربری خود وارد شوید یا ثبت نام کنید ]

2. فرض کنید این حاصل گویا باشد:

[ برای مشاهده لینک ، لطفا با نام کاربری خود وارد شوید یا ثبت نام کنید ]{3+%5Csq rt{5+%5Csqrt{2}}}%5C%5C%5C%5C%20%5Crightarrow%20a^ {2}=3+%5Csqrt{5+%5Csqrt{2}}%5C%5C%5C%5C%20%5Cright arrow%20(a^{2}-3)^{2}=5+%5Csqrt{2}%5C%5C%5C%5C%20%5Crightarrow%20 %5Csqrt{2}=(a^{2}-3)^{2}-5

می دانیم حاصلجمع و حاصلضرب دو عدد گویا، عددی گویاست.
در نتیجه [ برای مشاهده لینک ، لطفا با نام کاربری خود وارد شوید یا ثبت نام کنید ]{2} گویاست که این با فرض تناقض دارد.

3. مشابه قبلی حل می شود.

iranch
03-10-2010, 20:53
سلام
اگر میشه کمک کنید
اولی حاصل رو میخواهد
و دومی چطوری با استفاده از تعریف لگاریتم این شده؟
[ برای مشاهده لینک ، لطفا با نام کاربری خود وارد شوید یا ثبت نام کنید ]
[ برای مشاهده لینک ، لطفا با نام کاربری خود وارد شوید یا ثبت نام کنید ]

davy jones
05-10-2010, 12:50
سلام
اگر میشه کمک کنید
اولی حاصل رو میخواهد
و دومی چطوری با استفاده از تعریف لگاریتم این شده؟
[ برای مشاهده لینک ، لطفا با نام کاربری خود وارد شوید یا ثبت نام کنید ]
[ برای مشاهده لینک ، لطفا با نام کاربری خود وارد شوید یا ثبت نام کنید ]

سلام
یکم دیگه عکسها رو تار میکردی تا کاملا دیگه قابل خوندن نباشه:27:

سوال اولت اینه؟؟؟؟؟؟؟؟


[ برای مشاهده لینک ، لطفا با نام کاربری خود وارد شوید یا ثبت نام کنید ] rt%7B2%5Csqrt[3]%7B3%7D%7D


سوال دومت رو هم متوجه نشدم. لگاریتم x به پایه ی 9 مساوی با چنده؟

جون من برو تاپیک انتشار درست فرمولهای ریاضی رو بخون:

برای مشاهده محتوا ، لطفا وارد شوید یا ثبت نام کنید

موفق باشین.
89/7/13

a.g g.a
06-10-2010, 16:05
سلام
یک سوال از سال دوم دبیرستان بخش دنباله حسابی:

[ برای مشاهده لینک ، لطفا با نام کاربری خود وارد شوید یا ثبت نام کنید ]

a.g g.a
06-10-2010, 18:17
لطفا زود تر جواب بدید می خوام ببینم خودم درست حلش کردم یا نه.
با تشکر

afshin b
06-10-2010, 19:37
لطفا زود تر جواب بدید می خوام ببینم خودم درست حلش کردم یا نه.
با تشکر
من فکر میکنم جواب صفر بشه!!
شما به چه جوابی رسیدین؟

a.g g.a
06-10-2010, 19:46
نه جواب منmn-شد.
اگهaاندیسnمساویmباشه وaاندیسmمساویnباشه جواب صفر میشه.

pbmath
07-10-2010, 09:34
فرض کنید <a href="[ برای مشاهده لینک ، لطفا با نام کاربری خود وارد شوید یا ثبت نام کنید ]{0},x_{1},...,x_{n}" target="_blank"><img src="[ برای مشاهده لینک ، لطفا با نام کاربری خود وارد شوید یا ثبت نام کنید ]{0},x_{1},...,x_{n}" title="x_{0},x_{1},...,x_{n}" /></a> نقاط متمایز باشند وبرای هر <a href="[ برای مشاهده لینک ، لطفا با نام کاربری خود وارد شوید یا ثبت نام کنید ] i\leq n" target="_blank"><img src="[ برای مشاهده لینک ، لطفا با نام کاربری خود وارد شوید یا ثبت نام کنید ] i\leq n" title="0\leq i\leq n" /></a> و <a href="[ برای مشاهده لینک ، لطفا با نام کاربری خود وارد شوید یا ثبت نام کنید ]{i}" target="_blank"><img src="[ برای مشاهده لینک ، لطفا با نام کاربری خود وارد شوید یا ثبت نام کنید ]{i}" title="w_{i}" /></a> داشته باشیم <a href="[ برای مشاهده لینک ، لطفا با نام کاربری خود وارد شوید یا ثبت نام کنید ]" target="_blank"><img src="[ برای مشاهده لینک ، لطفا با نام کاربری خود وارد شوید یا ثبت نام کنید ]" title="\sum" /></a> را ثابت کنید
<a href="[ برای مشاهده لینک ، لطفا با نام کاربری خود وارد شوید یا ثبت نام کنید ]{i}= \prod_{j= 0;j\neq i}^{j= n}\left ( x_{i}-x_{j} \right )\Rightarrow \sum_{i= 0}^{i= n}\frac{{x_{i}}^{n}}{w_{i}}= 1" target="_blank"><img src="[ برای مشاهده لینک ، لطفا با نام کاربری خود وارد شوید یا ثبت نام کنید ]{i}= \prod_{j= 0;j\neq i}^{j= n}\left ( x_{i}-x_{j} \right )\Rightarrow \sum_{i= 0}^{i= n}\frac{{x_{i}}^{n}}{w_{i}}= 1" title="w_{i}= \prod_{j= 0;j\neq i}^{j= n}\left ( x_{i}-x_{j} \right )\Rightarrow \sum_{i= 0}^{i= n}\frac{{x_{i}}^{n}}{w_{i}}= 1" /></a>

esmaeelm2007
07-10-2010, 10:26
سلام
چطوری میشه ثابت کرد e^2 گنگه؟

CATALONIA
07-10-2010, 20:00
سلام دوستان عزيز !‌
ممنون ميشم به اين سوالات پاسخ بديد : 
1- 1000 عدد گنگ بين يك دوم و يك سوم بنويسيد . 
2- n عدد گنگ بين دو عدد گويا ي x و y بنويسيد . ( x< y ) 
3- ريشه ي چهارم 2 را روي محور اعدا نشان دهيد . 
  
  
  


فقط عزيزان هر چه زودتر اين سوالات رو جوابش رو بهم برسونيد خيلي بهتره ... 
شديدا بهشون نياز دارم 
ممنون و متشكر !‌

a.g g.a
09-10-2010, 18:40
سلام
یک سوال از سال دوم دبیرستان بخش دنباله حسابی:

[ برای مشاهده لینک ، لطفا با نام کاربری خود وارد شوید یا ثبت نام کنید ]
سلام
لطفا جواب بدین.

lebesgue
10-10-2010, 23:42
سلام
یک سوال از سال دوم دبیرستان بخش دنباله حسابی:

[ برای مشاهده لینک ، لطفا با نام کاربری خود وارد شوید یا ثبت نام کنید ]

اگر m≠n بوده و d قدر نسبت تصاعد باشد:

[ برای مشاهده لینک ، لطفا با نام کاربری خود وارد شوید یا ثبت نام کنید ]{m}-a_{n}=(m-n)d%5Crightarrow%20d=%5Cfrac{a_{m}-a_{n}}{m-n}=%5Cfrac{n^{2}-m^{2}}{m-n}=-(m+n)%5C%5C%5C%5C%20%5Crightarrow%20a_{m+n}=a_{m}+ nd=n^{2}-n(m+n)=-mn
اگر m=n باشد، هیچ نتیجه ای درباره {a_{m+n نمی توان گرفت.

lebesgue
10-10-2010, 23:52
سلام دوستان عزيز !‌
ممنون ميشم به اين سوالات پاسخ بديد : 
1- 1000 عدد گنگ بين يك دوم و يك سوم بنويسيد . 
2- n عدد گنگ بين دو عدد گويا ي x و y بنويسيد . ( x< y ) 
3- ريشه ي چهارم 2 را روي محور اعدا نشان دهيد . 
  
  
  


فقط عزيزان هر چه زودتر اين سوالات رو جوابش رو بهم برسونيد خيلي بهتره ... 
شديدا بهشون نياز دارم 
ممنون و متشكر !‌

1. حالت خاصی از 2 است.

2.
[ برای مشاهده لینک ، لطفا با نام کاربری خود وارد شوید یا ثبت نام کنید ]{%5Csqrt{2}}{ %5Cleft%20%5Clfloor%20%5Cfrac{%5Csqrt{2}}{y-x}%20%5Cright%20%5Crfloor+i}%5C%5C%5C%5C1%5Cleq%20 i%5Cleq%20n

3. می توانید روشی که در پست #3016 ارائه شد را دوبار به کار ببندید.

lebesgue
11-10-2010, 00:07
با سلام دوستان من اثبات این فرمول رو خواستم لطفا کمکم کنید نیاز فوری دارم
کسر مولد اعداد اعشاری متناوب :
دوره غیر گردش-دوره ی گردش و غیر گردش
ـــــــــــــــــــــــــ ـــــــــــــــــــــــــ ــــــــــــــــ
به تعداد ارقام گردش نه وبه تعدادارقام غیر گردش بعد از ممیز صفر

مثال:
4-425=...4/2525252525
ــــــــ
99
=
421
ــــــ
99

به عنوان یک راهنمایی، در همین مثال شما:

[ برای مشاهده لینک ، لطفا با نام کاربری خود وارد شوید یا ثبت نام کنید ]{25}%5C rightarrow%20100x=425.%5Coverline{25}%5C%5C%5C%5C% 20100x-x=425.%5Coverline{25}-4.%5Coverline{25}=421%5C%5C%5C%5C%2099x=421%5Crigh tarrow%20x=%5Cfrac{421}{99}

pianist1362
12-10-2010, 11:09
سلام بر مهندسين عزيز من يك سوال در مورد مجموعه ها داشتم
مجموعه هائي كه نامتناهي ولي شمارش پذير هستند آيا مجموعه تواني آنه هم شمارش پذير است يا نه ؟ با تشكر.

m.imeni
12-10-2010, 14:23
سلام دوستان ، خسته نباشید .
من می خوام یک مقادیر ویژه ی یک ماتریس را حساب کنم ، اما بهد از حل دترمینان به یک معادله یک مجهولی درجه سه میرسم و نمی تونم ریشه هاشو محاسبه کنم . فکر می کنم باید محاسباتی صورت بگیره که معادله درجه 3 تجزیه بشه یا اینکه ماتریس تغییر پیدا کنه قبل از عملیات دترمینان (مثلا تبدیل به ماتریس بالا مثلثی بشه بعد حل بشه) . در کل من اینارو نمی دونم . ممنون میشم کمکم کنید .


اینم ماتریس مورد نظر :



[ برای مشاهده لینک ، لطفا با نام کاربری خود وارد شوید یا ثبت نام کنید ]



این یکی هم یک ماتریس دیگه :



[ برای مشاهده لینک ، لطفا با نام کاربری خود وارد شوید یا ثبت نام کنید ]


بعد از محاسبه دترمینان هر دو این ماتریس ها به یک معادله یک مجهولی درجه سه می رسیم .
واقعا ممنون میشم اگر کمکم کنید .

davy jones
12-10-2010, 21:48
سلام دوستان ، خسته نباشید .
من می خوام یک مقادیر ویژه ی یک ماتریس را حساب کنم ، اما بهد از حل دترمینان به یک معادله یک مجهولی درجه سه میرسم و نمی تونم ریشه هاشو محاسبه کنم . فکر می کنم باید محاسباتی صورت بگیره که معادله درجه 3 تجزیه بشه یا اینکه ماتریس تغییر پیدا کنه قبل از عملیات دترمینان (مثلا تبدیل به ماتریس بالا مثلثی بشه بعد حل بشه) . در کل من اینارو نمی دونم . ممنون میشم کمکم کنید .


اینم ماتریس مورد نظر :



[ برای مشاهده لینک ، لطفا با نام کاربری خود وارد شوید یا ثبت نام کنید ]



این یکی هم یک ماتریس دیگه :



[ برای مشاهده لینک ، لطفا با نام کاربری خود وارد شوید یا ثبت نام کنید ]


بعد از محاسبه دترمینان هر دو این ماتریس ها به یک معادله یک مجهولی درجه سه می رسیم .
واقعا ممنون میشم اگر کمکم کنید .


برای مشاهده محتوا ، لطفا وارد شوید یا ثبت نام کنید

mohsen_blid
12-10-2010, 22:17
سلام دوستان دو سوال در رابطه با معادلات دیفرانسل داشتم که ممنون میشم جواب بدید
سوال اینه مقدار لاندار چقدر باشد تا معادله [ برای مشاهده لینک ، لطفا با نام کاربری خود وارد شوید یا ثبت نام کنید ] جواب معادله [ برای مشاهده لینک ، لطفا با نام کاربری خود وارد شوید یا ثبت نام کنید ] E2-%5Cfrac%7B1%7D%7B4%7D%29y=0 باشد؟

سوال دوم اینه که کدام گزینه معادله دیفرانسل منحنی های [ برای مشاهده لینک ، لطفا با نام کاربری خود وارد شوید یا ثبت نام کنید ] y%5E2%7D%7Bb%5E2%7D=1 است؟

الف) [ برای مشاهده لینک ، لطفا با نام کاربری خود وارد شوید یا ثبت نام کنید ]

ب) [ برای مشاهده لینک ، لطفا با نام کاربری خود وارد شوید یا ثبت نام کنید ]

ج) [ برای مشاهده لینک ، لطفا با نام کاربری خود وارد شوید یا ثبت نام کنید ] By%7D%27

د) [ برای مشاهده لینک ، لطفا با نام کاربری خود وارد شوید یا ثبت نام کنید ]


ممنون میشم در این رابطه کمک کنیدالبته با راه حل با تشکر

farzaneh*f
13-10-2010, 10:41
بچه ها منم یه سوال داشتم یه کم مبتدیه...اگه به ما fog و gof رو بدن چه طوری می تونیم f و g رو پیدا کنیم؟
سوالی که دستمه خیلی تابلوست جوابش ولی من روش حل سوالو که از کجا رسیدم به جواب نمی تونم توضیح بدم ممنون می شم اگه کمک کنین :20:

davy jones
13-10-2010, 15:05
سلام دوستان دو سوال در رابطه با معادلات دیفرانسل داشتم که ممنون میشم جواب بدید
سوال اینه مقدار لاندار چقدر باشد تا معادله [ برای مشاهده لینک ، لطفا با نام کاربری خود وارد شوید یا ثبت نام کنید ] جواب معادله [ برای مشاهده لینک ، لطفا با نام کاربری خود وارد شوید یا ثبت نام کنید ] E2-%5Cfrac%7B1%7D%7B4%7D%29y=0 باشد؟



بفرما جواب سوال اول:

[ برای مشاهده لینک ، لطفا با نام کاربری خود وارد شوید یا ثبت نام کنید ] 29%5Crightarrow&space;%7B%5Ccolor%7Bgreen%7D&space;%7By%7D%27= %5Clambda&space;x%5E%7B%5Clambda-1&space;%7Dcos%28x%29-x%5E%7B%5Clambda&space;%7Dsin%28x%29%7D%5Crightarrow&space;%7B %5Ccolor%7Bblue%7D&space;%7By%7D%27%27%7D=%5Clambda%28%5 Clambda&space;-1%29&space;x%5E%7B%5Clambda-2&space;%7Dcos%28x%29-%5Clambda&space;x%5E%7B%5Clambda-1&space;%7Dsin%28x%29-%5Clambda&space;x%5E%7B%5Clambda-1&space;%7Dsin%28x%29-&space;x%5E%7B%5Clambda&space;%7Dcos%28x%29=%7B%5Ccolor%7Bblue %7D&space;%5Clambda%28%5Clambda&space;-1%29&space;x%5E%7B%5Clambda-2&space;%7Dcos%28x%29-2%5Clambda&space;x%5E%7B%5Clambda-1&space;%7Dsin%28x%29-&space;x%5E%7B%5Clambda&space;%7Dcos%28x%29%7D%5CRightarrow&space;%7 B%5Ccolor%7Bred%7D&space;x%5E%7B2%7D%7By%7D%27%27&plus;x%7By% 7D%27&plus;%28x%5E%7B2%7D-%5Cfrac%7B1%7D%7B4%7D%29y%7D=%5Clambda%28%5Clambda &space;-1%29&space;x%5E%7B%5Clambda&space;%7Dcos%28x%29-2%5Clambda&space;x%5E%7B%5Clambda&plus;1&space;%7Dsin%28x%29-&space;x%5E%7B%5Clambda&space;&plus;2%7Dcos%28x%29&plus;%5Clambda&space;x%5E%7 B%5Clambda&space;%7Dcos%28x%29-x%5E%7B%5Clambda&space;&plus;1%7Dsin%28x%29&plus;x%5E%7B%5Clambda&space; &plus;2%7Dcos%28x%29-%5Cfrac%7B1%7D%7B4%7Dx%5E%7B%5Clambda&space;%7Dcos%28x%2 9=x%5E%7B%5Clambda&space;&plus;1%7Dsin%28x%29[1-2%5Clambda&space;]&plus;x%5E%7B%5Clambda&space;%7Dcos%28x%29[%5Clambda&space;%28%5Clambda&space;-1%29&plus;%5Clambda&space;-%5Cfrac%7B1%7D%7B4%7D]=0%5CRightarrow&space;%5Cleft%5C%7B%5Cbegin%7Bmatrix%7D&space; %5Clambda&space;%28%5Clambda&space;-1%29&plus;%5Clambda&space;-%5Cfrac%7B1%7D%7B4%7D=0%5CRightarrow&space;%5Clambda&space;%5E %7B2%7D-%5Cfrac%7B1%7D%7B4%7D=0%5CRightarrow&space;%5Clambda&space;=%5 Cpm&space;%5Cfrac%7B1%7D%7B2%7D%5C%5C&space;1-2%5Clambda&space;=0%5CRightarrow&space;%5Clambda&space;=%5Cfrac%7B1% 7D%7B2%7D&space;%5Cend%7Bmatrix%7D%5Cright.%5CRightarrow &space;%7B%5Ccolor%7Bred%7D&space;%5Clambda&space;=&plus;%5Cfrac%7B1%7D%7 B2%7D%7D


سوال دوم در دست حله:31:

-------------------


بچه ها منم یه سوال داشتم یه کم مبتدیه...اگه به ما fog و gof رو بدن چه طوری می تونیم f و g رو پیدا کنیم؟
سوالی که دستمه خیلی تابلوست جوابش ولی من روش حل سوالو که از کجا رسیدم به جواب نمی تونم توضیح بدم ممنون می شم اگه کمک کنین [ برای مشاهده لینک ، لطفا با نام کاربری خود وارد شوید یا ثبت نام کنید ]


باید حداقل یکی از توابع f یا g رو به ما بدن و ازمون بخوان که دومی رو پیدا کنیم. هر دو تاش همزمان مجهول باشه نمیشه حل کرد.

------------

موفق باشین.
89/7/21

farzaneh*f
13-10-2010, 17:14
باید حداقل یکی از توابع f یا g رو به ما بدن و ازمون بخوان که دومی رو پیدا کنیم. هر دو تاش همزمان مجهول باشه نمیشه حل کرد.

موفق باشین.
89/7/21

ممنون :11:
آخه نداده هیچ کدومو فقط گفته fog(x) = |sinx| , gof(x) = (sin _/x)^2
بعدش f و g رو ازمون خواسته (اونجا /_ رو به جای رادیکال گذاشتم که نداشتمش و ^ هم علامت توان هستش) من خودم از رو این تابعا یکی رو رادیکل ایکس و اون یکی رو سینوس ایکس به توان 2 آوردم ولی راه حل ندارم بیشتر می شه گفت به روش آزمون و خطا حلش کردم یعنی می گین هیچ راه حلی نداره ؟:41:

lebesgue
13-10-2010, 22:30
ممنون :11:
آخه نداده هیچ کدومو فقط گفته fog(x) = |sinx| , gof(x) = (sin _/x)^2
بعدش f و g رو ازمون خواسته (اونجا /_ رو به جای رادیکال گذاشتم که نداشتمش و ^ هم علامت توان هستش) من خودم از رو این تابعا یکی رو رادیکل ایکس و اون یکی رو سینوس ایکس به توان 2 آوردم ولی راه حل ندارم بیشتر می شه گفت به روش آزمون و خطا حلش کردم یعنی می گین هیچ راه حلی نداره ؟:41:

اگر gof و fog رو داشته باشید، تضمینی نیست که بتونید f و g رو بطور منحصر به فرد پیدا کنید. *
مثلا در همین نمونه شما، علاوه بر جواب شما، یک جواب دیگه هم میشه ارائه کرد:

[ برای مشاهده لینک ، لطفا با نام کاربری خود وارد شوید یا ثبت نام کنید ](x)=%5Cleft%20|%20sin(x)%20%5Cr ight%20|,gof(x)=sin^{2}(%5Csqrt{x})%5C%5C%5C%5C%20 f(x)=%5Csqrt{x},g(x)=sin^{2}(x)%5C%5C%5C%5C%20f(x) =%5Cleft%20|%20sin(%5Csqrt{x})%20%5Cright%20|,g(x) =x^{2}

* به نظر شما هیچ گاه نمی توان f و g را به طور منحصر به فرد تعیین کرد، یا اینکه تحت شرایط خاصی می توان؟

mohsen_blid
14-10-2010, 17:02
دیوی جونز عزیز سوال دوم رو تونستی حل کنی؟
بابت سوال اول ممنون

lebesgue
14-10-2010, 18:18
سوال دوم اینه که کدام گزینه معادله دیفرانسل منحنی های [ برای مشاهده لینک ، لطفا با نام کاربری خود وارد شوید یا ثبت نام کنید ] y%5E2%7D%7Bb%5E2%7D=1 است؟

الف) [ برای مشاهده لینک ، لطفا با نام کاربری خود وارد شوید یا ثبت نام کنید ]

ب) [ برای مشاهده لینک ، لطفا با نام کاربری خود وارد شوید یا ثبت نام کنید ]

ج) [ برای مشاهده لینک ، لطفا با نام کاربری خود وارد شوید یا ثبت نام کنید ] By%7D%27

د) [ برای مشاهده لینک ، لطفا با نام کاربری خود وارد شوید یا ثبت نام کنید ]


ممنون میشم در این رابطه کمک کنیدالبته با راه حل با تشکر

از معادله دو بار نسبت به x مشتق می گیریم، دو معادله بدست میاد:

[ برای مشاهده لینک ، لطفا با نام کاربری خود وارد شوید یا ثبت نام کنید ]{2x}{a^{2}}+%5Cfrac{2yy'}{b ^{2}}=0%5Crightarrow%20%5Cfrac{2x}{a^{2}}=-%5Cfrac{2yy'}{b^{2}}%5C%5C%5C%5C%20%5Cfrac{2}{a^{2 }}+%5Cfrac{2(yy''+y'^{2})}{b^{2}}=0%5Crightarrow%2 0%5Cfrac{2}{a^{2}}=-%5Cfrac{2(yy''+y'^{2})}{b^{2}}

طرفین دو معادله رو به هم تقسیم می کنیم:

[ برای مشاهده لینک ، لطفا با نام کاربری خود وارد شوید یا ثبت نام کنید ]{yy'}{yy''+y'^{2}}%5Crightarrow %20x(yy''+y'^{2})=yy'

mohsen_blid
15-10-2010, 13:06
سلام دوستان
ممنون بابت پاسخ
اول از 1233445566 یه سوال دارم اونم اینه که همیشه به این شکل هست که زمانی که می خوایم نسبت به x مشتق بگیریم y به صورت 'yy نشان داده می شود؟
سوال بعدیم نسبت به خطی بودن یا همگن بودن غیر این دو موضوع است
اولا خطی و غیر خطی بودن چه تاثیری در معادله داره منظورم اینه که حالا معادله دیفرانسیل ما غیر خطی چه اتفاقی در معادله میافته
البته می دونم که اگر در معادله دیفرانسیل بجای تابع در شرایط اولیه که همانا 0 هست تابعی قرار بگیره برای مثال tan , sin یا غیره تابع ما ناهمگن میشه منظور از ناهمگن چیه ؟
ممنون میشم در این رابطه ها با دو سه مثال کاربردی بهم کمک کنید

Paradise_human
15-10-2010, 15:21
سلام دوستان ....
میخواستم نمایش های مختلف این تابع مختلط رو برام بنویسید منظورم بر حسب (u(x,y و ( v(x,y

[ برای مشاهده لینک ، لطفا با نام کاربری خود وارد شوید یا ثبت نام کنید ]

و دوم مشتق پذیری این دو تابع مختلط رو هم بررسی کنید :


[ برای مشاهده لینک ، لطفا با نام کاربری خود وارد شوید یا ثبت نام کنید ]


این دو تابع از طریق معادلات کوشی ریمان قابل حل هستند یا نه ؟

ممنون.

alihn
15-10-2010, 15:29
تا حالا شده یه سوال در مورد مت لب داشته باشی و توی 4 تا کتاب برنامه نویسی مت لب دنبالش بگردی و کلاس آموزشیش رو هم بری و نتونی جوابشو پیدا کنی ؟
این الان مشکل منه .... کی میتونه یه کمکی بکنه ؟
من میخوام با مت لب برنامه بنویسم و اجازه استفاده از فانکشن های آماده مت لب رو هم ندارم یعنی خودم باید اونا رو بنویسم
چجوری میشه متغیرهارو به برنامه معرفی کرد که کاربری که بعدا از برنامه استفاده میکنه برنامه ازش یه ماتریس n*n رو بگیره و مقدار n رو هم همون کاربر تعیین کنه و بعد برنامه بیاد روی اون ماتریس یه کارایی بکنه مثلا با یه ماتریس دیگه جکع کنه یا معکوسشو حساب کنه (بدون استفاده از فانکشن آماده محاسبه معکوس مت لب )
بعد یه جای دیگه برنامه بیاد نتیجه این مرحله رو بازخونی کنه و باهاش یه کارای دیگه کنه و بعد نتیجه رو نشون کاربر بده (کاربری که از برنامه من استفاده میکنه فقط چیزایی رو که برنامه من ازش بخواد تایپ میکنه و به برنامه میده و در حقیقت چیزی از مت لب سرش نمیشه )
خلاصه کنم من فقط توی پنجره editor میتونم چیزی رو تایپ کنم و کلا باید فراموش کنم که پنجره ای به اسم command window هم وجود داره (برخلاف کتابا که همش و همش کار کردن توی پنجره command window رو آموزش میدن)
امیدوارم تونسته باشم مشکل رو درست توضیح بدم
کسی هست مارو از این چاله در بیاره؟

davy jones
15-10-2010, 17:23
تا حالا شده یه سوال در مورد مت لب داشته باشی و توی 4 تا کتاب برنامه نویسی مت لب دنبالش بگردی و کلاس آموزشیش رو هم بری و نتونی جوابشو پیدا کنی ؟
این الان مشکل منه .... کی میتونه یه کمکی بکنه ؟
من میخوام با مت لب برنامه بنویسم و اجازه استفاده از فانکشن های آماده مت لب رو هم ندارم یعنی خودم باید اونا رو بنویسم
چجوری میشه متغیرهارو به برنامه معرفی کرد که کاربری که بعدا از برنامه استفاده میکنه برنامه ازش یه ماتریس n*n رو بگیره و مقدار n رو هم همون کاربر تعیین کنه و بعد برنامه بیاد روی اون ماتریس یه کارایی بکنه مثلا با یه ماتریس دیگه جکع کنه یا معکوسشو حساب کنه (بدون استفاده از فانکشن آماده محاسبه معکوس مت لب )
بعد یه جای دیگه برنامه بیاد نتیجه این مرحله رو بازخونی کنه و باهاش یه کارای دیگه کنه و بعد نتیجه رو نشون کاربر بده (کاربری که از برنامه من استفاده میکنه فقط چیزایی رو که برنامه من ازش بخواد تایپ میکنه و به برنامه میده و در حقیقت چیزی از مت لب سرش نمیشه )
خلاصه کنم من فقط توی پنجره editor میتونم چیزی رو تایپ کنم و کلا باید فراموش کنم که پنجره ای به اسم command window هم وجود داره (برخلاف کتابا که همش و همش کار کردن توی پنجره command window رو آموزش میدن)
امیدوارم تونسته باشم مشکل رو درست توضیح بدم
کسی هست مارو از این چاله در بیاره؟

این چیزی که شما میخوای بیشتر شبیه برنامه نویسی به زبان ##C هستش تا متلب. پیشنهاد میکنم تا یه سری به کتابهای آموزشی برناممه نویسی با زبان C بزنین.

موفق باشین.
89/7/23

alihn
16-10-2010, 17:31
این چیزی که شما میخوای بیشتر شبیه برنامه نویسی به زبان ##C هستش تا متلب. پیشنهاد میکنم تا یه سری به کتابهای آموزشی برناممه نویسی با زبان C بزنین.

موفق باشین.
89/7/23

مرسی عزیز
من به برنامه نویسی به زبان C و پاسکال و دلفی واردم
اما استادمون گفته باید اینا رو با مت لب بنویسیم و البته میدونم با مت لب میشه این برنامه ها نوشت مثل C و شکل حلقه ها و دستورات شرطی و امثال اینو هم میدونم تو مت لب چجوریه ولی شکل کلی برنامه تو مت لب رو نمیدونم و اینکه چجوری متغیرا تعریف میشن و چجوری فانکشنا بازخوانی میشن

mahsa1469
16-10-2010, 18:48
سلام دوستان می شه این سوال رو حل کنید

ثابت کنبد نیمساز هر زاویه ی خارجی مثلث ضلع روبه روی آن زاویه را به نسبت دو ضلع آن زاویه تقسیم می کند..
بچه ها من حتی متوجه نمی شم منظورش چیه می شه یه شکل ازش برام بکشید؟

mohsen_blid
16-10-2010, 22:32
سلام دوستان
سوال
جواب عمومی معادله دیفرانسیل را هنگامی که به x بزرگتر از صفر باشد را بیابید
[ برای مشاهده لینک ، لطفا با نام کاربری خود وارد شوید یا ثبت نام کنید ] %5CRightarrow%20x%3E0


بعد یه سوال در رابطه با مول x دارم اونم اینه که چطور این تغییر ایجاد میشه

[ برای مشاهده لینک ، لطفا با نام کاربری خود وارد شوید یا ثبت نام کنید ] w%20%7B%5Ccolor%7Bred%7D%20e%5E3%5Ex%7D

Lady_Elena17
17-10-2010, 09:17
سلام
اگه میشه کمک کنید اینا حل شه


اثبات کنید lim sinx/x=1 وقتیکه xبسمت صفر میل کند

davy jones
17-10-2010, 15:40
مرسی عزیز
من به برنامه نویسی به زبان C و پاسکال و دلفی واردم
اما استادمون گفته باید اینا رو با مت لب بنویسیم و البته میدونم با مت لب میشه این برنامه ها نوشت مثل C و شکل حلقه ها و دستورات شرطی و امثال اینو هم میدونم تو مت لب چجوریه ولی شکل کلی برنامه تو مت لب رو نمیدونم و اینکه چجوری متغیرا تعریف میشن و چجوری فانکشنا بازخوانی میشن

عینا مث برنامه نویسی با C. شما چطوری تو C تابع تعریف و call میکردی؟ اگه برنامه ات رو تو متلب، کپی پیست کنی باید کار کنه. زمانی که من ترم دو، تو دانشگاه درس برنامه نویسی C رو برداشته بودم، پروژه ی آخر ترممون این بود که یه جور نرم افزار متلب رو با زبون C بنویسیم (البته به طور نسبتا مقدماتی) جوری که محاسبات ماتریسی و رسم نمودارهای دو بعدی و سه بعدی هم بشه باهاش انجام داد:18::27:

------------------


سلام دوستان می شه این سوال رو حل کنید

ثابت کنبد نیمساز هر زاویه ی خارجی مثلث ضلع روبه روی آن زاویه را به نسبت دو ضلع آن زاویه تقسیم می کند..
بچه ها من حتی متوجه نمی شم منظورش چیه می شه یه شکل ازش برام بکشید؟


سلام
اصلا نیمساز زاویه ی خارجی، ضلع روبروی اون زاویه رو قطع نمیکنه که حالا بیایم و نسبتش رو حساب کنیم؟ مطمئنین سوال درسته؟

----------------


سلام دوستان
سوال
جواب عمومی معادله دیفرانسیل را هنگامی که به x بزرگتر از صفر باشد را بیابید
[ برای مشاهده لینک ، لطفا با نام کاربری خود وارد شوید یا ثبت نام کنید ] %5CRightarrow%20x%3E0


بعد یه سوال در رابطه با مول x دارم اونم اینه که چطور این تغییر ایجاد میشه

[ برای مشاهده لینک ، لطفا با نام کاربری خود وارد شوید یا ثبت نام کنید ] w%20%7B%5Ccolor%7Bred%7D%20e%5E3%5Ex%7D


سلام.
برای پیدا کردن جواب عمومی باید ابتدا جواب معادله همگن رو پیدا کنیم، بعد یه جواب خصوصی برای معادله ناهمگن پیدا کنیم و اونها رو با هم جمع کنیم. پس یعنی ابتدا باید این معادله رو حل کنیم:


[ برای مشاهده لینک ، لطفا با نام کاربری خود وارد شوید یا ثبت نام کنید ]

برای حل این مساله به طور کلی ابتدا معادله رو به این صورت فرض میکنیم (دارم راه کلی شو میگم که دیگه برای همیشه ملکه ی ذهنت بشه):


[ برای مشاهده لینک ، لطفا با نام کاربری خود وارد شوید یا ثبت نام کنید ]


قدم اول: محاسبه ی انتگرال روبرو:

[ برای مشاهده لینک ، لطفا با نام کاربری خود وارد شوید یا ثبت نام کنید ]

(در حقیقت مشتق تابع f همان تابع p خواهد بود)


قدم دوم: ضرب کردن عبارت روبرو در دو طرف معادله ی دیفرانسیل:

[ برای مشاهده لینک ، لطفا با نام کاربری خود وارد شوید یا ثبت نام کنید ]


در نتیجه خواهیم داشت:

[ برای مشاهده لینک ، لطفا با نام کاربری خود وارد شوید یا ثبت نام کنید ] %5E%7Bf%28x%29%7Dy=0


که این عبارت در حقیقت به صورت زیر قابل ساده سازی است (چرا؟):


[ برای مشاهده لینک ، لطفا با نام کاربری خود وارد شوید یا ثبت نام کنید ]


در نتیجه:

[ برای مشاهده لینک ، لطفا با نام کاربری خود وارد شوید یا ثبت نام کنید ] ow%20%7B%5Ccolor%7Bred%7D%20y%7D=%5Cfrac%7Bk%7D%7B e%5E%7Bf%28x%29%7D%7D=%7B%5Ccolor%7Bred%7D%20ke%5E %7B-f%28x%29%7D%7D


که k در اینجا هر عدد حقیقی میتونه باشه. در سوالی که شما پرسیدین داریم p(x)=n/x. (بقیه ی محاسباتش به عهده ی خود شما)

خب، این از جواب معادله ی همگن!

برای بدست آوردن یک جواب خصوصی هم بسته به اینکه تابع p چی باشه جواب فرق میکنه و معمولا هم قاعده ی خاصی وجود نداره. مثلا در این مثال شما، به ازای: [ برای مشاهده لینک ، لطفا با نام کاربری خود وارد شوید یا ثبت نام کنید ] 5C;%20n=-k یک جواب خصوصی معادله [ برای مشاهده لینک ، لطفا با نام کاربری خود وارد شوید یا ثبت نام کنید ] %5CRightarrow%20x%3E0 بدست می آید.

-----------------------------


سلام
اگه میشه کمک کنید اینا حل شه


اثبات کنید lim sinx/x=1 وقتیکه xبسمت صفر میل کند



سلام.
والا منظورتون رو از اینا متوجه نشدم:31: (این که فقط یه دونه اس:27:)
و اما جواب:
از بسط تیلور سینوس استفاده میکنیم. برای سینوس داریم:


[ برای مشاهده لینک ، لطفا با نام کاربری خود وارد شوید یا ثبت نام کنید ] 7B5%7D%7D%7B5%21%7D-%5Cfrac%7Bx%5E%7B7%7D%7D%7B7%21%7D+...


بنابراین حد بالا برابر میشه با:


[ برای مشاهده لینک ، لطفا با نام کاربری خود وارد شوید یا ثبت نام کنید ] n%20%28x%29%7D%7Bx%7D=%5Cfrac%7Bx-%5Cfrac%7Bx%5E%7B3%7D%7D%7B3%21%7D+%5Cfrac%7Bx%5E% 7B5%7D%7D%7B5%21%7D-%5Cfrac%7Bx%5E%7B7%7D%7D%7B7%21%7D+...%7D%7Bx%7D=% 5Clim_%7Bx%5Cto%200%7D%281-%5Cfrac%7Bx%5E%7B2%7D%7D%7B3%21%7D+%5Cfrac%7Bx%5E% 7B4%7D%7D%7B5%21%7D-%5Cfrac%7Bx%5E%7B6%7D%7D%7B7%21%7D+...%29=1





موفق باشین.
89/7/25

mofidy1
17-10-2010, 16:07
سلام دوستان می شه این سوال رو حل کنید

ثابت کنبد نیمساز هر زاویه ی خارجی مثلث ضلع روبه روی آن زاویه را به نسبت دو ضلع آن زاویه تقسیم می کند..
بچه ها من حتی متوجه نمی شم منظورش چیه می شه یه شکل ازش برام بکشید؟

با سلام

نیمساز هر زاویه ی داخلی است نه نیمساز هر زاویه ی خارجی، ضمناً این قضیه در کتاب هندسه ی 2 سوم ریاضی صفحه ی 13 اثبات شده است و شکلی هم از آن در همان جا هست.

موفق باشید.

25 مهر 1389

lebesgue
17-10-2010, 18:00
اول از 1233445566 یه سوال دارم اونم اینه که همیشه به این شکل هست که زمانی که می خوایم نسبت به x مشتق بگیریم y به صورت 'yy نشان داده می شود؟


مشتق [ برای مشاهده لینک ، لطفا با نام کاربری خود وارد شوید یا ثبت نام کنید ]^{2} نسبت به x برابره با [ برای مشاهده لینک ، لطفا با نام کاربری خود وارد شوید یا ثبت نام کنید ]'. این از قاعده زنجیری بدست میاد:

[ برای مشاهده لینک ، لطفا با نام کاربری خود وارد شوید یا ثبت نام کنید ]{%5Cmathrm{d}%20(y^{2})}{%5Cmathr m{d}%20x}=%5Cfrac{%5Cmathrm{d}(y^2)%20}{%5Cmathrm{ d}%20y}%5Ctimes%20%5Cfrac{%5Cmathrm{d}%20y}{%5Cmat hrm{d}%20x}=2yy'

alihn
17-10-2010, 20:09
عینا مث برنامه نویسی با C. شما چطوری تو C تابع تعریف و call میکردی؟ اگه برنامه ات رو تو متلب، کپی پیست کنی باید کار کنه. زمانی که من ترم دو، تو دانشگاه درس برنامه نویسی C رو برداشته بودم، پروژه ی آخر ترممون این بود که یه جور نرم افزار متلب رو با زبون C بنویسیم (البته به طور نسبتا مقدماتی) جوری که محاسبات ماتریسی و رسم نمودارهای دو بعدی و سه بعدی هم بشه باهاش انجام داد:18::27:


[/CENTER]

اینو امتحان کردم ولی دستورات ز رو نمیشناسه مثلا نمیفهمه scanf چیه
تازگیا با کلی گشتن نت متوجه شدم به جای scanf از input استفاده میکنه مت لب
چجوری طول ماتریسو میگیره
چجوری چاپ میکنه
اینپوت چجوری کار میکنه؟
معادل دستورای c توی مت لب چیه؟

davy jones
17-10-2010, 21:17
اینو امتحان کردم ولی دستورات ز رو نمیشناسه مثلا نمیفهمه scanf چیه
تازگیا با کلی گشتن نت متوجه شدم به جای scanf از input استفاده میکنه مت لب
چجوری طول ماتریسو میگیره
چجوری چاپ میکنه
اینپوت چجوری کار میکنه؟
معادل دستورای c توی مت لب چیه؟

input اینطوریه که مثلا وقتی می نویسین: ; input a در حقیقت یه متغیر تعریف کردین (a) که مقدارش رو از شما میگیره (وقتی برنامه رو run میکنین)

اگه همونطور که گفتین نباید از دستورات پیش فرض متلب استفاده کنین باید طول و عرض ماتریس رو هم همون ابتدا در input وارد بشه و بعد با استفاده از حلقه ی for و تعریف کردن string در درون این حلقه، درایه ها رو یکی یکی سر جاش بنشونین و مابقی عملیاتها هم به همین صورت. (به عنوان مثال واضحه که برای ضرب دو ماتریس باید از دو حلقه ی تو در تو استفاده کنین)

منظورم من از تشابه متلب با C در این بود که نحوه ی تعریف توابع و ضابطه دادن به اونها و صدا کردن این توابع در هر دو برنامه شبیه به همه.

برای نمایش خروجی هم باید به جای printf از output استفاده کنین.

موفق باشین.
89/7/25

Paradise_human
18-10-2010, 11:14
سلام دوستان ....
میخواستم نمایش های مختلف این تابع مختلط رو برام بنویسید منظورم بر حسب (u(x,y و ( v(x,y

[ برای مشاهده لینک ، لطفا با نام کاربری خود وارد شوید یا ثبت نام کنید ]



ممنون.
آقا این یکی رو حداقل یه نفر میتونه کمک بکنه ؟
بقیه اش رو دیگه نمیتونم به یک تابعی بر حسب (u(x,y و ( v(x,y تبدیل کنم:
( x-iy=f(z

davy jones
18-10-2010, 13:19
آقا این یکی رو حداقل یه نفر میتونه کمک بکنه ؟
بقیه اش رو دیگه نمیتونم به یک تابعی بر حسب (u(x,y و ( v(x,y تبدیل کنم:
( x-iy=f(z

f(z)=x-iy ==> if f(z)=u(x,y)+i*v(x,y) then u(x,y)=x and v(x,y)=-y

موفق باشین.
89/7/26

amirhoseinmehri
20-10-2010, 06:58
یه نفر این سوال رو به من داد ببینید می تونید جوابش رو پیدا کنید
الگوریتم دو عدد a و b را جابه جا کنید.
a=2 ، b=3

a=3 ، b=3

davy jones
20-10-2010, 07:25
یه نفر این سوال رو به من داد ببینید می تونید جوابش رو پیدا کنید
الگوریتم دو عدد a و b را جابه جا کنید.
a=2 ، b=3

a=3 ، b=3

سلام. یعنی منظورتون اینه که با یه الگوریتم یکسان، کاری کنیم که الگوریتم ما 2 رو به 3 ببره ولی 3 رو تغییر نده. درسته؟

موفق باشین.
89/7/28

lebesgue
20-10-2010, 11:05
یه نفر این سوال رو به من داد ببینید می تونید جوابش رو پیدا کنید
الگوریتم دو عدد a و b را جابه جا کنید.
a=2 ، b=3

a=3 ، b=3

بدون استفاده از متغیر واسطه:


a = a + b :دستور اول
b = a - b :دستور دوم
a = a - b :دستور سوم

ویرایش:
من الان مثالتون رو دیدم، منظورتون از جابجا شدن دقیقا چیه؟

amirhoseinmehri
20-10-2010, 16:34
سلام. یعنی منظورتون اینه که با یه الگوریتم یکسان، کاری کنیم که الگوریتم ما 2 رو به 3 ببره ولی 3 رو تغییر نده. درسته؟

موفق باشین.
89/7/28

ببخشید اشتباه نوشتم a=2 b=3
a=3 b=2

davy jones
20-10-2010, 17:20
ببخشید اشتباه نوشتم a=2 b=3
a=3 b=2

خب همون جوابی که دوست خوبم جناب 1233445566 دادند درسته.
البته یه راه قدیمی تر هم وجود داره که استفاده از یک متغیر کمکیه:


t=a

a=b

b=t

قله بلند
21-10-2010, 11:42
سلام
فرض کنید چنین دنباله ای از اعداد داریم. حالا می خواهیم آن را جمع کنیم. حاصل آن 3n می شود.

برای مشاهده محتوا ، لطفا وارد شوید یا ثبت نام کنیدیعنی هر گاه i= توان زوج باشد، ما به انداره i، یک ها را جمع می کنیم. یعنی برای s2 ما داریم:1+1 برای s4 داریم:1+1+1+1 و برای فقط یک عدد یک داریم.حالا اگر این جملات را جمع کنیم باید به جواب 3n برسیم.اینگونه حساب کرده است که: سیگمای 2 به قوه I در حالیکه I از صفر تا logn به پایه 2 می باشد.
اگر این سوال قبلاً پاسخ داده شده لطفاً بفرمایید کدوم پست.

davy jones
21-10-2010, 12:31
سلام
فرض کنید چنین دنباله ای از اعداد داریم. حالا می خواهیم آن را جمع کنیم. حاصل آن 3n می شود.

برای مشاهده محتوا ، لطفا وارد شوید یا ثبت نام کنیدیعنی هر گاه i= توان زوج باشد، ما به انداره i، یک ها را جمع می کنیم. یعنی برای s2 ما داریم:1+1 برای s4 داریم:1+1+1+1 و برای فقط یک عدد یک داریم.حالا اگر این جملات را جمع کنیم باید به جواب 3n برسیم.اینگونه حساب کرده است که: سیگمای 2 به قوه I در حالیکه I از صفر تا logn به پایه 2 می باشد.
اگر این سوال قبلاً پاسخ داده شده لطفاً بفرمایید کدوم پست.


مطمئنی سوال درسته؟ n رو بذار 1، حاصل برابر با 3 نمیشه.
n رو بذار 2، حاصل برابر با 3 میشه، در حالی که باید برابر با 6 بشه طبق این چیزی که شما گفتین.

سوال رو مجددا بررسی کنین.

موفق باشین.
89/7/29

m.imeni
21-10-2010, 13:35
سلام دوستان
کسی می تونه بردار یکه مماس معادله متحرک زیر رو بدست بیاره ؟


[ برای مشاهده لینک ، لطفا با نام کاربری خود وارد شوید یا ثبت نام کنید ] 29i@plus;%28e%5Et%29sin%28e%5Et%29j@plus;%28e%5Et% 29k%22%20target=%22_blank%22%3E%3Cimg%20src=%22htt p://latex.codecogs.com/gif.latex?R%28t%29=%28e%5Et%29cos%28e%5Et%29i+%28e %5Et%29sin%28e%5Et%29j+%28e%5Et%29k%22%20title=%22 R%28t%29=%28e%5Et%29cos%28e%5Et%29i+%28e%5Et%29sin %28e%5Et%29j+%28e%5Et%29k%22%20/%3E%3C/a%3E[ برای مشاهده لینک ، لطفا با نام کاربری خود وارد شوید یا ثبت نام کنید ] 29i@plus;%28e%5Et%29sin%28e%5Et%29j@plus;%28e%5Et% 29k%22%20target=%22_blank%22%3E%3Cimg%20src=%22htt p://latex.codecogs.com/gif.latex?R%28t%29=%28e%5Et%29cos%28e%5Et%29i+%28e %5Et%29sin%28e%5Et%29j+%28e%5Et%29k%22%20title=%22 R%28t%29=%28e%5Et%29cos%28e%5Et%29i+%28e%5Et%29sin %28e%5Et%29j+%28e%5Et%29k%22%20/%3E%3C/a%3E[ برای مشاهده لینک ، لطفا با نام کاربری خود وارد شوید یا ثبت نام کنید ] %5Et%29sin%28e%5Et%29j+%28e%5Et%29k

اینم فرمول محاسبه بردار یکه مماس :

[ برای مشاهده لینک ، لطفا با نام کاربری خود وارد شوید یا ثبت نام کنید ] %7D%7D%7B%5Cbegin%7BVmatrix%7D%20%5Cfrac%7BdR%7D%7 Bdt%7D%20%5Cend%7BVmatrix%7D%7D%22%20target=%22_bl ank%22%3E%3Cimg%20src=%22[ برای مشاهده لینک ، لطفا با نام کاربری خود وارد شوید یا ثبت نام کنید ] B%5Cbegin%7BVmatrix%7D%20%5Cfrac%7BdR%7D%7Bdt%7D%2 0%5Cend%7BVmatrix%7D%7D%22%20title=%22T=%5Cfrac%7B %5Cfrac%7BdR%7D%7Bdt%7D%7D%7B%5Cbegin%7BVmatrix%7D %20%5Cfrac%7BdR%7D%7Bdt%7D%20%5Cend%7BVmatrix%7D%7 D%22%20/%3E%3C/a%3E[ برای مشاهده لینک ، لطفا با نام کاربری خود وارد شوید یا ثبت نام کنید ] %7D%7D%7B%5Cbegin%7BVmatrix%7D%20%5Cfrac%7BdR%7D%7 Bdt%7D%20%5Cend%7BVmatrix%7D%7D%22%20target=%22_bl ank%22%3E%3Cimg%20src=%22[ برای مشاهده لینک ، لطفا با نام کاربری خود وارد شوید یا ثبت نام کنید ] B%5Cbegin%7BVmatrix%7D%20%5Cfrac%7BdR%7D%7Bdt%7D%2 0%5Cend%7BVmatrix%7D%7D%22%20title=%22T=%5Cfrac%7B %5Cfrac%7BdR%7D%7Bdt%7D%7D%7B%5Cbegin%7BVmatrix%7D %20%5Cfrac%7BdR%7D%7Bdt%7D%20%5Cend%7BVmatrix%7D%7 D%22%20/%3E%3C/a%3E[ برای مشاهده لینک ، لطفا با نام کاربری خود وارد شوید یا ثبت نام کنید ] B%5Cbegin%7BVmatrix%7D&space;%5Cfrac%7BdR%7D%7Bdt%7D&space;%5C end%7BVmatrix%7D%7D

قله بلند
21-10-2010, 14:52
مطمئنی سوال درسته؟ n رو بذار 1، حاصل برابر با 3 نمیشه.
n رو بذار 2، حاصل برابر با 3 میشه، در حالی که باید برابر با 6 بشه طبق این چیزی که شما گفتین.

سوال رو مجددا بررسی کنین.

موفق باشین.
89/7/29

سلام. به معادله ای که رسیده این هست:

برای مشاهده محتوا ، لطفا وارد شوید یا ثبت نام کنیدکه کران پایین سیگما، مساوی صفر و کران بالای سیگما مساوی logn هست که مبنای آن 2 است. جوابی هم که به دست اومده، 3n هست.

مساله رو هم اینجوری تعریف کرده:

Ci را اینگونه تعریف کرده است:
1-اگر i-1 توان دقیقی از 2 باشد نگاه ci=i است و در غیر این صورت عدد ثابت 1 است.

davy jones
21-10-2010, 17:02
مساله رو هم اینجوری تعریف کرده:

Ci را اینگونه تعریف کرده است:
1-اگر i-1 توان دقیقی از 2 باشد نگاه ci=i است و در غیر این صورت عدد ثابت 1 است.


بازم درست از آب در نمیآد. شما n رو بذار 3:
x=1+1+3=5
ولی طبق مساله باید جواب 9=3*3 رو به دست بیاریم.

موفق باشین.
89/7/29

قله بلند
21-10-2010, 18:37
سلام
این طرح یک مساله است.
مثلاً فرض کنید که یک جدول دو خانه ای دارید و هزینه درج در هر خانه اصولاً و در هر حالتی عدد ثابت 1 است.
1-A و B را در این دو خانه درج می کنیم. پس تا اینجا مثلاً 2 تومان هزینه کردیم.
2-می خواهیم C را درج کنیم ولی حافظه ما فقط 2 خانه داشت. حالا چه کنیم؟ فرض مساله به ما می گوید که یک حافظه 4 خانه ای اختیار کن و A و B را به آن منتقل کن و حالا C را در آن درج کن ولی چون درج C باعث اینهمه بیچارگی و آوارگی شده پس باید C را جریمه کنیم و هزینه انتقال A و B را از او بگیریم. پس تا اینجا 3 درج برای A و B و C داشتیم که شد عدد 3 و C را هم جریمه کردیم که هزینه انتقال A و B را که عدد 2 هست بپردازد. پس شد 2+3=5
3-حالا D را درج می کنیم. چون هنوز یک خانه حافظه باقی مانده است پس فقط هزیه درج برای D که همان 1 تومان است پرداخت می شود.
4-حالا فرض کنید که می خواهیم E را درج کنیم. دوباره همان بلایی که سر درج C امد، سر E هم می آوریم. ابتدا حافظه را دو برابر کرده و A و B و C و D را به حافظه 8 تایی منتقل می کنیم و هزینه این 4 انتقال را از جیب جناب E می پردازیم و سپس در ج E را انجام می دهیم.
و الی آخر

همانطور که می بینید اگر عمل i باعث شود که سایز جدول دو برابر شود و عمل انتقال جدول قبلی به جدول بزرگ شده انجام شود، هزینه پرداختی دقیقاً عدد i است. مثلاً عمل درج C که سومین عمل بود منجر به پرداخت هزینه 3 تومان شد و عمل درج E که پنجمین عمل بود منجر به پرداخت هزینه 5 تومان شد ولی بقیه فقط 1 تومان هزینه کردند.
حالا فرض کنید به همین منوال کار را ادامه دهیم.
الف-همه مجبورند هزینه 1 تومان را که هزینه درج است بپردازند پس مجموع n عملیاتی یعنی مجموع n تا 1 تومان که می شود n
ب-ولی فقط انهایی مجبورند هزینه اضافی بپردازند که با جدول پر مواجه می شوند. مثلاض درج C باعث می شود که 2 خانه جدول به جدول 4 خانه ای مبدل شود پس باید 2 هزینه اضافی دیگر برای انتقال A و B به جدول 4 خانه ای بپردازد.
حالا جمع اینهایی که توان 2 می سازند می شود:

برای مشاهده محتوا ، لطفا وارد شوید یا ثبت نام کنید
که کران پایین رو 0 و کران بالا رو مقدار صحیح logn فرض می کند. من نمی تونم از این سری به عدد 2n برسم چون این سری واگراست.

قله بلند
21-10-2010, 20:32
سلام

من فرض می کنم که قراره 32 بار این عمل رو انجام بدم. در این 32 عمل درج، فقط 5 عمل درج است که باعث می شود هزینه های تنبیهی به وجود آید. که مجموع این هزینه های تنبیهی می شود 32+(16+8+4+2+1) که مجموع داخل پرانتز تقریباً می شود 32 تا.

خوب ما 32 تا عمل درج داشتیم و حالا مجموع این سیگما می شد 2 تا 32 تا یعنی 2n که با n اول می شود 3n.

جناب davy jones عزیز، صحبت شما درسته ولی شاید یه جوری هست که می شه از این اختلاف ها صرفنظر کرد. شاید در نهایت اونقدر زیاد نمی شه که مشکل ساز بشه.

ولی هنوز برای من این سوال مطرحه که چه جوری باید این سری واگرا رو حساب کرد که با فرمول در بیاد نه با حساب و کتاب.

davy jones
21-10-2010, 22:49
سلام

من فرض می کنم که قراره 32 بار این عمل رو انجام بدم. در این 32 عمل درج، فقط 5 عمل درج است که باعث می شود هزینه های تنبیهی به وجود آید. که مجموع این هزینه های تنبیهی می شود 32+(16+8+4+2+1) که مجموع داخل پرانتز تقریباً می شود 32 تا.

خوب ما 32 تا عمل درج داشتیم و حالا مجموع این سیگما می شد 2 تا 32 تا یعنی 2n که با n اول می شود 3n.

جناب davy jones عزیز، صحبت شما درسته ولی شاید یه جوری هست که می شه از این اختلاف ها صرفنظر کرد. شاید در نهایت اونقدر زیاد نمی شه که مشکل ساز بشه.

ولی هنوز برای من این سوال مطرحه که چه جوری باید این سری واگرا رو حساب کرد که با فرمول در بیاد نه با حساب و کتاب.

سلام.
مهم نیست که این سری واگراست. ما که نمیخوایم مجموع جملات رو تا بینهایت حساب کنیم.
در حالت کلی برای سریهای هندسی (توانی) داریم:

[ برای مشاهده لینک ، لطفا با نام کاربری خود وارد شوید یا ثبت نام کنید ] n%7Dx%5E%7Bi%7D=1&plus;x&plus;x%5E%7B2%7D&plus;x%5E%7B3%7D&plus;x%5E%7 B4%7D&plus;...&plus;x%5E%7Bn%7D=%5Cfrac%7B%281-x%29%281&plus;x&plus;x%5E%7B2%7D&plus;x%5E%7B3%7D&plus;x%5E%7B4%7D&plus;... &plus;x%5E%7Bn%7D%29%7D%7B1-x%7D=%7B%5Ccolor%7Bred%7D&space;%5Cfrac%7B1-x%5E%7Bn&plus;1%7D%7D%7B1-x%7D%7D

که اگه اینجا به جای x عدد 2 و به جای n قرار بدیم [ برای مشاهده لینک ، لطفا با نام کاربری خود وارد شوید یا ثبت نام کنید ]، اونوقت حاصل سیگما برابر میشه با


[ برای مشاهده لینک ، لطفا با نام کاربری خود وارد شوید یا ثبت نام کنید ]


که البته با جواب مورد انتظار ما یک واحد تفاوت داره و جمع کل برابر میشه با 3n-1.

موفق باشین.
89/7/29

قله بلند
22-10-2010, 00:22
واقعاً جالب بود. دستتون درد نکنه.

davy jones
22-10-2010, 11:17
واقعاً جالب بود. دستتون درد نکنه.

البته ظاهرا یه جاشو اشتباه کردم. 2 به توان لوگاریتم n (به پایه ی 2) برابر با خود n میشه که اشتباها نوشتم 2n بنابراین حاصل سیگما برابر با n-1 میشه و حاصل کل برابر با 2n-1.

موفق باشین.
89/7/30

قله بلند
22-10-2010, 13:03
سلام. درست نوشتید چون یک عدد 2 در n ضرب می شه و حاصل صورت می شه
برای مشاهده محتوا ، لطفا وارد شوید یا ثبت نام کنیدو در مخرج هم داریم 1- که روی هم رفته حاصل سیگما می شود
برای مشاهده محتوا ، لطفا وارد شوید یا ثبت نام کنیدحالا یک n هم که از قبل داشتیم که جمع کل می شود
برای مشاهده محتوا ، لطفا وارد شوید یا ثبت نام کنید

Life24
22-10-2010, 15:13
اقا ک . م . م یک عدد چطوری میگیرند؟
ب . م .م چطور؟

میخوام الگوریتم رسم کنم
اصلا نمیدونم تاین دو تا چیه/.
ب. م . م 36 و 18 مشترکش میشه 9؟ یا 6؟
مگر نمیکن بزرگ ترین مقسوم علیه مشترک
خوب 9 میشه

قله بلند
22-10-2010, 16:04
سلام آدرس زیر و نگاه کن. به صورت آن لاین به چهار روش برات ب م م رو حساب می کنه.
برای مشاهده محتوا ، لطفا وارد شوید یا ثبت نام کنید اصولاً ب م م یعنی بزرگترین مقسوم علیه ای که بین دو عدد مشترک هست. برای عدد شما 18 هست. مثلاً در روش نردبانی:یک نردبان را به صورت افقی فرض کن. توی خونه اول نردبان عدد 36 و در خانه دوم نردبان عدد 18 را بگذار. حالا چند تا 18 تا حساب کنیم تا از 36 کمتر یا با 36 مساوی بشه. در اینجا دقیقاً 2 تا 18 تا می خواهیم. عدد 2 را بالای سر 18 در نردبان می گذاریم و باقیمانده را در خانه سوم نردبان که در اینجا صفر هست و محاسبه خاتمه می یابد.برای ک م م که کوچکترین مضرب مشترک هست، ابتدا دو عدد را ضرب می کنیم و بر ب م م این دو عدد تقسیم می کنیم. یعنی 36*18/18 که می شود 36.آدرس زیر رو هم ببینید. بامزه توضیح داده:
برای مشاهده محتوا ، لطفا وارد شوید یا ثبت نام کنید

davy jones
23-10-2010, 10:54
سلام. درست نوشتید چون یک عدد 2 در n ضرب می شه و حاصل صورت می شه
برای مشاهده محتوا ، لطفا وارد شوید یا ثبت نام کنیدو در مخرج هم داریم 1- که روی هم رفته حاصل سیگما می شود
برای مشاهده محتوا ، لطفا وارد شوید یا ثبت نام کنیدحالا یک n هم که از قبل داشتیم که جمع کل می شود
برای مشاهده محتوا ، لطفا وارد شوید یا ثبت نام کنید

سلام. جواب اولیه ی من غلطه و اصلاحیه ای که نوشتم درسته. زیرا که به ازای هر x>0 و A>0 ، داریم:


[ برای مشاهده لینک ، لطفا با نام کاربری خود وارد شوید یا ثبت نام کنید ] %28x%29%29%7D=x


که البته نباید A مساوی با یک باشد.
در اینجا ما به ازای A عدد 2 و به ازای x مقدار n را قرار بدهیم به جواب اشاره شده میرسیم.
بنابراین حاصل سیگما برابر با n-1 میشه و حاصل جواب کل مساله برابر با 2n-1 میشه.

موفق باشین.
89/8/1

قله بلند
23-10-2010, 18:37
سلام
پس تکلیف بودن عدد 1 که با logn جمع شده و شما برای راحتی محاسبه، اون ها رو از هم جدا کردید و شد
برای مشاهده محتوا ، لطفا وارد شوید یا ثبت نام کنیدچی می شه؟ درسته؟

Life24
23-10-2010, 18:55
سلام آدرس زیر و نگاه کن. به صورت آن لاین به چهار روش برات ب م م رو حساب می کنه.
برای مشاهده محتوا ، لطفا وارد شوید یا ثبت نام کنید اصولاً ب م م یعنی بزرگترین مقسوم علیه ای که بین دو عدد مشترک هست. برای عدد شما 18 هست. مثلاً در روش نردبانی:یک نردبان را به صورت افقی فرض کن. توی خونه اول نردبان عدد 36 و در خانه دوم نردبان عدد 18 را بگذار. حالا چند تا 18 تا حساب کنیم تا از 36 کمتر یا با 36 مساوی بشه. در اینجا دقیقاً 2 تا 18 تا می خواهیم. عدد 2 را بالای سر 18 در نردبان می گذاریم و باقیمانده را در خانه سوم نردبان که در اینجا صفر هست و محاسبه خاتمه می یابد.برای ک م م که کوچکترین مضرب مشترک هست، ابتدا دو عدد را ضرب می کنیم و بر ب م م این دو عدد تقسیم می کنیم. یعنی 36*18/18 که می شود 36.آدرس زیر رو هم ببینید. بامزه توضیح داده:
برای مشاهده محتوا ، لطفا وارد شوید یا ثبت نام کنید
خوب پس
ب. م . م 36 و18 میشه 9
این طرف به ما گفت 6
:2:

davy jones
23-10-2010, 20:24
سلام
پس تکلیف بودن عدد 1 که با logn جمع شده و شما برای راحتی محاسبه، اون ها رو از هم جدا کردید و شد
برای مشاهده محتوا ، لطفا وارد شوید یا ثبت نام کنیدچی می شه؟ درسته؟

راست میگی. حق با شماست. حواسم به اون 2 نبود. اشتباه از من بود.

موفق باشین.
89/8/1

ehsan-inventor
23-10-2010, 21:52
سلام دوستان:11:
من می خوام دستگاه معادلات غیرخطی شامل 5 معادله و 5 مجهول رو با متلب حل کنم...
باید تو M-file برنامه بنویسم؟ یا نه با دستورات خود متلب هم حله؟ من یه خرده با متلب آشنایی دارم.یه کتاب گرفتم و اونو شروع کردم(متلب برای مهندسان مکانیک حامد منتظری) ولی نمیرسم اینا رو تا 4شنبه یاد بگیرم...
اگر از دوستان کسی هست که بلد باشه ممنون میشم راهنمایی کنه و اگر هم دستورشو بده که خیلی بهتر میشه...
اینم معادله:

A*cos(x+Φ)-B*cos(y+Ψ)+C=cos(x+Φ-y-Ψ)

حالا به ازای پنج مقدارX پنج مقدار Y دارم که توی پنج معادله می ذارم و در نهایت پنج مجهول دارم(AوBوCوΨوΦ)
خلاصه که پنج معادله دارم و پنج مجهول...

البته بعد از این هم باید 11 معادله و 5 مجهول رو حل کنم که استادمون گفت باید بهینه سازی کنید !
این بهینه سازی چطوریه با این 11 معادله؟ باید توM-file برنامه بنویسم؟
ممنون میشم راهنمایی کنید.
موفق باشید:11:

Life24
24-10-2010, 12:36
سلام آدرس زیر و نگاه کن. به صورت آن لاین به چهار روش برات ب م م رو حساب می کنه.
برای مشاهده محتوا ، لطفا وارد شوید یا ثبت نام کنید اصولاً ب م م یعنی بزرگترین مقسوم علیه ای که بین دو عدد مشترک هست. برای عدد شما 18 هست. مثلاً در روش نردبانی:یک نردبان را به صورت افقی فرض کن. توی خونه اول نردبان عدد 36 و در خانه دوم نردبان عدد 18 را بگذار. حالا چند تا 18 تا حساب کنیم تا از 36 کمتر یا با 36 مساوی بشه. در اینجا دقیقاً 2 تا 18 تا می خواهیم. عدد 2 را بالای سر 18 در نردبان می گذاریم و باقیمانده را در خانه سوم نردبان که در اینجا صفر هست و محاسبه خاتمه می یابد.برای ک م م که کوچکترین مضرب مشترک هست، ابتدا دو عدد را ضرب می کنیم و بر ب م م این دو عدد تقسیم می کنیم. یعنی 36*18/18 که می شود 36.آدرس زیر رو هم ببینید. بامزه توضیح داده:
برای مشاهده محتوا ، لطفا وارد شوید یا ثبت نام کنید

اون تقسیم که نشون داد به دردم خورد
و تونستم حل کنم مسئله رو
ایول :11:

قله بلند
25-10-2010, 00:03
خوب پس
ب. م . م 36 و18 میشه 9
این طرف به ما گفت 6
:2:

سلام. ب م م یعنی بزرگترین مقسوم علیه مشترک. 9 هم میتونه باشه ولی بزرگترین نیست. مثلاً 36 بر 9 قابل قسمت هست و بر 18 هم هست و بر 12 هم هست. عدد 18 بر 9 و 3 و 2 و.. قابل قسمت هست و بر خود 18 هم قابل قسمت هست. عدد 36 هم بر خودش قابل قسمت هست ولی 18 بر 36 قابل قسمت نیست. پس بزرگترین عددی که بتوان آن دو عدد را بر آن بخش کرد در این مثال 18 است.

قله بلند
25-10-2010, 00:11
سلام دوستان:11:
من می خوام دستگاه معادلات غیرخطی شامل 5 معادله و 5 مجهول رو با متلب حل کنم...
باید تو M-file برنامه بنویسم؟ یا نه با دستورات خود متلب هم حله؟ من یه خرده با متلب آشنایی دارم.یه کتاب گرفتم و اونو شروع کردم(متلب برای مهندسان مکانیک حامد منتظری) ولی نمیرسم اینا رو تا 4شنبه یاد بگیرم...
اگر از دوستان کسی هست که بلد باشه ممنون میشم راهنمایی کنه و اگر هم دستورشو بده که خیلی بهتر میشه...
اینم معادله:

A*cos(x+Φ)-B*cos(y+Ψ)+C=cos(x+Φ-y-Ψ)

حالا به ازای پنج مقدارX پنج مقدار Y دارم که توی پنج معادله می ذارم و در نهایت پنج مجهول دارم(AوBوCوΨوΦ)
خلاصه که پنج معادله دارم و پنج مجهول...

البته بعد از این هم باید 11 معادله و 5 مجهول رو حل کنم که استادمون گفت باید بهینه سازی کنید !
این بهینه سازی چطوریه با این 11 معادله؟ باید توM-file برنامه بنویسم؟
ممنون میشم راهنمایی کنید.
موفق باشید:11:

سلام. من با این نرم افزار کار نکردم ولی راجع به سوال بهینه سازی باید بگم که هر کاری که باعث بشه، کار ما مطلوب تر انجام بشه رو بهینه سازی می گن. مثلاً به جای اینکه شما با 5 ضرب و جمع و منها و تقسیم به جواب نهایی برسید، راهی پیدا کنید که با 3 گام این مساله رو حل کنید. اینجوری بار ناشی از جمع و ضرب و ... کمتر می شه و شما راه بهینه تر و بهتری رو برای حل مساله پیدا کردید.
حالا چون این مساله خیلی می تونه معادله داشته باشه، شما با 3 معادلهف 3 مجهول یا 4 معادله 4 مجهول این بهینه سازی رو امتحان کنید و ببینید می تونید راه حلی پیدا کنید که با محاسبات کمتری به جواب نهایی برسید.

Life24
27-10-2010, 19:15
میگم عزیزان یک سوال
اسون هست ها اما من هر کاری میکنم نمیشه

یک تقسیم بر x-1
منهای یک تقسیم بر a-1
تقسیم بر

x-a

یعنی شد
یک تقسیم بر ایکس منهای یک بعد علامت منفی داریم وسطش بعد یک تقسیم بر aمنهای یک
تقسیم بر x-a

میخوام ساده اش کنم طوری که یک x-a ازش در بیاد که با مخرج بزنمش
یا حداقل نزدیک بشه
اصلا یک جوری اسده بشه این صورت کسر:41:

davy jones
27-10-2010, 19:19
میگم عزیزان یک سوال
اسون هست ها اما من هر کاری میکنم نمیشه

یک تقسیم بر x-1
منهای یک تقسیم بر a-1
تقسیم بر

x-a

یعنی شد
یک تقسیم بر ایکس منهای یک بعد علامت منفی داریم وسطش بعد یک تقسیم بر aمنهای یک
تقسیم بر x-a

میخوام ساده اش کنم طوری که یک x-a ازش در بیاد که با مخرج بزنمش
یا حداقل نزدیک بشه
اصلا یک جوری اسده بشه این صورت کسر:41:

سلام.
منظورتون اینه؟

[ برای مشاهده لینک ، لطفا با نام کاربری خود وارد شوید یا ثبت نام کنید ] %7D%7Bx-1%7D-%5Cfrac%7B1%7D%7Ba-1%7D%7D%7Bx-a%7D


اگه منظورتون همینه که سوال خیلی ساده ایه. بفرمایین:


[ برای مشاهده لینک ، لطفا با نام کاربری خود وارد شوید یا ثبت نام کنید ] %7D%7Bx-1%7D-%5Cfrac%7B1%7D%7Ba-1%7D%7D%7Bx-a%7D=%5Cfrac%7B%5Cfrac%7Ba-1-%28x-1%29%7D%7B%28x-1%29%28a-1%29%7D%7D%7Bx-a%7D=%5Cfrac%7B%5Cfrac%7Ba-x%7D%7B%28x-1%29%28a-1%29%7D%7D%7Bx-a%7D=%5Cfrac%7B-1%7D%7B%28x-1%29%28a-1%29%7D





ضمنا این تاپیک رو هم مطالعه بفرمایین بد نیست:31:

تاپیک نحوه ی انتشار درست فرمولهای ریاضی

برای مشاهده محتوا ، لطفا وارد شوید یا ثبت نام کنید



موفق باشین.
89/8/5

Life24
27-10-2010, 19:23
سلام.
منظورتون اینه؟

[ برای مشاهده لینک ، لطفا با نام کاربری خود وارد شوید یا ثبت نام کنید ] %7D%7Bx-1%7D-%5Cfrac%7B1%7D%7Ba-1%7D%7D%7Bx-a%7D



ضمنا این تاپیک رو هم مطالعه بفرمایین بد نیست:31:

تاپیک نحوه ی انتشار درست فرمولهای ریاضی

برای مشاهده محتوا ، لطفا وارد شوید یا ثبت نام کنید



موفق باشین.
89/8/5



بله حتما
بله خودشه
اگر بشه صورت رو این طور نوشت خوب میشد x-a
که خط بخوره :31:

davy jones
27-10-2010, 19:25
بله حتما
بله خودشه
اگر بشه صورت رو این طور نوشت خوب میشد x-a
که خط بخوره :31:

جوابتون رو در همون پست قبلیم ویرایش کردم.

موفق باشین.
89/8/5

Life24
27-10-2010, 19:42
جوابتون رو در همون پست قبلیم ویرایش کردم.

موفق باشین.
89/8/5

[ برای مشاهده لینک ، لطفا با نام کاربری خود وارد شوید یا ثبت نام کنید ]

از راه فرمول مشتق مشتق این باید حساب بشه.
همونی که x به سمت a میره (حد)
خوب بعد از این که جایگزاری میکنی
در مزدوجش ضرب میکنی
بعد اخر سر ساده میکنی جواب این میشه
فکر کنم اشتباه هست :41:
[ برای مشاهده لینک ، لطفا با نام کاربری خود وارد شوید یا ثبت نام کنید ]

davy jones
28-10-2010, 00:20
[ برای مشاهده لینک ، لطفا با نام کاربری خود وارد شوید یا ثبت نام کنید ]

از راه فرمول مشتق مشتق این باید حساب بشه.
همونی که x به سمت a میره (حد)
خوب بعد از این که جایگزاری میکنی
در مزدوجش ضرب میکنی
بعد اخر سر ساده میکنی جواب این میشه
فکر کنم اشتباه هست :41:
[ برای مشاهده لینک ، لطفا با نام کاربری خود وارد شوید یا ثبت نام کنید ]

سلام. در حالت کلی طبق فرمول تعریف مشتق داریم:


[ برای مشاهده لینک ، لطفا با نام کاربری خود وارد شوید یا ثبت نام کنید ] lim_%7B%5Cdelta&space;%5Crightarrow&space;0%7D%5Cfrac%7Bf%28x&plus; %5Cdelta&space;%29-f%28x%29%7D%7B%5Cdelta&space;%7D


که در این جا داریم:

[ برای مشاهده لینک ، لطفا با نام کاربری خود وارد شوید یا ثبت نام کنید ] qrt%7Bx-1%7D%7D


بنابراین داریم:


[ برای مشاهده لینک ، لطفا با نام کاربری خود وارد شوید یا ثبت نام کنید ] Cdelta&space;%5Crightarrow&space;0%7D%5Cfrac%7Bf%28x&plus;%5Cdelta&space; %29-f%28x%29%7D%7B%5Cdelta&space;%7D=%5Clim_%7B%5Cdelta&space;%5Cr ightarrow&space;0%7D%5Cfrac%7B%5Cfrac%7B1%7D%7B%5Csqrt%7 Bx&plus;%5Cdelta&space;-1%7D%7D-%5Cfrac%7B1%7D%7B%5Csqrt%7Bx-1%7D%7D%7D%7B%5Cdelta&space;%7D=%5Clim_%7B%5Cdelta&space;%5Cri ghtarrow&space;0%7D%5Cfrac%7B%5Cfrac%7B%5Csqrt%7Bx-1%7D-%5Csqrt%7Bx&plus;%5Cdelta&space;-1%7D%7D%7B%5Csqrt%7B%28x-1%29%5E%7B2%7D&plus;%5Cdelta&space;%28x-1%29%7D%7D%7D%7B%5Cdelta&space;%7D=%5Clim_%7B%5Cdelta&space;%5 Crightarrow&space;0%7D%5Cfrac%7B%28x-1%29-%28x&plus;%5Cdelta&space;-1%29%7D%7B%5Cdelta&space;%5Ctimes&space;%28%5Csqrt%7B%28x-1%29%5E%7B2%7D&plus;%5Cdelta&space;%28x-1%29%7D%29%28%5Csqrt%7Bx-1%7D&plus;%5Csqrt%7Bx&plus;%5Cdelta&space;-1%7D%29%7D=%5Clim_%7B%5Cdelta&space;%5Crightarrow&space;0%7D%5 Cfrac%7B-%5Cdelta&space;%7D%7B%5Cdelta&space;%5Ctimes&space;%28%5Csqrt%7B%28x-1%29%5E%7B2%7D&plus;%5Cdelta&space;%28x-1%29%7D%29%28%5Csqrt%7Bx-1%7D&plus;%5Csqrt%7Bx&plus;%5Cdelta&space;-1%7D%29%7D=%5Clim_%7B%5Cdelta&space;%5Crightarrow&space;0%7D%5 Cfrac%7B-1%7D%7B%28%5Csqrt%7B%28x-1%29%5E%7B2%7D&plus;%5Cdelta&space;%28x-1%29%7D%29%28%5Csqrt%7Bx-1%7D&plus;%5Csqrt%7Bx&plus;%5Cdelta&space;-1%7D%29%7D=%5Cfrac%7B-1%7D%7B2%28x-1%29%5Csqrt%7Bx-1%7D%7D=%7B%5Ccolor%7Bred%7D&space;%5Cfrac%7B-1%7D%7B2%5Csqrt%7B%28x-1%29%5E%7B3%7D%7D%7D%7D



موفق باشین.
89/8/6

moh72
28-10-2010, 10:24
این چجوری اثبات میشه؟

[ برای مشاهده لینک ، لطفا با نام کاربری خود وارد شوید یا ثبت نام کنید ]

arsalne
28-10-2010, 11:41
سلام ،در زمينه ي كنكور كارداني به كارشناسي و همچنين ارشد كدام موسسه كنكوري يا آموزش عالي بهتر توانسته درس رياضيات بهتر توضيح بده و بهتر كارش كرده باشه؟

---------------------------------------------------------------------
با سلام

کاربر گرامی اینجا مکانی برای بحث و بررسی مسائل علمی است نه تبلیغ برای موسسات!!

موفق باشید.

مفیدی

6 آبان 1389

davy jones
28-10-2010, 15:26
این چجوری اثبات میشه؟

[ برای مشاهده لینک ، لطفا با نام کاربری خود وارد شوید یا ثبت نام کنید ]



سلام. این تساوی به ازای همه ی مقادیر تتا برقرار نیست. این یک معادله است که باید شما مقدار تتا رو از توش پیدا کنین.


موفق باشین.
89/8/6

Life24
28-10-2010, 16:46
سلام. در حالت کلی طبق فرمول تعریف مشتق داریم:


[ برای مشاهده لینک ، لطفا با نام کاربری خود وارد شوید یا ثبت نام کنید ] lim_%7B%5Cdelta&space;%5Crightarrow&space;0%7D%5Cfrac%7Bf%28x&plus; %5Cdelta&space;%29-f%28x%29%7D%7B%5Cdelta&space;%7D


که در این جا داریم:

[ برای مشاهده لینک ، لطفا با نام کاربری خود وارد شوید یا ثبت نام کنید ] qrt%7Bx-1%7D%7D


بنابراین داریم:


[ برای مشاهده لینک ، لطفا با نام کاربری خود وارد شوید یا ثبت نام کنید ] Cdelta&space;%5Crightarrow&space;0%7D%5Cfrac%7Bf%28x&plus;%5Cdelta&space; %29-f%28x%29%7D%7B%5Cdelta&space;%7D=%5Clim_%7B%5Cdelta&space;%5Cr ightarrow&space;0%7D%5Cfrac%7B%5Cfrac%7B1%7D%7B%5Csqrt%7 Bx&plus;%5Cdelta&space;-1%7D%7D-%5Cfrac%7B1%7D%7B%5Csqrt%7Bx-1%7D%7D%7D%7B%5Cdelta&space;%7D=%5Clim_%7B%5Cdelta&space;%5Cri ghtarrow&space;0%7D%5Cfrac%7B%5Cfrac%7B%5Csqrt%7Bx-1%7D-%5Csqrt%7Bx&plus;%5Cdelta&space;-1%7D%7D%7B%5Csqrt%7B%28x-1%29%5E%7B2%7D&plus;%5Cdelta&space;%28x-1%29%7D%7D%7D%7B%5Cdelta&space;%7D=%5Clim_%7B%5Cdelta&space;%5 Crightarrow&space;0%7D%5Cfrac%7B%28x-1%29-%28x&plus;%5Cdelta&space;-1%29%7D%7B%5Cdelta&space;%5Ctimes&space;%28%5Csqrt%7B%28x-1%29%5E%7B2%7D&plus;%5Cdelta&space;%28x-1%29%7D%29%28%5Csqrt%7Bx-1%7D&plus;%5Csqrt%7Bx&plus;%5Cdelta&space;-1%7D%29%7D=%5Clim_%7B%5Cdelta&space;%5Crightarrow&space;0%7D%5 Cfrac%7B-%5Cdelta&space;%7D%7B%5Cdelta&space;%5Ctimes&space;%28%5Csqrt%7B%28x-1%29%5E%7B2%7D&plus;%5Cdelta&space;%28x-1%29%7D%29%28%5Csqrt%7Bx-1%7D&plus;%5Csqrt%7Bx&plus;%5Cdelta&space;-1%7D%29%7D=%5Clim_%7B%5Cdelta&space;%5Crightarrow&space;0%7D%5 Cfrac%7B-1%7D%7B%28%5Csqrt%7B%28x-1%29%5E%7B2%7D&plus;%5Cdelta&space;%28x-1%29%7D%29%28%5Csqrt%7Bx-1%7D&plus;%5Csqrt%7Bx&plus;%5Cdelta&space;-1%7D%29%7D=%5Cfrac%7B-1%7D%7B2%28x-1%29%5Csqrt%7Bx-1%7D%7D=%7B%5Ccolor%7Bred%7D&space;%5Cfrac%7B-1%7D%7B2%5Csqrt%7B%28x-1%29%5E%7B3%7D%7D%7D%7D



موفق باشین.
89/8/6







داداش قسمت آخر توانش سه نیست توانش دو هست
اگر از رمول مشتق هم بریم همون میشه :31:

davy jones
28-10-2010, 17:11
داداش قسمت آخر توانش سه نیست توانش دو هست
اگر از رمول مشتق هم بریم همون میشه :31:
توانش زیر رادیکال 3 هست. یعنی توان کل میشه 1.5

قله بلند
28-10-2010, 23:17
با سلام خدمت دوستان ریاضی دوست
فرض کنید می خواهیم عددی صحیح و نامنفی را به روش دو دویی کد کنیم. مثلاً:
برای مشاهده محتوا ، لطفا وارد شوید یا ثبت نام کنیدکه عدد 13 در مبنای 10 و عدد 1101 در مبنای 2 است.
عدد 13، 4 بیت را اشغال کرده است. یک فرمولی هست که می گوید تعداد بیت هایی که برای هر عدد صحیح نامنفی اشغال می شود حد پایین logn در مبنای 2 بعلاوه یک است. مثلاً برای مثال ما:
برای مشاهده محتوا ، لطفا وارد شوید یا ثبت نام کنیددر اینجا، حد پایین عدد 13مساوی با 3 و حد بالای عدد 13 مساوی با 4 است. اگر حد پایین عدد 13 را با عدد 1 جمع کنیم، عدد 4 به دست می آید . یعنی عدد 13 در مبنای 2، 4 بیت فضا لازم دارد.
سوال: چرا یک راست، حد بالای 13 را نمی گیریم که عدد 4 را به ما می دهد و می آییم حد پایین را حساب کرده و با عدد 1 جمع می کنیم؟
این مطلب، صحیح است ولی علتش را نمی دانم.

davy jones
28-10-2010, 23:47
با سلام خدمت دوستان ریاضی دوست
فرض کنید می خواهیم عددی صحیح و نامنفی را به روش دو دویی کد کنیم. مثلاً:
برای مشاهده محتوا ، لطفا وارد شوید یا ثبت نام کنیدکه عدد 13 در مبنای 10 و عدد 1101 در مبنای 2 است.
عدد 13، 4 بیت را اشغال کرده است. یک فرمولی هست که می گوید تعداد بیت هایی که برای هر عدد صحیح نامنفی اشغال می شود حد پایین logn در مبنای 2 بعلاوه یک است. مثلاً برای مثال ما:
برای مشاهده محتوا ، لطفا وارد شوید یا ثبت نام کنیددر اینجا، حد پایین عدد 13مساوی با 3 و حد بالای عدد 13 مساوی با 4 است. اگر حد پایین عدد 13 را با عدد 1 جمع کنیم، عدد 4 به دست می آید . یعنی عدد 13 در مبنای 2، 4 بیت فضا لازم دارد.
سوال: چرا یک راست، حد بالای 13 را نمی گیریم که عدد 4 را به ما می دهد و می آییم حد پایین را حساب کرده و با عدد 1 جمع می کنیم؟
این مطلب، صحیح است ولی علتش را نمی دانم.

خب برای اینکه مواقعی که مقدار لگاریتم عدد صحیح است (مثلا وقتی که n=16) حد پایین و بالا برابر هم میشه و دیگه حد بالا برابر با تعداد بیتهای مورد نیاز نمیشه. این حالت برای اعدادی از n اتفاق میفته که توان عدد 2 باشه. (یعنی ..., n=1,2,4,8,16,32,64)

موفق باشین.
89/8/7

قله بلند
29-10-2010, 12:11
سلام
بله مطلب شما کاملاً درسته ولی یه سوالی اینجا مطرحه و اون اینکه اصولاً می شه حد بالا و پایین عددی، خودش بشه؟


وقتی از نماد کوچکتر از و بزرگتر از استفاده می کنم، بقیه متن از بین می رود. برای همین مجبورم بدون استفاده از نمادهای ریاضی توضیح بدم.

برای اعدادی که توانی از دو نیستند هم می شود از کران پایین logn +1 استفاده کرد و هم می شود از کران بالا استفاده کرد.

ولی برای آنهایی که توانی از دو هستند یعنی 2 و 4 و 8 و 16 فقط باید از کران بالا استفاده کرد چون کران پایین log4 +1 می شود عدد 3 که یکی کمتر از عدد 4 است. چون عدد 8 دقیقاً 4 بیت را اشغال می کند ولی اگر کران بالای عدد 8 یعنی 16 را حساب کنیم می شود 4 و حساب و کتاب ما درست در می آید.پس ما برای اعدادی که توانی از 2 هستند دچار مشکل هستیم.

ولی اگر همیشه کران بالا را در نظر بگیریم مشکل همه اعداد حل می شود.

عدد 4، اولین عددی است که 3 بیت را اشغال می کند و عدد 8 نیز اولین عددی است که 4 بیت را اشغال می کند و عدد 16 هم اولین عددی است که 5 بیت را اشغال می کند.
ولی برای اعدادی که توانی از 2 نیستند این مطلب، بینابین است. مثلاً عدد 7 همان بیتی را اشغال می کند که عدد 4.
می شه شما هم نظر بدید؟

davy jones
29-10-2010, 14:04
سلام
بله مطلب شما کاملاً درسته ولی یه سوالی اینجا مطرحه و اون اینکه اصولاً می شه حد بالا و پایین عددی، خودش بشه؟


وقتی از نماد کوچکتر از و بزرگتر از استفاده می کنم، بقیه متن از بین می رود. برای همین مجبورم بدون استفاده از نمادهای ریاضی توضیح بدم.

برای اعدادی که توانی از دو نیستند هم می شود از کران پایین logn +1 استفاده کرد و هم می شود از کران بالا استفاده کرد.

ولی برای آنهایی که توانی از دو هستند یعنی 2 و 4 و 8 و 16 فقط باید از کران بالا استفاده کرد چون کران پایین log4 +1 می شود عدد 3 که یکی کمتر از عدد 4 است. چون عدد 8 دقیقاً 4 بیت را اشغال می کند ولی اگر کران بالای عدد 8 یعنی 16 را حساب کنیم می شود 4 و حساب و کتاب ما درست در می آید.پس ما برای اعدادی که توانی از 2 هستند دچار مشکل هستیم.

ولی اگر همیشه کران بالا را در نظر بگیریم مشکل همه اعداد حل می شود.

عدد 4، اولین عددی است که 3 بیت را اشغال می کند و عدد 8 نیز اولین عددی است که 4 بیت را اشغال می کند و عدد 16 هم اولین عددی است که 5 بیت را اشغال می کند.
ولی برای اعدادی که توانی از 2 نیستند این مطلب، بینابین است. مثلاً عدد 7 همان بیتی را اشغال می کند که عدد 4.
می شه شما هم نظر بدید؟

این که حد پایین اعداد صحیح با حد بالای اونها تفاوتی نداره رو قبول دارین؟
در ثانی:
log 4=2
که حد پایین عبارت بالا هم برابر با 2 میشه
در نتیجه:
log 4 + 1=2+1=3
از طرفی عدد 4 در مبنای دو به صورت 100 هستش که 3 بیت اشغال میکنه. خب این کجاش دچار مشکل شد؟

برای عدد 8 هم همین طور. داریم:
log 8=3
حد پایین 3 هم برابر با خود 3 میشه.
3+1 هم برابر با 4 میشه.
8 در مبنای دو هم به صورت 1000 هستش که 4 بیت اشغال میکنه و عدد 4 بدست اومده در خط بالا درسته.

اتفاقا حد بالا دچار مشکل میشه چون برای اعداد صحیح حد بالا برابر با حد پایین میشه و دیگه چون مثل قبل با یک جمع نمیشه جواب غلط به دست میآد.

(داخل پرانتز بگم که اصطلاح ریاضی حد پایین و بالا در براکت گیری تا اونجایی که من یاد گرفتم، کف و سقف هستش ولی خب به خاطر اینکه دچار مشکل و سردرگمینشین استفاده نکردم:31:)

موفق باشین.
89/8/7

قله بلند
29-10-2010, 15:58
سلام. دقیقاً من با این مورد مشکل دارم که آیا حد پایین و بالای یک عدد، خودش هست یا اینکه یه کوچولو کمتر و یه کوچولو بیشتر می شن حد پایین و بالاش.
اگر حد بالا و پایین عدد 7 خودش باشه پس چرا به دنبال سقف و کف هستیم.(من هم دقیقاً از سقف و کف استفاده می کنم)
در ضمن من با اون فرمول مشکلی ندارم ولی نمی دونم بر اساس چه منطقی درست شده؟ روز چهارشنبه سر کلاس سر این مطلب بحث راه افتاد. استادمون گفتند که یه حکمتی داره ولی یادم نمی یاد چرا از حد پایین استفاده می شه.

moh72
29-10-2010, 17:08
سلام. این تساوی به ازای همه ی مقادیر تتا برقرار نیست. این یک معادله است که باید شما مقدار تتا رو از توش پیدا کنین.


موفق باشین.
89/8/6
خیلی ممنون چرا برقرار نیست؟میشه یه خورده بیشتر توضیح بدید؟:11:

davy jones
29-10-2010, 17:45
خیلی ممنون چرا برقرار نیست؟میشه یه خورده بیشتر توضیح بدید؟:11:

اگه قرار باشه تساوی به ازای تمام مقادیر تتا برقرار باشه باید هر مقدار تتا یی رو که در دو طرف معادله قرار میدیم به یک جواب برسیم که واضحه که به ازای تمام مقادیر تتا برقرار نیست. شما تتا رو بذار صفر.

قله بلند
29-10-2010, 20:52
سلام. این مطلب رو می شه با فرمول هم ثابت کرد:
می دانیم:

برای مشاهده محتوا ، لطفا وارد شوید یا ثبت نام کنیدکه کران پایین سیگما عدد صفر و کران بالای آن کف لگاریتم عدد صحیح ماست.
اگر تمام بیت ها یک باشد یعنی bi ها همگی یک باشند، می شود مجموع یک سری ساده که جوابش می شود:
برای مشاهده محتوا ، لطفا وارد شوید یا ثبت نام کنیداگر تمام بیت ها صفر باشد یعنی bi ها همگی صفر باشند، می شود مجموع عدد صفر که آنهم می شود صفر.
حالا بقیه اعداد، بین این دو مقدار با این بیت مفرض قرار می گیرند.
حداکثر عددی که مثلاً در 4 بیت جا می گیرد عدد 15 است و

برای مشاهده محتوا ، لطفا وارد شوید یا ثبت نام کنیدهم تعداد بیت های آن است.

davy jones
30-10-2010, 04:44
سلام. دقیقاً من با این مورد مشکل دارم که آیا حد پایین و بالای یک عدد، خودش هست یا اینکه یه کوچولو کمتر و یه کوچولو بیشتر می شن حد پایین و بالاش.
اگر حد بالا و پایین عدد 7 خودش باشه پس چرا به دنبال سقف و کف هستیم.(من هم دقیقاً از سقف و کف استفاده می کنم)
در ضمن من با اون فرمول مشکلی ندارم ولی نمی دونم بر اساس چه منطقی درست شده؟ روز چهارشنبه سر کلاس سر این مطلب بحث راه افتاد. استادمون گفتند که یه حکمتی داره ولی یادم نمی یاد چرا از حد پایین استفاده می شه.

اگه عددی اعشاری باشه، کف اون عدد میشه مقدار صحیح اون عدد بدون احتساب ارقام بعد از اعشار (یعنی در حقیقت کف یک عدد اعشاری، نزدیک ترین عدد صحیح کوچکتر از آن عدد است)

و سقف اون عدد هم برابر میشه با کف عدد اعشاری به علاوه ی یک (در حقیقت سقف یک عدد اعشاری، نزدیک ترین عدد صحیح بزرگتر از آن عدد است)

ولی برای خود اعداد صحیح، نزدیک ترین عدد صحیح به اون عدد خودشه. بنابراین جزء صحیح (براکت) اعداد صحیح خود همون عدد میشه. نه یه ذره کمتر و نه یه ذره بیشتر. (حالا چه کف باشه و چه سقف)

اگر حد بالا و پایین عدد 7 خودش باشه پس چرا به دنبال سقف و کف هستیم.اصولا تابع جزء صحیح برای بدست آوردن قسمت صحیح اعداد اعشاری (با آپشن های متفاوت مد نظر ما) به کار میره و واضحه که این تابع بر روی خود اعداد صحیح تاثیری نداره و انتظار هم نداریم که داشته باشه.

استادمون گفتند که یه حکمتی داره ولی یادم نمی یاد چرا از حد پایین استفاده می شه.
سلام منو به استادتون برسون:31:

موفق باشین.
89/8/8

arsalne
30-10-2010, 19:07
سلام ،در زمينه ي كنكور كارداني به كارشناسي و همچنين ارشد كدام موسسه كنكوري يا آموزش عالي بهتر توانسته درس رياضيات بهتر توضيح بده و بهتر كارش كرده باشه؟

---------------------------------------------------------------------
با سلام

کاربر گرامی اینجا مکانی برای بحث و بررسی مسائل علمی است نه تبلیغ برای موسسات!!

موفق باشید.

مفیدی

6 آبان 1389

سلام بسيار معذرت از همه ي دوستان.

mohsen_blid
02-11-2010, 18:44
دوستان یه سوال
هشت به توان -1 و همچنین به توان -2 چند میشه
منظورم منفی هست یعنی به توان منفی یک و منفی 2
ممنون

ask_bl
02-11-2010, 19:03
دوستان یه سوال
هشت به توان -1 و همچنین به توان -2 چند میشه
منظورم منفی هست یعنی به توان منفی یک و منفی 2
ممنون
[ برای مشاهده لینک ، لطفا با نام کاربری خود وارد شوید یا ثبت نام کنید ] {\color{blue} 8^{-2}=\frac{1}{8^{2}}=0.015625}

[ برای مشاهده لینک ، لطفا با نام کاربری خود وارد شوید یا ثبت نام کنید ] {\color{blue} 8^{-1}=\frac{1}{8}=0.125}

davy jones
02-11-2010, 19:37
سلام یه سوال داشتم که گفتم اگه بشه توی بخش حل مسایل بگم شاید کسی کمک کرد.
[ برای مشاهده لینک ، لطفا با نام کاربری خود وارد شوید یا ثبت نام کنید ]
روش حل این دو تا انتگرال رو می خواست.به طور کامل با جزئیات مثلا" از چه نوه تغییر متغیری استفاده شده و ...
ام و ان می تونه هر عددی بزرگتر تر از 1 باشه.

سلام. اگه منظورتون از انتگرال دوم اینه:


[ برای مشاهده لینک ، لطفا با نام کاربری خود وارد شوید یا ثبت نام کنید ] dx

باید بگم که این انتگرال به صورت نامعین جواب نداره. ولی اگه منظورتون اینه:


[ برای مشاهده لینک ، لطفا با نام کاربری خود وارد شوید یا ثبت نام کنید ] 7Bm%7Ddx

این انتگرال رو وقتی به برنامه mathematica دادم این جواب عجیب و غریب رو بهم داد:


[ برای مشاهده لینک ، لطفا با نام کاربری خود وارد شوید یا ثبت نام کنید ] 7Bm%7Ddx=%5Cfrac%7B%5Csqrt%7B%5Csin&space;%5E%7B2%7D%28x %29%7D%5Ccsc&space;%28x%29%5Ccos&space;%5E%7B%281-m%29%7D%28x%29_%7B2%7D%5Ctextrm%7BF%7D_%7B1%7D%28% 5Cfrac%7B1%7D%7B2%7D,%5Cfrac%7B1-m%7D%7B2%7D;%5Cfrac%7B3-m%7D%7B2%7D;%5Ccos&space;%5E%7B2%7D%28x%29%29%7D%7Bm-1%7D


که در اون تابع (...)2F1 به صورت زیر تعریف میشه:


[ برای مشاهده لینک ، لطفا با نام کاربری خود وارد شوید یا ثبت نام کنید ]

که منظور از [ برای مشاهده لینک ، لطفا با نام کاربری خود وارد شوید یا ثبت نام کنید ] تابع معروف گاما هستش.


** البته یه تعریف دیگه از تابع (...)2F1 هم در یک سایت دیگه پیدا کردم که با استفاده از سری تعریف شده:


[ برای مشاهده لینک ، لطفا با نام کاربری خود وارد شوید یا ثبت نام کنید ] [ برای مشاهده لینک ، لطفا با نام کاربری خود وارد شوید یا ثبت نام کنید ] [ برای مشاهده لینک ، لطفا با نام کاربری خود وارد شوید یا ثبت نام کنید ]
[ برای مشاهده لینک ، لطفا با نام کاربری خود وارد شوید یا ثبت نام کنید ] [ برای مشاهده لینک ، لطفا با نام کاربری خود وارد شوید یا ثبت نام کنید ]


برای پیدا کردن جواب انتگرال اول هم در حال اقدام هستم:31:

موفق باشین.
89/8/11

mehrdad_ati
03-11-2010, 12:42
سلام دوستان .

یکی اومد پیش من و یه سوال جالبی ازم پرسید . ولی خوب من خودم دارم برای کنکور میخونم و خیلی وقته با این جور مسائل ابتکاری کار نکردم . اما برام جالب بود که چه جوری میشه این عبارت رو ساده کرد و به جواب رسید . لطفا تا جایی که میشه به صورت مرحله به مرحله توضیح بدید تا کامل متوجه بشم .

این صورت سوال هست که باید به جواب برسیم تا جایی که میشه ساده کنیم ...


[ برای مشاهده لینک ، لطفا با نام کاربری خود وارد شوید یا ثبت نام کنید ]

تو صورت عبارات داخل پرانتز بینشون صرب هست و بین خود پرانتز ها ضرب هست . در مخرج دقیقا برعکس یعنی بین عددهای درون پرانتز جمع هست اما بین پرانتز ها ضرب هست .

پیشاپیش ممنون

davy jones
03-11-2010, 13:11
سلام دوستان .

یکی اومد پیش من و یه سوال جالبی ازم پرسید . ولی خوب من خودم دارم برای کنکور میخونم و خیلی وقته با این جور مسائل ابتکاری کار نکردم . اما برام جالب بود که چه جوری میشه این عبارت رو ساده کرد و به جواب رسید . لطفا تا جایی که میشه به صورت مرحله به مرحله توضیح بدید تا کامل متوجه بشم .

این صورت سوال هست که باید به جواب برسیم تا جایی که میشه ساده کنیم ...


[ برای مشاهده لینک ، لطفا با نام کاربری خود وارد شوید یا ثبت نام کنید ]

تو صورت عبارات داخل پرانتز بینشون صرب هست و بین خود پرانتز ها ضرب هست . در مخرج دقیقا برعکس یعنی بین عددهای درون پرانتز جمع هست اما بین پرانتز ها ضرب هست .

پیشاپیش ممنون

بفرما دوست عزیز. من تا اینجا تونستم ساده اش کنم:


[ برای مشاهده لینک ، لطفا با نام کاربری خود وارد شوید یا ثبت نام کنید ] 2%29&plus;%282%5Cast&space;3%29&plus;...%2899%5Cast&space;100%29%7D%7B%2 81&plus;2%29%5Cast&space;%282&plus;3%29%5Cast&space;...%2899&plus;100%29%7D=% 5Cfrac%7B%5Csum_%7Bi=1%7D%5E%7Bn%7Di%28i&plus;1%29%7D%7 B3%5Cast&space;5%5Cast&space;7%5Cast&space;...199%7D=%5Cfrac%7B%5Csu m_%7Bi=1%7D%5E%7Bn%7Di%5E%7B2%7D&plus;%5Csum_%7Bi=1%7D% 5E%7Bn%7Di%7D%7B%5Cfrac%7B200%21%7D%7B2%5Cast&space;4%5C ast&space;6%5Cast&space;...200%7D%7D=%5Cfrac%7B%5Cfrac%7Bn%28n &plus;1%29%282n&plus;1%29%7D%7B6%7D&plus;%5Cfrac%7Bn%28n&plus;1%29%7D% 7B2%7D%7D%7B%5Cfrac%7B200%21%7D%7B%282%29%5E%7B100 %7D%5Cast&space;100%21%7D%7D=%5Cfrac%7B%28%5Cfrac%7Bn%5E %7B3%7D%7D%7B3%7D&plus;n%5E%7B2%7D&plus;%5Cfrac%7B2n%7D%7B3% 7D%29%5Cast&space;2%5E%7B100%7D%5Cast&space;100%21%7D%7B200%21 %7D%5Crightarrow&space;%28n=99%29
[ برای مشاهده لینک ، لطفا با نام کاربری خود وارد شوید یا ثبت نام کنید ] %5Cast&space;2%5E%7B100%7D%5Cast&space;%5Cfrac%7B100%21%7D%7B2 00%21%7D=33%5Cast&space;101%5Cast&space;2%5E%7B101%7D%5Cast&space;%5 Cfrac%7B100%21%7D%7B200%21%7D

موفق باشین.
89/8/13

davy jones
03-11-2010, 16:09
برای پیدا کردن جواب انتگرال اول هم در حال اقدام هستم[ برای مشاهده لینک ، لطفا با نام کاربری خود وارد شوید یا ثبت نام کنید ]


[ برای مشاهده لینک ، لطفا با نام کاربری خود وارد شوید یا ثبت نام کنید ]


بفرما. اینم جواب انتگرال اول که به همون اندازه ی انتگرال دوم جوابش عجیب هستش:


[ برای مشاهده لینک ، لطفا با نام کاربری خود وارد شوید یا ثبت نام کنید ] 5E%7Bm%7D%28x%29dx=%5Cint&space;%5Cfrac%7B%5Csin&space;%5E%7Bn %7D%28x%29%7D%7B%5Ccos&space;%5E%7Bm%7D%28x%29%7Ddx=[%5Cfrac%7B1%7D%7Bm-1%7D.%5Ccos&space;%5E%7B%281-m%29%7D%28x%29.%5Csin&space;%5E%7B%28n&plus;1%29%7D%28x%29.%5 Csin&space;%5E%7B2%7D%28x%29%5E%7B-%5Cfrac%7Bn%7D%7B2%7D-%5Cfrac%7B1%7D%7B2%7D%7D]%5C;&space;_%7B2%7DF_%7B1%7D%28%5Cfrac%7B1-m%7D%7B2%7D,%5Cfrac%7B1-n%7D%7B2%7D;%5Cfrac%7B3-m%7D%7B2%7D;%5Ccos&space;%5E%7B2%7D%28x%29%29


که تابع 2F1 همونیه که در جواب قبلی معرفیش کردم.

موفق باشین.
89/8/13

Abismo Negro
04-11-2010, 17:25
ببخشید . اولین ارسالم بود و به اشتباه در این تاپیک نوشتم.

199069
06-11-2010, 11:54
سلام. اگه منظورتون از انتگرال دوم اینه:


[ برای مشاهده لینک ، لطفا با نام کاربری خود وارد شوید یا ثبت نام کنید ] dx

باید بگم که این انتگرال به صورت نامعین جواب نداره. ولی اگه منظورتون اینه:


[ برای مشاهده لینک ، لطفا با نام کاربری خود وارد شوید یا ثبت نام کنید ] 7Bm%7Ddx

این انتگرال رو وقتی به برنامه mathematica دادم این جواب عجیب و غریب رو بهم داد:


[ برای مشاهده لینک ، لطفا با نام کاربری خود وارد شوید یا ثبت نام کنید ] 7Bm%7Ddx=%5Cfrac%7B%5Csqrt%7B%5Csin&space;%5E%7B2%7D%28x %29%7D%5Ccsc&space;%28x%29%5Ccos&space;%5E%7B%281-m%29%7D%28x%29_%7B2%7D%5Ctextrm%7BF%7D_%7B1%7D%28% 5Cfrac%7B1%7D%7B2%7D,%5Cfrac%7B1-m%7D%7B2%7D;%5Cfrac%7B3-m%7D%7B2%7D;%5Ccos&space;%5E%7B2%7D%28x%29%29%7D%7Bm-1%7D


که در اون تابع (...)2F1 به صورت زیر تعریف میشه:


[ برای مشاهده لینک ، لطفا با نام کاربری خود وارد شوید یا ثبت نام کنید ]

که منظور از [ برای مشاهده لینک ، لطفا با نام کاربری خود وارد شوید یا ثبت نام کنید ] تابع معروف گاما هستش.


** البته یه تعریف دیگه از تابع (...)2F1 هم در یک سایت دیگه پیدا کردم که با استفاده از سری تعریف شده:


[ برای مشاهده لینک ، لطفا با نام کاربری خود وارد شوید یا ثبت نام کنید ] [ برای مشاهده لینک ، لطفا با نام کاربری خود وارد شوید یا ثبت نام کنید ] [ برای مشاهده لینک ، لطفا با نام کاربری خود وارد شوید یا ثبت نام کنید ]
[ برای مشاهده لینک ، لطفا با نام کاربری خود وارد شوید یا ثبت نام کنید ] [ برای مشاهده لینک ، لطفا با نام کاربری خود وارد شوید یا ثبت نام کنید ]


برای پیدا کردن جواب انتگرال اول هم در حال اقدام هستم:31:

موفق باشین.
89/8/11













ممنون از ارسال جواب ولی جای ام رو اشتباهی گذاشتم
[ برای مشاهده لینک ، لطفا با نام کاربری خود وارد شوید یا ثبت نام کنید ]
این درست تر.اگه ممکنه کامل با مثال توزیح بدین.
خیلی خیلی ممنون

saeedta
07-11-2010, 00:10
سلام نمی دونم جای این سوال اینجاست یا نه امیدوارم باشه
من می خوام یک گراف خیلی بزرگ با استفاده از ماتریس مجاورتش رسم کنم.
منظورم از خیلی بزرگ بالای چندصد هزار عضو غیر صفره
با چی میشه این کارو کرد؟
متلب می تونه
maple چی
لطفا راهنمایی کنید با تشکر

davy jones
07-11-2010, 08:06
ممنون از ارسال جواب ولی جای ام رو اشتباهی گذاشتم
[ برای مشاهده لینک ، لطفا با نام کاربری خود وارد شوید یا ثبت نام کنید ]
این درست تر.اگه ممکنه کامل با مثال توزیح بدین.
خیلی خیلی ممنون

جواب همونیه که در پستهای قبلی گفتم. همونطور که میدونین توان عبارتهای سینوس و کوسینوس رو میشه بدون استفاده از پرانتز کلی در بالای آنها هم نوشت. یعنی اینطوری:


[ برای مشاهده لینک ، لطفا با نام کاربری خود وارد شوید یا ثبت نام کنید ] E%7Bm%7D%28x%29


بنابراین منظور از جوابی که بنده نوشتم و عبارت مد نظر شما یکیه.


سلام نمی دونم جای این سوال اینجاست یا نه امیدوارم باشه
من می خوام یک گراف خیلی بزرگ با استفاده از ماتریس مجاورتش رسم کنم.
منظورم از خیلی بزرگ بالای چندصد هزار عضو غیر صفره
با چی میشه این کارو کرد؟
متلب می تونه
maple چی
لطفا راهنمایی کنید با تشکر


با متلب که مطمئنا میشه این کار رو کرد. ولی میپل رو شک دارم.


موفق باشین.
89/8/16

saeedta
07-11-2010, 19:33
دیوی جونز عزیز (تو دزدان دریایی کارائیب این شکلی نبودی!)
میشه راهنمایی کنی چطور میشه تو متلب رسمش کرد

arash7960
09-11-2010, 03:17
سلام .
مسئله به همراه پاسخ:

Martina has three weeks to prepare for a tennis tournament. She decides to play at least one set every day but not more than 36 sets in all. Show that there is a period of consecutive days during which she will play exactly 21 sets.?
Soln. Let a1 represent # sets played on day 1, a2, … a21 defined similarly. Then create a list of 21 natural numbers: { a1, a1 + a2, … , a1 + a2 + … + a21 }. Since she plays at least one set each day, each ai ≥ 1. Also a1 + a2 + … + a21 represents the total number of sets she will play, which is at most 36. Therefore 1 ≤ a1 < a1 + a2 < … < a1 + a2 + … + a21 ≤ 36. There is only one natural number between 1 and 36 that is divisible by 21, namely 21. So if a1 + a2 + … + ai = 21 for some i, then we are done. Otherwise all the sums are not divisible by 21 so they must fall into the 20 congruence classes of the integers mod 21 = 20 , , 2 , 1 K . However, we have 21 numbers and 20 congruence classes, so by the Pigeon-Hole Principle, at least two terms,
bs = a1 + a2 + … + as and bt = a1 + a2 + … + at, belong to the same congruence class, and so are divisible by 21. Therefore bs − bt = at+1 + at+2 + … + as and she plays 21 games on the days t+1 through s
.

پس اگر بازیکن در روز 21 1م 16 بازی انجام بده و 20 روز قبلی را روزی فقط 1 بازی بکنه چگونه بر خلاف خواسته مسئله ، دنباله‌ای از روزهای متوالی که بازیکن طی آنها دقیقاً 21 بار بازی کرده ، نخواهیم داشت!.
البته مطمئنم که صورت سوال و پاسخش رو خوب متوجه نشدم.
لطفآ یه پاسخ قانع کننده بدید.
ممنون.

Iron
09-11-2010, 10:51
سلام

چرا دیگه، از روز 16 ام تا بیست و یکم 21 بازی انجام داده.

میگه فرض می کنیم a1 تعداد بازی در روز اول و به همین ترتیب a2 تا a21 تعداد بازی در هر روز باشه.
بعد یه دنباله b تعریف می کنیم بصورت:

{ a1, a1 + a2, … , a1 + a2 + … + a21 }

از اونجا که کمترین مقدار ai برابر با یک هست، مسلمه که این دنباله صعودیه.
حالا یا حاصل تقسیم یکی از اعضای این دنباله بر 21 صفره که از اونجاییکه اعضای این دنباله بین 1 تا 36 هستن، پس اون عضو برابر با 21ه. پس حکم در این حالت صادقه.
یا اینطور نیست، پس اگر یک دنباله دیگه تعریف کنیم که اعضای اون باقیمانده تقسیم دنباله قبلی بر عدد بیست و یک باشه، اعضای اون اعداد طبیعی بین 1 تا بیست خواهند بود. پس بنابر اصل لانه کبوتری حداقل دوتا از این باقیمانده ها با هم برابر خواهند بود. (21 عدد که میتونن مقدار طبیعی بین 1 تا بیست داشته باشن)
اگر ایندوتا دنباله s ام و t ام باشن داریم:

bs = a1 + a2 + … + as
bt = a1 + a2 + … + at

پس باقیمانده تقسیم تفاضل ایندو بر 21 بخشپذیره و از اونجا که تفاضل بین اعضای دنباله b همیشه 1 تا 35ه میشه نتیجه گرفت که باقیمانده این تفاضل برابر با 21ه. یعنی

bs-bt=a(t+1)+...+as=21
منظور از a(t+1) همون a با اندیس t+1 هست.

پس در این حالت هم حکم اثبات شد.

arash7960
09-11-2010, 18:14
سلام

چرا دیگه، از روز 16 ام تا بیست و یکم 21 بازی انجام داده.

میگه فرض می کنیم a1 تعداد بازی در روز اول و به همین ترتیب a2 تا a21 تعداد بازی در هر روز باشه.
بعد یه دنباله b تعریف می کنیم بصورت:

{ a1, a1 + a2, … , a1 + a2 + … + a21 }

از اونجا که کمترین مقدار ai برابر با یک هست، مسلمه که این دنباله صعودیه.
حالا یا یکی از اعضای این دنباله بر 21 صفره که از اونجاییکه اعضای این دنباله بین 1 تا 36 هستن، پس اون عضو برابر با 21ه. پس حکم در این حالت صادقه.
یا اینطور نیست، پس اگر یک دنباله دیگه تعریف کنیم که اعضای اون باقیمانده تقسیم دنباله قبلی بر عدد بیست باشه، اعضای اون اعداد طبیعی بین 1 تا بیست خواهند بود. پس بنابر اصل لانه کبوتری حداقل دوتا از این باقیمانده ها با هم برابر خواهند بود. (21 عدد که میتونن مقدار طبیعی بین 1 تا بیست داشته باشن)
اگر ایندوتا دنباله s ام و t ام باشن داریم:

bs = a1 + a2 + … + as
bt = a1 + a2 + … + at

پس باقیمانده تقسیم تفاضل ایندو بر 21 بخشپذیره و از اونجا که تفاضل بین اعضای دنباله b همیشه 1 تا 35ه میشه نتیجه گرفت که باقیمانده این تفاضل برابر با 21ه. یعنی

bs-bt=at+...+as=21

پس در این حالت هم حکم اثبات شد.

ممنون از توضیحت.( تازه عضو شدم. دکمه تشکر داریم؟ کجاست؟)
در مورد پاسخ سوال، فکر کنم چون هنوز با بعضی مفاهیم آشنایی کامل ندارم( پایه دوم دبیرستان هستم ) یکم گیج میزنم. البته اصل لانه کبوتر رو خوندم.
اگر دنباله { 36, 20 ,..., 1,2,3} = { a1, a1 + a2, … , a1 + a2 + … + a21 } را داشته باشیم، اونوقت اگر یک دنباله دیگه تعریف کنیم که اعضای اون باقیمانده تقسیم دنباله قبلی بر عدد بیست باشه، چگونه اعضای اون اعداد طبیعی بین 1 تا بیست خواهند بود؟. منظورم اینه که باقیمانده 0 هم داریم! (باقیمانده 20 بر 20(.
اگر برای دنباله ای که گفتم یکبار دیگر توضیح دهید ممنون خواهم شد.
یک خواهش دیگه: متن سوال را ترجمه دقیقش را بنوسید.

davy jones
10-11-2010, 17:21
دیوی جونز عزیز (تو دزدان دریایی کارائیب این شکلی نبودی!)
میشه راهنمایی کنی چطور میشه تو متلب رسمش کرد



راستش من خیلی به متلب وارد نیستم.

باید ماتریسهای مجاورت رو به صورت string تعریف کنی (با استفاده از loop هر درایه ای رو سر جای خودش بنشونی)
بعد به هر روشی که بلدی (یا از خودت ابتکار میزنی) به ازای هر راس گراف، یک مختصات تعریف کنی (میتونه تو فضای سه بعدی هم باشه)
بعد با استفاده از دستور plot ، راسهایی که قرارن به هم وصل بشن رو وصل کنی.

شاید دستورهای مستقیمی برای گراف وجود داشته باشه ولی من بلد نیستم.:31:
ببخشین که دیر جواب دادم. پستتون رو دیر دیدم.

موفق باشین.
89/8/19

hamedlo
10-11-2010, 19:51
من در خصوص درونیابی چند گانه به شیوه تفاضلات متناهی سوال دارم و می خوام بدونم برای یک حالت سه متخیره فرمول عمومی به چه شکل خواهد بود. یک نفر به من گفت باید درونیابی لاگرانژ چند متغیره رو جستجو کنم ولی چیزی پیدا نکردم ممنون میشم اگه راهنمایی کنید.

با تشکر

panizir
11-11-2010, 20:30
يكي هم جواب اينو بده لطفا:
اگر f مشتق پذير باشد و



[ برای مشاهده لینک ، لطفا با نام کاربری خود وارد شوید یا ثبت نام کنید ] f(x) _{x \to- \infty}=\lim f(x) _{x \to + \infty}=0
ثابت كنيد وجود دارد c به طوري كه


[ برای مشاهده لینک ، لطفا با نام کاربری خود وارد شوید یا ثبت نام کنید ]^{\prime} (c) = 0

kian99
11-11-2010, 21:17
سلام اگر یک کاغذ را 12 بار تا کنیم ضخامت آن 41 سانتیمتر میشود برای ارتفاع اورست کاغذ را چند بار تا کنیم؟

Mohammad Hosseyn
11-11-2010, 21:18
يكي هم جواب اينو بده لطفا:
اگر f مشتق پذير باشد و



[ برای مشاهده لینک ، لطفا با نام کاربری خود وارد شوید یا ثبت نام کنید ] 0+%20%5Cinfty%7D=0
ثابت كنيد وجود دارد c به طوري كه


[ برای مشاهده لینک ، لطفا با نام کاربری خود وارد شوید یا ثبت نام کنید ] %200


سادست دوست من ...

اول گفته مشتق پذیر باشه ... شروط مشتق پذیری اینه :

در دامنه پیوشته و شکستگی نداشته باشه. شکستگی رو فرض می کنیم نداشته باشه. ولی پیوسته بودن مهمه.

مثلا این نمودار رو نگاه کن :

[ برای مشاهده لینک ، لطفا با نام کاربری خود وارد شوید یا ثبت نام کنید ]

شرط اون حدود بی نهایت رو داره. ولی پیوسته نیست.

حالا اگر پیوسته باشه ، چرا باید یه c باشه با مشخصات مسئله ؟

علت اینه که اگه پیوسته باشه و در دو بینهایت به صفر میل کنه در واقع تابع نمی تونه یکنوا باشه (اکیدا صعودی یا نزولی). به همین خاطر حتما باید یه نقطه اکسترمم درش پیدا بشه. نقطه اکسترمم هم می دونیم که اگر در اون نقطه مشتق وجود داشته باشه ، باید مشتقش صفر باشه. طبق فرض هم شکستگی نداریم (نقاطی که مشتق پذیر نیستند ولی می تونن اکسترمم باشند.).

شکل ساده و کلی این نوع توابع اینجوریه... ممکنه تابعی باشه که چندین نقطه مثل c داره. یا تابع خطی y=0 باشه که بازم در مسئله صدق میکنه و بی نهایت c در اون یافت میشه.
[ برای مشاهده لینک ، لطفا با نام کاربری خود وارد شوید یا ثبت نام کنید ]


حالا نمی دونم به زبان ریاضی راه حلی داره یا نه . ولی این توضیح مسئله هست . اگه این توضیح رو می دونستی و دنبال راه حل ریاضی بودی ، عذر خواهی میکنم. :31:

Mohammad Hosseyn
11-11-2010, 21:59
سلام اگر یک کاغذ را 12 بار تا کنیم ضخامت آن 41 سانتیمتر میشود برای ارتفاع اورست کاغذ را چند بار تا کنیم؟
برای مسئله تابع باید بسازی ... تابع ارتفاع برای کاغذ های تا شده هست :

[ برای مشاهده لینک ، لطفا با نام کاربری خود وارد شوید یا ثبت نام کنید ]

n هم تعداد دفعات تا زدن هست. L هم طول اولیه. حالا طول اولیه رو نداریم. از همون 41 سانت شدن بعد 12 بار تا زدن میشه طول اولیه رو بدست آورد.

[ برای مشاهده لینک ، لطفا با نام کاربری خود وارد شوید یا ثبت نام کنید ]

[ برای مشاهده لینک ، لطفا با نام کاربری خود وارد شوید یا ثبت نام کنید ]

حالا طول اورست (884800 سانتی متر) به عنوان h میزاریم تو معادله که n جواب مسئله بدست بیاد.

[ برای مشاهده لینک ، لطفا با نام کاربری خود وارد شوید یا ثبت نام کنید ] 41%7D%7B4096%7D.2%5E%7Bn%7D

[ برای مشاهده لینک ، لطفا با نام کاربری خود وارد شوید یا ثبت نام کنید ] 8848.10%5E%7B2%7D%7D%7B41%7D

n هم میشه لگاریتم عدد سمت راست در پایه 2 !!!

جواب نهایی هم با ماشین حساب ، حساب کردم ، شده :

26.397439855391348034443923210629

حالا شما 27 بار تا بکن درست میشه :31:

امیدوارم جایی در قرار دادن اعداد مشکل نبوده باشه. وگرنه راه حل فکر میکنم همینه .

davy jones
11-11-2010, 22:07
من در خصوص درونیابی چند گانه به شیوه تفاضلات متناهی سوال دارم و می خوام بدونم برای یک حالت سه متخیره فرمول عمومی به چه شکل خواهد بود. یک نفر به من گفت باید درونیابی لاگرانژ چند متغیره رو جستجو کنم ولی چیزی پیدا نکردم ممنون میشم اگه راهنمایی کنید.

با تشکر

سلام. بحث در این مورد خیلی کلی است. اگه میشه مساله یا مسائلی که مد نظرتونه رو اینجا قرار بدین.


يكي هم جواب اينو بده لطفا:
اگر f مشتق پذير باشد و



[ برای مشاهده لینک ، لطفا با نام کاربری خود وارد شوید یا ثبت نام کنید ] 0+%20%5Cinfty%7D=0
ثابت كنيد وجود دارد c به طوري كه


[ برای مشاهده لینک ، لطفا با نام کاربری خود وارد شوید یا ثبت نام کنید ] %200


علاوه بر جوابی که دوست خوبم Mohammad Hosseyn زحمت اون رو کشیدن، باید اضافه کنم که این شرایط برای هر بازه ی متناهی و بسته مانند [a,b] روی دامنه ی توابعی که در اون بازه مشتق پذیر باشند و حد دو طرف بازه یکسان باشد، صادق است.

-----------------------------


حالا شما 27 بار تا بکن درست میشه [ برای مشاهده لینک ، لطفا با نام کاربری خود وارد شوید یا ثبت نام کنید ]شاید خیلی به مساله ی این دوستمون ربطی نداشته باشه ولی من در جایی خوندم که به طور علمی ثابت شده هیچ کاغذی رو نمیشه از 9 بار بیشتر تا زد:31:


موفق باشین.
89/8/20

panizir
12-11-2010, 00:24
حالا نمی دونم به زبان ریاضی راه حلی داره یا نه . ولی این توضیح مسئله هست . اگه این توضیح رو می دونستی و دنبال راه حل ریاضی بودی ، عذر خواهی میکنم. :31:

راستشو بخواین فک کنم اگه تو امتحان اینجوری بنویسیم بهمون نمره ندن! :31:
بیشتر دنبال راه حل ریاضیش بودم اما بابت توضیحاتتون هم ممنون.

davy jones
12-11-2010, 00:32
راستشو بخواین فک کنم اگه تو امتحان اینجوری بنویسیم بهمون نمره ندن! :31:
بیشتر دنبال راه حل ریاضیش بودم اما بابت توضیحاتتون هم ممنون.

برای اینکه توضیحاتتون وجهه ی علمی تری داشته باشه از برهان خلف استفاده کنین و نتیجه بگیرین که حتما باید تابع مد نظر ما یکنوا (اکیدا صعودی یا اکیدا نزولی) باشه تا نقطه ی اکسترمم نداشته باشه که در این صورت با فرض کردن اینکه حد تابع در منفی بینهایت صفره، صفر بودن حد تابع در مثبت بینهایت به تناقض میرسه.

موفق باشین.
89/8/21

Mohammad Hosseyn
12-11-2010, 00:46
شاید خیلی به مساله ی این دوستمون ربطی نداشته باشه ولی من در جایی خوندم که به طور علمی ثابت شده هیچ کاغذی رو نمیشه از 9 بار بیشتر تا زد:31:


موفق باشین.
89/8/20

ها ؟!!!! :31: ... منم انگار یه چیزایی در این مورد شنیدم . ولی همینجوری که فک میکنم ، می بینم درست به نظر نمیرسه ... من الان یه کاغذ A3 دستم هست ، تا 6 بار تونستم تا بزنم :31::5:. حتما اگه کاغذ بزرگتر باشه میشه . نشد میندازیمش لای دستگاه پرس ، کاغذو علم و ... همه رو پرس کنه بره :27:.
در ضمن کاغذ هرچی نازکتر باشه راحتتر تا میخوره.
کلا فک کنم این شرایط رو برای a4 گفته بودن :46:.



راستشو بخواین فک کنم اگه تو امتحان اینجوری بنویسیم بهمون نمره ندن! :31:
بیشتر دنبال راه حل ریاضیش بودم اما بابت توضیحاتتون هم ممنون.


برای اینکه توضیحاتتون وجهه ی علمی تری داشته باشه از برهان خلف استفاده کنین و نتیجه بگیرین که حتما باید تابع مد نظر ما یکنوا (اکیدا صعودی یا اکیدا نزولی) باشه تا نقطه ی اکسترمم نداشته باشه که در این صورت با فرض کردن اینکه حد تابع در منفی بینهایت صفره، صفر بودن حد تابع در مثبت بینهایت به تناقض میرسه.

موفق باشین.
89/8/21
آره ... همینجوری بنویس... اینکه تابع مجبوره که در بازه ای نزولی و در بازه ای دیگر صعودی باشه . و چون مشتق پذیره حتما باید در این بین اکسترمم داشته باشه .

موفق باشید .

berhooznajafi
12-11-2010, 13:35
[ برای مشاهده لینک ، لطفا با نام کاربری خود وارد شوید یا ثبت نام کنید ]

davy jones
13-11-2010, 07:33
[ برای مشاهده لینک ، لطفا با نام کاربری خود وارد شوید یا ثبت نام کنید ]

سلام.

راه بدست آوردن تابع وارون به زبون ساده اینه که سعی کنیم ابتدا x رو بر حسب y بدست بیاریم و سپس جای x و y را عوض کنیم. به همین راحتی.

جواب سوال اول:


[ برای مشاهده لینک ، لطفا با نام کاربری خود وارد شوید یا ثبت نام کنید ] E%7By%7D-1%5CRightarrow&space;x=%5Cfrac%7Be%5E%7By%7D-1%7D%7B1&plus;e%5E%7By%7D%7D%5CRightarrow&space;%7B%5Ccolor%7 Bred%7D&space;f%5E%7B-1%7D%28x%29=%5Cfrac%7Be%5E%7Bx%7D-1%7D%7Be%5E%7Bx%7D&plus;1%7D%7D


جواب سوال دوم:


[ برای مشاهده لینک ، لطفا با نام کاربری خود وارد شوید یا ثبت نام کنید ] ow&space;y-1=%5Csqrt%7B1&plus;x%7D%5CRightarrow&space;x&plus;1=%28y-1%29%5E%7B2%7D%5CRightarrow&space;x=%28y-1%29%5E%7B2%7D-1%5CRightarrow&space;%7B%5Ccolor%7Bred%7D&space;f%5E%7B-1%7D%28x%29=%28x-1%29%5E%7B2%7D-1%7D


جواب سوال سوم:


[ برای مشاهده لینک ، لطفا با نام کاربری خود وارد شوید یا ثبت نام کنید ] 29%5CRightarrow&space;Arc%5C,&space;cos%28y%29=Arc%5C,&space;tan%28x %29%5CRightarrow&space;x=%5Ctan&space;%28&space;Arc%5C,&space;cos%28y%29%2 9%5CRightarrow&space;%7B%5Ccolor%7Bred%7D&space;f%5E%7B-1%7D%28x%29=%5Ctan&space;%28&space;Arc%5C,&space;cos%28x%29%29%7D


جواب سوال چهارم:



[ برای مشاهده لینک ، لطفا با نام کاربری خود وارد شوید یا ثبت نام کنید ] ghtarrow&space;ln%28y%29=%5Csqrt%7Bx%7D%5CRightarrow&space;x=% 28ln%28y%29%29%5E%7B2%7D%5CRightarrow&space;%7B%5Ccolor% 7Bred%7D&space;f%5E%7B-1%7D%28x%29=%28ln%28x%29%29%5E%7B2%7D%7D




موفق باشین.
89/8/22

Iron
13-11-2010, 13:52
سلام

چرا دیگه، از روز 16 ام تا بیست و یکم 21 بازی انجام داده.

میگه فرض می کنیم a1 تعداد بازی در روز اول و به همین ترتیب a2 تا a21 تعداد بازی در هر روز باشه.
بعد یه دنباله b تعریف می کنیم بصورت:

{ a1, a1 + a2, … , a1 + a2 + … + a21 }

از اونجا که کمترین مقدار ai برابر با یک هست، مسلمه که این دنباله صعودیه.
حالا یا حاصل تقسیم یکی از اعضای این دنباله بر 21 صفره که از اونجاییکه اعضای این دنباله بین 1 تا 36 هستن، پس اون عضو برابر با 21ه. پس حکم در این حالت صادقه.
یا اینطور نیست، پس اگر یک دنباله دیگه تعریف کنیم که اعضای اون باقیمانده تقسیم دنباله قبلی بر عدد بیست و یک باشه، اعضای اون اعداد طبیعی بین 1 تا بیست خواهند بود. پس بنابر اصل لانه کبوتری حداقل دوتا از این باقیمانده ها با هم برابر خواهند بود. (21 عدد که میتونن مقدار طبیعی بین 1 تا بیست داشته باشن)
اگر ایندوتا دنباله s ام و t ام باشن داریم:

bs = a1 + a2 + … + as
bt = a1 + a2 + … + at

پس باقیمانده تقسیم تفاضل ایندو بر 21 بخشپذیره و از اونجا که تفاضل بین اعضای دنباله b همیشه 1 تا 35ه میشه نتیجه گرفت که باقیمانده این تفاضل برابر با 21ه. یعنی

bs-bt=a(t+1)+...+as=21
منظور از a(t+1) همون a با اندیس t+1 هست.

پس در این حالت هم حکم اثبات شد.

سلام مجدد

تو دو قسمتی که قرمز کردم گفتم یا یکی از اعضا بر 21 بخشپذیره که مساله حله یا نیست که ...

مورد دیگه اینکه اعضای اون دنباله باقیمانده تقسیم بر 21 هستند نه بیست. با توجه به اینکه هیچکدام بر 21 بخشپذیر نبودند، پس اعضا بین 1 تا 20 خواهند بود.

مارتینا سه هفته برای آمادگی برای تورنومنت تنیس وقت دارد. او تصمیم می گیرد هر روز حداقل یک بازی و در کل حداکثر 36 بازی انجام دهد. نشان دهید بازه پیوسته ای از روزها وجود دارد که دقیقا 21 بازی در آن انجام شده است.

Iron
13-11-2010, 14:06
يكي هم جواب اينو بده لطفا:
اگر f مشتق پذير باشد و



[ برای مشاهده لینک ، لطفا با نام کاربری خود وارد شوید یا ثبت نام کنید ] f(x) _{x \to- \infty}=\lim f(x) _{x \to + \infty}=0
ثابت كنيد وجود دارد c به طوري كه


[ برای مشاهده لینک ، لطفا با نام کاربری خود وارد شوید یا ثبت نام کنید ]^{\prime} (c) = 0

سلام

البته توضیحاتو دوستان دادن. ولی برای نوشتن تو امتحان یه قضیه وجود داره که میگه اگر f(x) در بازه [a,b] مشتقپذیر باشه وجود دارد یک c که:

[ برای مشاهده لینک ، لطفا با نام کاربری خود وارد شوید یا ثبت نام کنید ]{f}'(c)=\frac{f(b)-f(a)}{b-a}

پس اگر a رو به منفی بینهایت و b رو به مثبت بینهایت میل بدیم بر اساس قضیه بالا c وجود خواهد داشت که مشتق تابع توش صفر بشه.

panizir
13-11-2010, 14:50
سلام

البته توضیحاتو دوستان دادن. ولی برای نوشتن تو امتحان یه قضیه وجود داره که میگه اگر f(x) در بازه [a,b] مشتقپذیر باشه وجود دارد یک c که:

[ برای مشاهده لینک ، لطفا با نام کاربری خود وارد شوید یا ثبت نام کنید ]{f}'(c)=\frac{f(b)-f(a)}{b-a}

پس اگر a رو به منفی بینهایت و b رو به مثبت بینهایت میل بدیم بر اساس قضیه بالا c وجود خواهد داشت که مشتق تابع توش صفر بشه.

تو قضیه رول a و b باید عدد باشن. یعنی باید یه عدد وجود داشته باشه. فک کنم به صورت حدی نمیشه گفت.

lebesgue
13-11-2010, 15:17
يكي هم جواب اينو بده لطفا:
اگر f مشتق پذير باشد و



[ برای مشاهده لینک ، لطفا با نام کاربری خود وارد شوید یا ثبت نام کنید ] f(x) _{x \to- \infty}=\lim f(x) _{x \to + \infty}=0
ثابت كنيد وجود دارد c به طوري كه


[ برای مشاهده لینک ، لطفا با نام کاربری خود وارد شوید یا ثبت نام کنید ]^{\prime} (c) = 0

ایده های دوستان به نظر من درست هستند، اما اثبات دقیق آنها مقداری دشوار به نظر می رسد.

می توان با استفاده از قضیه رُل و قضیه مقدار میانی، به این صورت مسئله را پاسخ داد:

اگر f تابع ثابت صفر باشد، که حکم ثابت شده است، اگر نباشد، پس a ای وجود دارد که داشته باشیم f(a) ≠ 0.
از تعریف حدهای ارائه شده در صورت سوال، نتیجه می شود که M > 0 ای وجود دارد که به ازای x > M و x < - M داشته باشیم: |f(x)|< |f(a)/3| . (کافی است اپسیلون را برابر |f(a)/3| انتخاب کنید)
پس بنا به قضیه مقدار میانی، معادله f(x) = f(a)/2 دستکم یک ریشه در بازه [a , M+1] مانند p و یک ریشه در بازه [M-1 , a-] مانند q دارد. در نتیجه (f(p) = f(q ، و بنا به قضیه رُل در این بازه c ای وجود دارد که f'(c) =0 باشد.

Iron
13-11-2010, 15:46
اینجا هم اعداد رو به بینهایت میل می دیم.
البته این ایراد وارده که با استفاده از این قضیه میشه نتیجه گرفت که c وجود داره که حد مشتقش به صفر میل می کنه. اما چه بسا برابر با صفر مطلق نشه.

ولی چون من گیر دادم به این قضیه یه مورد دیگه بنظرم رسید. اگر تابع در تمام نقاط برابر با صفر باشه که مشتقشم برابر صفر خواهد بود و قضیه ثابته. درغیر اینصورت فرض می کنیم تابع در نقطه c برابر با مقدار d باشه. با توجه به پیوستگی تابع و تعریف حد، حتما نقطه b در سمت راست و و نقطه a در سمت چپ c وجود خواهد داشت که مقدار تابع بازای اونها برابر d/2 باشه. پس با توجه به قضیه رول یک نقطه بین a و b وجود خواهد داشت که مشتقش برابر با صفر بشه.

---------------------------------------------------------

ادیت: پست جناب 1233445566 رو الان دیدم :31:

Paradise_human
13-11-2010, 20:52
ُلام دوستان کسی میتونه این انتگرال رو برام حل کنه ؟
میدونم که باید به سه قسمت تبدیل بشه و بعد محاسبه بشه ولی توی بدست آوردن Aو B و C مشکل دارم .

[ برای مشاهده لینک ، لطفا با نام کاربری خود وارد شوید یا ثبت نام کنید ]

Dokhtar_Bache
14-11-2010, 00:22
این پست ویرایش شد

Mohammad Hosseyn
14-11-2010, 08:20
راه حلشو بلدم
اگه خواستی بگو بهت میگم
ایشون پرسیدن که بگید ... :31:
تعارف می کنید :13:

شوخی میکنم ... :10:

این انتگرال یک کران داره ... جواب نهاییش چجوریه ؟! بعد z هم ثابته فکر کنم !

meral22
14-11-2010, 11:05
میدونیم که واسه توابع لگاریتمی همیشه یه سری شروط برقراره...
میخوام بدونم چرا مبنای لگاریتم نمیتونه منفی باشه؟؟؟

davy jones
14-11-2010, 11:34
ُلام دوستان کسی میتونه این انتگرال رو برام حل کنه ؟
میدونم که باید به سه قسمت تبدیل بشه و بعد محاسبه بشه ولی توی بدست آوردن Aو B و C مشکل دارم .

[ برای مشاهده لینک ، لطفا با نام کاربری خود وارد شوید یا ثبت نام کنید ]

سلام.

ابتدا شما به این سوالات من پاسخ بده:

1- Z تابعی از x و y هستش یا عدد ثابته؟
2- انتگرال قراره در فضای چند بعدی گرفته بشه؟ 3 بعدی یا یک بعدی؟
3- متغیری که داریم نسبت به اون انتگرال میگیریم چیه؟ dz ؟؟ یا چیز دیگه ای تو صورت سوال بوده؟


-------------


راه حلشو بلدم
اگه خواستی بگو بهت میگم

ایشون پرسیدن که بگید ... [ برای مشاهده لینک ، لطفا با نام کاربری خود وارد شوید یا ثبت نام کنید ]
تعارف می کنید [ برای مشاهده لینک ، لطفا با نام کاربری خود وارد شوید یا ثبت نام کنید ]
شوخی میکنم ... [ برای مشاهده لینک ، لطفا با نام کاربری خود وارد شوید یا ثبت نام کنید ]



!! Here is not good place for spamming


-------------------------------


میدونیم که واسه توابع لگاریتمی همیشه یه سری شروط برقراره...
میخوام بدونم چرا مبنای لگاریتم نمیتونه منفی باشه؟؟؟

مبنای لگاریتم در حقیقت پایه ی توان هستش و حاصل لگاریتم، توان روی پایه:


[ برای مشاهده لینک ، لطفا با نام کاربری خود وارد شوید یا ثبت نام کنید ] &space;%5CLeftrightarrow&space;%5C;&space;a%5E%7Bb%7D=X


بنابراین اگه پایه منفی باشه، مثلا هنگامی که توان منفی یک دوم باشه به تناقض میرسیم.


موفق باشین.
89/8/23

Iron
14-11-2010, 11:49
سلام

ببخشید من صرفا برای سرعت بخشیدن، بجای ایشون جواب میدم.
بنظر میرسه این انتگرال مختلطه. z=x+yi
این انتگرال روی یه دایره به مرکز مبدا مختصات و شعاع یک سوم گرفته میشه و البته مشخص نکردن مسیر ساعتگرده یا پادساعتگرد.
انتگرال هم نسبت به dz گرفته میشه که جا افتاده.

iranch
14-11-2010, 12:10
سلام
این چطور حل میشه
من گیج شدم

[ برای مشاهده لینک ، لطفا با نام کاربری خود وارد شوید یا ثبت نام کنید ](x)=(2-3x)^{-1/2

رو داده و
f(0)
و مشتق اول و دوم از ما خواسته
اگر جای ایکس صفر بزاریم که میشه عدد عبارت ما
و مشتق اون میشه صفر-ولی جواب شده-چندین بار حل کردم جواب نداد.
[ برای مشاهده لینک ، لطفا با نام کاربری خود وارد شوید یا ثبت نام کنید ]{1}{\sqrt{2}}

iranch
14-11-2010, 12:15
[ برای مشاهده لینک ، لطفا با نام کاربری خود وارد شوید یا ثبت نام کنید ]{x2}{a2}-\frac{y2}{b2}=1

مشتق دوم این رو میخواهیم(ضمنی) دقیقا 2 تا صفحه حل انجام دادم توش دریغ از جواب گرفتن !! البته دو تا مدل ساده ترشو حل کردم این گیج کننده میشه اگر امکان داره حلش رو همن بزارید وقتی حل کردید که بفهمم چی شده تشکر

davy jones
14-11-2010, 12:54
سلام
این چطور حل میشه
من گیج شدم

[ برای مشاهده لینک ، لطفا با نام کاربری خود وارد شوید یا ثبت نام کنید ]

رو داده و
f(0)
و مشتق اول و دوم از ما خواسته
اگر جای ایکس صفر بزاریم که میشه عدد عبارت ما
و مشتق اون میشه صفر-ولی جواب شده-چندین بار حل کردم جواب نداد.
[ برای مشاهده لینک ، لطفا با نام کاربری خود وارد شوید یا ثبت نام کنید ]

[ برای مشاهده لینک ، لطفا با نام کاربری خود وارد شوید یا ثبت نام کنید ] که واضحه. شما هم درست محاسبه کردین.
برای مشتق گرفتن ابتدا باید از تابع مورد نظرمون مشتق بگیریم و سپس در اون عدد دهی کنیم:


[ برای مشاهده لینک ، لطفا با نام کاربری خود وارد شوید یا ثبت نام کنید ] 8x%29=-3%282-3x%29%5E%7B-%5Cfrac%7B1%7D%7B2%7D-1%7D=-3%282-3x%29%5E%7B-%5Cfrac%7B3%7D%7B2%7D%7D%5CRightarrow&space;%7Bf%7D%27%2 80%29=-3%5Ctimes&space;2%5E%7B-%5Cfrac%7B3%7D%7B2%7D%7D=%5Cfrac%7B-3%5Csqrt%7B2%7D%7D%7B4%7D


برای مشتق دوم هم به همین ترتیب:


[ برای مشاهده لینک ، لطفا با نام کاربری خود وارد شوید یا ثبت نام کنید ] 7%28x%29=3%5E%7B2%7D%282-3x%29%5E%7B-%5Cfrac%7B3%7D%7B2%7D-1%7D=9%282-3x%29%5E%7B-%5Cfrac%7B5%7D%7B2%7D%7D%5CRightarrow&space;%7Bf%7D%27%2 7%280%29=9%5Ctimes&space;2%5E%7B-%5Cfrac%7B5%7D%7B2%7D%7D=%5Cfrac%7B9%5Csqrt%7B2%7D %7D%7B8%7D







[ برای مشاهده لینک ، لطفا با نام کاربری خود وارد شوید یا ثبت نام کنید ]

مشتق دوم این رو میخواهیم(ضمنی) دقیقا 2 تا صفحه حل انجام دادم توش دریغ از جواب گرفتن !! البته دو تا مدل ساده ترشو حل کردم این گیج کننده میشه اگر امکان داره حلش رو همن بزارید وقتی حل کردید که بفهمم چی شده تشکر


برای مشتق ضمنی در حالت کلی برای تابع (f(x,y داریم:


[ برای مشاهده لینک ، لطفا با نام کاربری خود وارد شوید یا ثبت نام کنید ] 28x,y%29=k&space;%5CRightarrow&space;%5Cfrac%7Bdy%7D%7Bdx%7D=% 5Cfrac%7B-%5Cfrac%7B%5Cpartial&space;f%7D%7B%5Cpartial&space;x%7D%7D%7B% 5Cfrac%7B%5Cpartial&space;f%7D%7B%5Cpartial&space;y%7D%7D

که منظور از k عدد ثابت است.

بنابراین طبق این رابطه داریم:

[ برای مشاهده لینک ، لطفا با نام کاربری خود وارد شوید یا ثبت نام کنید ] x%5E%7B2%7D%7D%7Ba%5E%7B2%7D%7D&plus;%5Cfrac%7By%5E%7B2 %7D%7D%7Bb%5E%7B2%7D%7D%5CRightarrow&space;%5Cfrac%7B%5C mathrm%7Bd%7D&space;y%7D%7B%5Cmathrm%7Bd%7D&space;x%7D=%5Cfrac %7B-%5Cfrac%7B2x%7D%7Ba%5E%7B2%7D%7D%7D%7B%5Cfrac%7B2y %7D%7Bb%5E%7B2%7D%7D%7D=%5Cfrac%7B-b%5E%7B2%7Dx%7D%7Ba%5E%7B2%7Dy%7D

حالا برای محاسبه مشتق دوم دقت میکنیم که:


[ برای مشاهده لینک ، لطفا با نام کاربری خود وارد شوید یا ثبت نام کنید ] Bd%5E%7B2%7D%7D&space;%7D%7B%5Cmathrm%7Bd%7D&space;x%5E%7B2%7D %7D%28%5Cfrac%7Bx%5E%7B2%7D%7D%7Ba%5E%7B2%7D%7D&plus;%5 Cfrac%7By%5E%7B2%7D%7D%7Bb%5E%7B2%7D%7D%29=%5Cfrac %7B2%7D%7Ba%5E%7B2%7D%7D&plus;%5Cfrac%7B2%28%28%5Cfrac% 7B%5Cmathrm%7Bd%7D&space;y%7D%7B%5Cmathrm%7Bd%7D&space;x%7D%29 %5E%7B2%7D&plus;y%5Ctimes&space;%5Cfrac%7B%5Cmathrm%7Bd%5E%7B 2%7D%7D&space;y%7D%7B%5Cmathrm%7Bd%7D&space;x%5E%7B2%7D%7D%29% 7D%7Bb%5E%7B2%7D%7D=%5Cfrac%7B%5Cmathrm%7Bd%5E%7B2 %7D%7D&space;%281%29%7D%7B%5Cmathrm%7Bd%7D&space;x%5E%7B2%7D%7 D=0


[ برای مشاهده لینک ، لطفا با نام کاربری خود وارد شوید یا ثبت نام کنید ] %7B%5Cmathrm%7Bd%5E%7B2%7D%7D&space;y%7D%7B%5Cmathrm%7Bd %7D&space;x%5E%7B2%7D%7D=%5Cfrac%7B%5Cfrac%7B-b%5E%7B2%7D%7D%7Ba%5E%7B2%7D%7D-%28%5Cfrac%7B%5Cmathrm%7Bd%7D&space;y%7D%7B%5Cmathrm%7Bd %7D&space;x%7D%29%5E%7B2%7D%7D%7By%7D=%5Cfrac%7B%5Cfrac% 7B-b%5E%7B2%7D%7D%7Ba%5E%7B2%7D%7D-%28%5Cfrac%7B-b%5E%7B2%7Dx%7D%7Ba%5E%7B2%7Dy%7D%29%5E%7B2%7D%7D% 7By%7D=%7B%5Ccolor%7Bred%7D&space;%5Cfrac%7B-b%5E%7B2%7D%7D%7Ba%5E%7B2%7D%7D%28%5Cfrac%7B1&plus;%28% 5Cfrac%7Bbx%7D%7Bay%7D%29%5E%7B2%7D%7D%7By%7D%29%7 D



موفق باشین.
89/8/23

Dokhtar_Bache
14-11-2010, 14:52
ابتدا شما به این سوالات من پاسخ بده:

1- Z تابعی از x و y هستش یا عدد ثابته؟
2- انتگرال قراره در فضای چند بعدی گرفته بشه؟ 3 بعدی یا یک بعدی؟
3- متغیری که داریم نسبت به اون انتگرال میگیریم چیه؟ dz ؟؟ یا چیز دیگه ای تو صورت سوال بوده؟


چرا اینقدر می پیچونید؟ من فکر کنم Z یه متغیر معمولی باشه مثل x
بعدشم ایشون تو تفکیک کسر مشکل داره میخواد A B C رو بدست بیاره
من میتونم بدست بیارم اما بلد نیستم چطوری بزنم تو فروم :37:

davy jones
14-11-2010, 14:56
من میتونم بدست بیارم اما بلد نیستم چطوری بزنم تو فروم

تاپیک آموزش نحوه انتشار درست فرمولهای ریاضی در فروم:

برای مشاهده محتوا ، لطفا وارد شوید یا ثبت نام کنید

موفق باشین.
89/8/23

Iron
14-11-2010, 15:17
چرا اینقدر می پیچونید؟ من فکر کنم Z یه متغیر معمولی باشه مثل x
بعدشم ایشون تو تفکیک کسر مشکل داره میخواد A B C رو بدست بیاره
من میتونم بدست بیارم اما بلد نیستم چطوری بزنم تو فروم :37:

اگر منظورتون اینه که Z حقیقیه، تو چه بازه ای میخواین انتگرال بگیرید.
البته با فرض مختلط بودن هم باید کسرها رو تفکیک کنید و بعد ضرایب کسرایی رو که ریشه مخرجشون توی دایره میافته میزارین تو یه رابطه، اونایی که رو مرز میافتن رو هم میزارین تو یه رابطه دیگه و با هم جمع می کنیم. متاسفانه روابط یادم نیست ولی که یادمه توشون عدد پی هم بود. روابط رو توی کتابای ریاضیات مهندسی میتونید پیدا کنید.

meral22
15-11-2010, 13:06
میدونیم که واسه توابع لگاریتمی همیشه یه سری شروط برقراره...
میخوام بدونم چرا مبنای لگاریتم نمیتونه منفی باشه؟؟؟


دوستان کسی نمیخواد کمک کنه؟؟؟؟؟:41:

davy jones
15-11-2010, 13:16
دوستان کسی نمیخواد کمک کنه؟؟؟؟؟:41:

جوابتون رو که در صفحه ی پیش دادم.
بفرمایین اینم آدرس پستی که توش جواب هستش:


برای مشاهده محتوا ، لطفا وارد شوید یا ثبت نام کنید

اگه بازم مشکلی بود در خدمتتون هستم.

موفق باشین.
89/8/24

Paradise_human
17-11-2010, 00:58
سلام.

ابتدا شما به این سوالات من پاسخ بده:

1- Z تابعی از x و y هستش یا عدد ثابته؟
2- انتگرال قراره در فضای چند بعدی گرفته بشه؟ 3 بعدی یا یک بعدی؟
3- متغیری که داریم نسبت به اون انتگرال میگیریم چیه؟ dz ؟؟ یا چیز دیگه ای تو صورت سوال بوده؟





بله تابع مختلط هست و بر حسب xوiy هستش.
در فضای مختلط دیگه ...
بله متغییر dz هستش که من فراموش کردم بنویسمش.

davy jones
20-11-2010, 16:00
ُلام دوستان کسی میتونه این انتگرال رو برام حل کنه ؟
میدونم که باید به سه قسمت تبدیل بشه و بعد محاسبه بشه ولی توی بدست آوردن Aو B و C مشکل دارم .

[ برای مشاهده لینک ، لطفا با نام کاربری خود وارد شوید یا ثبت نام کنید ]

سلام.
بفرما:

[ برای مشاهده لینک ، لطفا با نام کاربری خود وارد شوید یا ثبت نام کنید ] Ccos&space;z%7D%7Bz%28z%5E%7B2%7D&plus;1%29%7Ddz%5C;&space;%5C;&space;%5C ;%5C;&space;%5Crightarrow&space;%28C=%5Cbegin%7BBmatrix%7D&space;z:% 7Cz%7C=%5Cfrac%7B1%7D%7B3%7D&space;%5Cend%7BBmatrix%7D%2 9%5CRightarrow&space;z=%5Ccos&space;%5Ctheta&space;&plus;i%5Csin&space;%5Ctheta &space;=e%5E%7Bi%5Ctheta%7D%5CRightarrow&space;C=%5Cbegin%7BBm atrix%7D&space;z=%5Cfrac%7B1%7D%7B3%7De%5E%7Bi%5Ctheta%7 D%5C;&space;:0%3C%5Ctheta&space;%3C2%5Cpi&space;%5Cend%7BBmatrix%7D% 5CRightarrow&space;I=%5Cint_%7B0%7D%5E%7B2%5Cpi&space;%7D%5Cfr ac%7B%5Ccos&space;%28%5Cfrac%7B1%7D%7B3%7De%5E%7Bi%5Cthe ta%7D%29%7D%7B%5Cfrac%7B1%7D%7B3%7De%5E%7Bi%5Cthet a%7D%28%5Cfrac%7B1%7D%7B9%7De%5E%7B2i%5Ctheta%7D&plus;1 %29%7Dd%5Ctheta&space;=%5Cint_%7B0%7D%5E%7B2%5Cpi&space;%7D%5C frac%7B%5Ccos&space;%28%5Cfrac%7B1%7D%7B3%7De%5E%7Bi%5Ct heta%7D%29%7D%7B%5Cfrac%7B1%7D%7B27%7De%5E%7B3i%5C theta&space;%7D&plus;%5Cfrac%7B1%7D%7B3%7De%5E%7Bi%5Ctheta%7D %7Dd%5Ctheta

که چون محاسبه ی انتگرال آخر کار حضرت فیل بود(:31:) اون رو به نرم افزار Mathematica دادم و ایشون هم پاسخی فرمودند که تایپش به زبون LaTeX خیلی طول میکشه ولی میتونین در این آدرس حاصل آخرین انتگرالی که در بالا نوشتم رو مشاهده بفرمایین:


برای مشاهده محتوا ، لطفا وارد شوید یا ثبت نام کنیدالبته در محاسبه از توابع Si و Ei و Ci استفاده کرده که بنده هم تعریف دقیق این دو تابع رو نمیدونم. در پایین صفحه ی همین آدرس، خودش لینک مربوط به توضیحات این توابع رو گذاشته ولی با خوندن اونها هم نکته ی جدیدی عایدتون نمیشه. همونطور که برای من نشد چون انصافا بد توضیح داده. من بازم دنبالش میگردم و اگه پیدا کردم براتون میذارم.

موفق باشین.
89/8/29

ali_hp
23-11-2010, 18:27
ُلام دوستان کسی میتونه این انتگرال رو برام حل کنه ؟
میدونم که باید به سه قسمت تبدیل بشه و بعد محاسبه بشه ولی توی بدست آوردن Aو B و C مشکل دارم .

[ برای مشاهده لینک ، لطفا با نام کاربری خود وارد شوید یا ثبت نام کنید ]

سلام
نیازی به محاسبه انتگرال نیست،می تونید از فرمول انتگرال کوشی استفاده کنید.

برای مشاهده محتوا ، لطفا وارد شوید یا ثبت نام کنید

قله بلند
26-11-2010, 01:00
سلام.
فرض کنید ما 4 تا عدد داریم که دو به دو نسبت به هم اولند مثل

برای مشاهده محتوا ، لطفا وارد شوید یا ثبت نام کنید
حالا با دونستن این موضوع می خواهیم ثابت کنیم که بزرگترین مقسوم علیه مشترک

برای مشاهده محتوا ، لطفا وارد شوید یا ثبت نام کنید
است و بالعکس.

اگر فرض کنیم که ما 4 عدد اول داریم، به راحتی می توانیم به

برای مشاهده محتوا ، لطفا وارد شوید یا ثبت نام کنید
برسیم

و با داشتن gcd(2*3,5*7)=1 و gcd(2*5,3*7)=1 نیز با فرض اینکه عدد 10 و عدد 21 نسبت به هم اولند یعنی عوامل مشترک ندارند پس اجزای تشکیل دهنده آنها هم نسبت به هم اولند، می توان به اول بودن این 4 عدد رسید. یعنی:


برای مشاهده محتوا ، لطفا وارد شوید یا ثبت نام کنید
طرز تفکرم صحیحه؟ و دیگه احتیاج به استدلال دیگه ای نیست؟

ممنون می شم اگه نقصی هست تذکر بدید.

javad2015
26-11-2010, 12:16
با استفاده از اصل لانه کبوتری ثابت کنید اگر از اعداد طبیعی زیر به تعداد n+1 عدد انتخاب کنیم ثابت کنید که حداقل دو عدد وجود دارند که یکی بر دیگری بخش پذیر هست
1 2 3 4 .....2n

davy jones
26-11-2010, 12:49
به نظر من که درسته. اگه چند عدد رو به عوامل اولشون تجزیه کنیم (که حالا هر کدوم از این عوامل اول میتونه دارای توان بزرگتر از 1 هم باشه) بزرگترین مقسوم علیه مشترک بین این اعداد میشه اشتراک پایه ها (منظور از پایه همون عوامل اول هستش) که توان هر پایه برابر با کمترین توان موجودیه که در بین چند عدد اولیه وجود داره. بنابراین وقتی دو عدد نسبت به هم اولند (مثل 10 و 21) هر مقسوم علیه این دو عدد هم نسبت به هم اولند.

دوستا دیگه هم نظراتشون رو بگن و ما رو از سیاهی جهالت به نور معرفت رهنمون بشن:31:

موفق باشین.
89/9/5

قله بلند
26-11-2010, 13:16
سلام
جناب davy jones عزیز
وقتی ما دو تا عدد اول داریم(مثلاً 2 و3) می گویم:

برای مشاهده محتوا ، لطفا وارد شوید یا ثبت نام کنید
حالا وقتی 4 تا عدد اول داریم(مثلاً 2 و3 و5 و7) می تونیم بگیم:

برای مشاهده محتوا ، لطفا وارد شوید یا ثبت نام کنید
اکنون فرض کنیم 8 عدد اول داریم یعنی 2 و 3 و 5 و 7 و 11 و 13 و 17 و19
می توانیم بگوییم

برای مشاهده محتوا ، لطفا وارد شوید یا ثبت نام کنید
یعنی انگار یه جورایی لگاریتم تعداد اعداد اول را در پایه 2 به دست می آوریم که طول دسته های ما می شود.
مثلاً با وجود 2 تا عدد اول داریم:

برای مشاهده محتوا ، لطفا وارد شوید یا ثبت نام کنید
پس می نویسیم:

برای مشاهده محتوا ، لطفا وارد شوید یا ثبت نام کنید
و برای 4 تا عدد اول می نویسیم:

برای مشاهده محتوا ، لطفا وارد شوید یا ثبت نام کنید
و برای 8 تا عدد اول می نویسیم:

برای مشاهده محتوا ، لطفا وارد شوید یا ثبت نام کنید

چه طوری این گفته ثابت می شه؟

davy jones
26-11-2010, 13:26
با برهان خلف به راحتی ثابت میشه. فرض کن که مثلا (n1n2,n3n4) برابر با یک نشه. پس حتما یه عامل اول مشترک بین قسمت اول و دوم هست. پس بنابراین مثلا n1 و n4 نسبت به هم اول نیستند که این تناقض داره چون اعدادی که در ابتدا انتخاب کرده بودیم همگی جزء اعداد اول بودند

موفق باشین.
89/9/5

قله بلند
26-11-2010, 13:59
سلام. ممنونم.
ولی من در پست قبلی ام به این مساله رسیدم که مثلاً اگر 8 تا عدد اول داشته باشیم می تونیم اینجوری هم از اونها ب.م.م بگیریم:

برای مشاهده محتوا ، لطفا وارد شوید یا ثبت نام کنید
مثلاً اگر 16 تا عدد اول داشته باشیم می تونیم اینجوری هم از اونها ب.م.م بگیریم:

برای مشاهده محتوا ، لطفا وارد شوید یا ثبت نام کنید
این رو می خواستم بدونم چه طوری ثابت می شه؟ یعنی چه طوری می شه با شمارش تعداد اعداد اول به این نتیجه رسید؟

davy jones
26-11-2010, 20:57
یعنی چه طوری می شه با شمارش تعداد اعداد اول به این نتیجه رسید؟ فکر نکنم به تعدادشون و اون لگاریتم ربطی داشته باشه. اعداد اول به هر تعداد و با هر تقسیم بندی همیشه و همواره نسبت به هم اولند. مثل این میمونه که شما 16 تا بار الکتریکی همنام رو در دو دسته ی 8 تایی (یا بعضی از اونها رو که تصادفی هم انتخابشون میکنین در دو دسته 4 تایی) قرار بدین و بگین چون دسته اول داره دسته دوم رو دفع میکنه پس هر کدوم از بارهای دسته ی اول هر کدوم از بارهای دسته ی دوم رو باید دفه کنه که این نتیجه گیری درست نیست چون بارهای همنام همواره و در هر شرایط و در هر تقسیم بندی که قرار بگیرن همواره همدیگه رو دفع میکنن. اعداد اول هم تا حدودی همینطور هستن. اونها همیشه نسبت به هم اولند و به خاطر این دسته بندی نیست که این نتیجه حاصل شده. امیدوارم مثالم گویا بوده باشه.

موفق باشین.
89/9/5

قله بلند
27-11-2010, 13:25
سلام.
دقیقاً صحبت شما درسته و قاعدتاً نباید قید logn در کار باشه ولی نمی دونم منظور از این قسمت سوال چیه؟


سوال:
در حالت کلی تر نشان دهید که n1,n2,….,nk دو به دو نسبت به هم اولند اگر و فقط اگر مجموعه ای از سقف logk جفت از اعداد مشتق شده از ni، نسبت به هم اول باشند.


منظورش از logk جفت از اعداد مشتق شده از ni چیه؟

1-وقتی n1 ,n2 داریم، داریم:


gcd(n1,n2)=1
و log2=1

یعنی 1 جفت از اعداد مشتق شده از ni
یعنی فقط می توانیم یک ب.م.م بنویسیم؟


2-وقتی n1 ,n2, n3 ,n4 داریم، داریم:


gcd(n1n2,n3n4)=1
gcd(n1n3,n2n4)=1
gcd(n1n4,n2n3)=1



و log4=2
یعنی 2 جفت از اعداد مشتق شده از ni
یعنی فقط می توانیم دو تا ب.م.م بنویسیم؟ که در اینجا توانستیم 3 تا ب.م.م بنویسیم
یا اینکه به معنی حاصضلرب دو تا عدد اول هست؟

هر مدلی که فکر می کنم، منظورش رو از logn نمی فهمم چون بی معنی هست. ما می تونیم بنویسیم:

gcd(n1,n2n3n4)=1

و یا حتی بنویسیم:

gcd(n1n2n3n4)=1

به نظر شما منظور سوال از logk چیه؟

davy jones
27-11-2010, 16:54
به نظرم قبل از پرداختن به مساله ای که مطرح کردین بهتره اول این نکته رو برای خودمون روشن کنیم که آیا وقتی میگیم که n1 تا nk دو به دو نسبت به هم اولند، میشه نتیجه گرفت که n1 تا nk خودشون همگی عضو مجموعه ی اعداد اول هستند یا نه؟ که جواب منفیه. هر کدوم از ni ها میتونن خودشون جزء اعداد اول نباشن ولی با هیچیک از اعداد دیگه عامل مشترک نداشته باشند. مثل اعداد 6 ، 55 ، 91 ، 323 که اگه دقت کنیم هیچکدوم از این 4 عدد، اول نیستند ولی اینها 2 به 2 نسبت به هم اولند. توی این قضیه هم داره میگه که اگه k تا عدد داشته باشیم که دو به دو نسبت به هم اول باشند به هر روشی که به 2 دسته تقسیم بندی شون کنیم و اعداد هر دسته رو تو هم ضرب کنیم (اتفاقا مهم هم نیست که تعداد اعضای دو دسته با هم مساوی باشه) حاصلضرب اول نسبت به حاصلضرب دوم، اول هستش. یعنی ب.م.م حاصلضرب اول و دوم برابر با یک میشه. البته تو قضیه اثبات اینو از ما میخواد که به نظرم ساده ترین راه ممکن برای اثباتش استفاده از برهان خلفه. (اثبات مستقیم درسرش بیشتره) البته چون لفظ اگر و فقط اگر به کار رفته بنابراین باید یبار از فرض به حکم نتیجه بگیریم و قضیه رو ثابت کنیم و یه بار هم بالعکس. چون قضیه دوطرفه است. فکر کنم که مراحل اثبات از راه برهان خلف هم واضح باشه. دیگه من نمینویسم چون الان حال و حوصله فکر کردن ندارم :31:

موفق باشین.
89/9/6

javad2015
27-11-2010, 17:51
کسی نیست؟..

davy jones
27-11-2010, 19:42
سلام.
قرار بود تو تاپیک اتاق ریاضیات بپرسین. من الان تازه متوجه شدم. دیگه به بزرگواری خودتون ببخشین:31:
همونطور که واضحه هر عددی مانند x=2k به غیر از خودش که مقسوم علیه خودش هست، نزدیکترین مقسوم علیه بعدی اون عدد مطمئنا عدد k خواهد بود و اعداد بین k و 2k مطمئنا نمیتوانند مقسوم علیه عدد x باشند. بنابراین اعداد 2k , 2k-1 , 2k-2 , ... , k+1 مطمئنا اعدادی هستند که هیچکدام مقسوم علیه همدیگر نیستند. بدیهی است که بیشترین تعداد اعداد موجود در این رشته ی اعداد که همانطور که گفتیم هیچکدامشان مقسوم علیه همدیگه نیستند هنگامی رخ خواهد داد که ما بزرگترین عدد موجود در بین اعداد 1 , 2 , 3 , 4 , ... , 2n را که همان 2n هست را انتخاب کنیم. در این صورت مجموعه ی اعداد 2n , 2n-1 , ... n+1 هیچکدام مطمئنا مقسوم علیه یکدیگر نخواهند بود. تعداد اعداد این رشته برابر با n تاست. و اما در ادامه بدیهی است که هر کدام از اعداد بین 1 تا n را که انتخاب کنیم (مانند y) از آنجایی که 2y قطعا از 2n بزرگتر نیست، پس حتما 2y برابر با یکی از اعداد 2n , 2n-1 , ... n+1 خواهد بود و بنابراین عدد n+1 ام را نمیتوان طوری انتخاب کرد که مقسوم علیه هیچکدام از اعداد انتخاب شده ی قبلی نباشد.

موفق باشین.
89/9/6

davy jones
27-11-2010, 20:32
سلام. برای عوض شدن حال و هوای تاپیک هم که شده، این سوال آسون رو طرح کردم. دوستان هر کی مایله جواب بده.

حاصل انتگرال زیر را بدست آورید:


[ برای مشاهده لینک ، لطفا با نام کاربری خود وارد شوید یا ثبت نام کنید ] rt%7Bx%281-x%29%7D%7D


موفق باشین.
89/9/6

قله بلند
28-11-2010, 14:25
سلام. من به جای x می گذارم

برای مشاهده محتوا ، لطفا وارد شوید یا ثبت نام کنید
و

برای مشاهده محتوا ، لطفا وارد شوید یا ثبت نام کنید
اگر این مقادیر رو در انتگرال جاگذاری کنیم به انتگرال

برای مشاهده محتوا ، لطفا وارد شوید یا ثبت نام کنید
می رسیم و سپس به جواب

برای مشاهده محتوا ، لطفا وارد شوید یا ثبت نام کنید
حالا داریم:

برای مشاهده محتوا ، لطفا وارد شوید یا ثبت نام کنید
و جایگذاری می کنیم. جواب می شود:

برای مشاهده محتوا ، لطفا وارد شوید یا ثبت نام کنید
درست بود؟
ببینید من هم حل کردم. پس خواهش می کنم نگید حال و حوصله فکر کردن ندارم چون وقتی سوالی مطرح می کنم یعنی جوابش رو نتونستم پیدا کنم و به یه امیدی می یام تا پاسخی بگیرم.:46:

davy jones
28-11-2010, 14:41
درست بود؟

کاملا :46:


ببینید من هم حل کردم. پس خواهش می کنم نگید حال و حوصله فکر کردن ندارم چون وقتی سوالی مطرح می کنم یعنی جوابش رو نتونستم پیدا کنم و به یه امیدی می یام تا پاسخی بگیرم.[ برای مشاهده لینک ، لطفا با نام کاربری خود وارد شوید یا ثبت نام کنید ]
من هم سعی کردم تا امیدتون رو به یاس تبدیل نکنم و تقریبا راه حل رو کامل توضیح دادم. فقط اونو ننوشتم :31: شما به حساب این بذار که من بلد نبودم و بنابراین نمیخواستم که ضایع بشم :31:

موفق باشین.
89/9/7

قله بلند
30-11-2010, 09:57
سلام دوستان
داشتم مطلبی رو می خوندم که شکلی رو ترسیم کرده بود که خطی بود که از دو نقطه

برای مشاهده محتوا ، لطفا وارد شوید یا ثبت نام کنید

عبور می کنه. اون چیزی که همه با اون آشنا هستیم خط زیر هست:

برای مشاهده محتوا ، لطفا وارد شوید یا ثبت نام کنید

ولی این جا نوشته که شکل دیگر نمایش خط اینگونه است:

برای مشاهده محتوا ، لطفا وارد شوید یا ثبت نام کنید


می شه بگید این فرمول چه طوری به دست اومده؟

قله بلند
30-11-2010, 17:15
به نظرم قبل از پرداختن به مساله ای که مطرح کردین بهتره اول این نکته رو برای خودمون روشن کنیم که آیا وقتی میگیم که n1 تا nk دو به دو نسبت به هم اولند، میشه نتیجه گرفت که n1 تا nk خودشون همگی عضو مجموعه ی اعداد اول هستند یا نه؟ که جواب منفیه. هر کدوم از ni ها میتونن خودشون جزء اعداد اول نباشن ولی با هیچیک از اعداد دیگه عامل مشترک نداشته باشند. مثل اعداد 6 ، 55 ، 91 ، 323 که اگه دقت کنیم هیچکدوم از این 4 عدد، اول نیستند ولی اینها 2 به 2 نسبت به هم اولند. توی این قضیه هم داره میگه که اگه k تا عدد داشته باشیم که دو به دو نسبت به هم اول باشند به هر روشی که به 2 دسته تقسیم بندی شون کنیم و اعداد هر دسته رو تو هم ضرب کنیم (اتفاقا مهم هم نیست که تعداد اعضای دو دسته با هم مساوی باشه) حاصلضرب اول نسبت به حاصلضرب دوم، اول هستش. یعنی ب.م.م حاصلضرب اول و دوم برابر با یک میشه. البته تو قضیه اثبات اینو از ما میخواد که به نظرم ساده ترین راه ممکن برای اثباتش استفاده از برهان خلفه. (اثبات مستقیم درسرش بیشتره) البته چون لفظ اگر و فقط اگر به کار رفته بنابراین باید یبار از فرض به حکم نتیجه بگیریم و قضیه رو ثابت کنیم و یه بار هم بالعکس. چون قضیه دوطرفه است. فکر کنم که مراحل اثبات از راه برهان خلف هم واضح باشه. دیگه من نمینویسم چون الان حال و حوصله فکر کردن ندارم :31:

موفق باشین.
89/9/6
سلام.
من هنوز نفهمیدم این logk اینجا چه نقشی داره.
مثلاً فرض کنید عدد 17 هم به لیست آن 4 عددی که زیرشان خط کشیده شده اضافه شود. حالا فرض می کنیم که log5=2.1 که سقفش می شود عدد 3. این عدد 3 چه ارتباطی با تقسیم بندی ما داره؟ چون ما می تونیم بیشتر از 3 دسته تولید کنیم و این عددهای دو به دو اول را در آن دسته ها بریزیم و کارمان هم کاملاً بدون ایراد خواهد بود.

davy jones
30-11-2010, 20:41
سلام. شما در اینکه عبارتی که نوشتین، یعنی :

(1-λ)p1+ λ p2معادله ی یک خط راست هست که مشکلی ندارین؟ (بدیهی هستش که این در هر صورت معادله ی یک خط راسته)
حالا شما اگه به جای لاندا مقدار صفر رو قرار بدین به p1 میرسین. یعنی این معادله، با نقطه ی p1 اشتراک داره. حالا اگه اینبار لاندا رو برابر با یک قرار بدین به p2 میرسین. یعنی این خط راست، با نقطه ی p2 هم تلاقی داره. پس مطمئنا این معادله، معادله ی خطی مستقیمیه که از این دو نقطه میگذره. به ازای مقادیری از لاندا که بین صفر و یک قرار میگیرن در حقیقت شما نقاطی از خط واصل بین p1 و p2 رو طی میکنین که در بین این دو نقطه روی خط قرار دارند و به ازای مقادیری از لاندا که بیرون از بازه ی (0,1) باشند در حقیقت شما از نقاط p1 و p2 در حال دور شدن به دو سمت خط واصل هستین. این فرمول در حقیقت یک فرمول ابداعی است که از قضا شرط تلاقی با دو نقطه ی مورد نظر ما رو برآورده میکنه و از اونجایی که در فضای سه بعدی، از هر دو نقطه ی غیر منطبق در فضا تنها یک خط عبور میکنه پس این معادله میتونه جواب مورد نظر ما باشه.


سلام.
من هنوز نفهمیدم این logk اینجا چه نقشی داره.
مثلاً فرض کنید عدد 17 هم به لیست آن 4 عددی که زیرشان خط کشیده شده اضافه شود. حالا فرض می کنیم که log5=2.1 که سقفش می شود عدد 3. این عدد 3 چه ارتباطی با تقسیم بندی ما داره؟ چون ما می تونیم بیشتر از 3 دسته تولید کنیم و این عددهای دو به دو اول را در آن دسته ها بریزیم و کارمان هم کاملاً بدون ایراد خواهد بود.

من هم همین مشکل رو داشتم که ادامه ندادم دیگه ...:31: اگه فهمیدی منم روشن کن.

موفق باشین.
89/9/9

قله بلند
30-11-2010, 23:32
سلام.
جالبه. خوب چرا معادله خط رو به همون شکل استاندارش نمی نویسن؟ چرا به این شکل از معادله رسیدند؟
وقتی ما می خواهیم بگیم که مثلاً فلان نقطه در فلان خط هست یا نه، مختصات اون رو در اون خط می گذاریم و اگر صدق کرد پس اون نقطه در اون خط وجود داره ولی اینجا باید از λ استفاده کنیم.
اون وقت اگر از شرط -∞< λ<∞ استفاده بشه، همیشه به 0 می رسیم. درسته؟ اصولاً این شرط برای چیه؟



من هم همین مشکل رو داشتم که ادامه ندادم دیگه ...:دی اگه فهمیدی منم روشن کن.

حتماً اگر جوابش رو پیدا کردم، برای اطلاع شما و دوستان در همین تاپیک قرارش می دم.

mohsen_blid
02-12-2010, 09:29
سلام دوباره به دوستان
من دو تا سوال دارم در رابطه با حل کردن لگاریتم ممنون میشم راه حل و جواب این دو لگاریتم را محبت کنید بدید
ممنون میشم


[ برای مشاهده لینک ، لطفا با نام کاربری خود وارد شوید یا ثبت نام کنید ]
و


[ برای مشاهده لینک ، لطفا با نام کاربری خود وارد شوید یا ثبت نام کنید ]

davy jones
02-12-2010, 19:16
سلام.
جالبه. خوب چرا معادله خط رو به همون شکل استاندارش نمی نویسن؟ چرا به این شکل از معادله رسیدند؟
وقتی ما می خواهیم بگیم که مثلاً فلان نقطه در فلان خط هست یا نه، مختصات اون رو در اون خط می گذاریم و اگر صدق کرد پس اون نقطه در اون خط وجود داره ولی اینجا باید از λ استفاده کنیم.
اون وقت اگر از شرط -∞< λ<∞ استفاده بشه، همیشه به 0 می رسیم. درسته؟ اصولاً این شرط برای چیه؟

سلام.

خب هر دو روش به یک معادله ختم میشه. در حقیقت این دو تا یک معادله هستند ولی طریقه ی نمایششون فرق میکنه.


اون وقت اگر از شرط -∞< λ<∞ استفاده بشه، همیشه به 0 می رسیم. درسته؟تو این روش برای راحتی کار ، خودمون یه کاری کردیم که وقتی لاندا رو صفر بذاریم به مختصات یکی از نقاط برسیم و نقطه ی دیگه هم وقتی بدست بیاریم که لاندا برابر با 1 باشه.


سلام دوباره به دوستان
من دو تا سوال دارم در رابطه با حل کردن لگاریتم ممنون میشم راه حل و جواب این دو لگاریتم را محبت کنید بدید
ممنون میشم


[ برای مشاهده لینک ، لطفا با نام کاربری خود وارد شوید یا ثبت نام کنید ]
و


[ برای مشاهده لینک ، لطفا با نام کاربری خود وارد شوید یا ثبت نام کنید ]

سلام.

از اونجایی که تصمیم گرفتم دیگه برای سوالات اینچنینی (منظورم سوالاتیه که مربوط به دروس دبیرستان و دانشگاه میشه) جواب کامل رو نذارم تا خود کاربران تمرین بیشتری بکنن، بنابراین راه حل و جواب رو محبت نمیکنم :31: و فقط به راهنمایی های لازمه بسنده میکنم. بقیه اش به عنوان تمرین به عهده ی خودتون:31:

در سوال اول از این قانون استفاده کنین:

[ برای مشاهده لینک ، لطفا با نام کاربری خود وارد شوید یا ثبت نام کنید ] %28x%29%7D=x


و در سوال دوم هم از این قانون:

[ برای مشاهده لینک ، لطفا با نام کاربری خود وارد شوید یا ثبت نام کنید ] &plus;%5Clog_%7BA%7D%28Y%29=%5Clog_%7BA%7D%28XY%29


موفق باشین.
89/9/11

قله بلند
02-12-2010, 20:00
سلام


[ برای مشاهده لینک ، لطفا با نام کاربری خود وارد شوید یا ثبت نام کنید ](49)49^{-log_7^2}+5^{-log_5^4}=(49)(\frac{1}{2})^{2}+4^{-1}=(49)\frac{1}{4}+\frac{1}{4}=12.5





[ برای مشاهده لینک ، لطفا با نام کاربری خود وارد شوید یا ثبت نام کنید ]{2}^{(x+14)}+log_{2}^{(x+2)}=6&space;\R ightarrow&space;log_{2}^{(x+14)(x+2)}=6&space;\Rightarrow&space;2^{6 }=(x+14)(x+2)&space;\Rightarrow&space;x=-18&space;,&space;x=2

میگم، عجب کار سختیه این فرمول نویسی!

---------- Post added at 08:00 PM ---------- Previous post was at 07:58 PM ----------

پس چرا نمایش داده نمی شن؟


حالا نمایش داده شد:20:

davy jones
02-12-2010, 20:13
پس چرا نمایش داده نمی شن؟

اگه توی فرمولت از * استفاده کنی متاسفانه هنگ میکنه. به جای * از همین علامت در منوی بالای صفحه ی لاتکس استفاده کن.

موفق باشین.
89/9/11

قله بلند
02-12-2010, 20:17
سلام
جناب davy jones عزیز
یه مطلبی رو در مورد اعداد اول می خوندم که دو تا قضیه روثابت میکرد.



قضیه اول:
فرض کنید دو عدد صحیح n و h نسبت به هم اولند. حالا فرض کنید عدد صحیح m را در عدد n ضرب کنیم به طوریکه h، حاصل ضرب mn را عاد کند. از اینجا متوجه می شویم که h حتماً m را عاد میکند. این امر واضح است.





قضیه دوم:
اگر چند عدد صحیح، دو به دو نسبت به هم اول باشند، یعنی:
n1,n2,…nj

آنگاه m و k ای یافت می شود که به پیمانه ni ها با هم، هم نهشت هستند. و از این اینکه این ni ها دو به دو نسبت به هم اول هستند نتیجه میگیرد که، m و k با حاصل ضرب ni ها هم، هم نهشت است.



مثلاً:

برای مشاهده محتوا ، لطفا وارد شوید یا ثبت نام کنید


من که از این دو قضیه نتونستم به اون logk برسم. شما چیزی به ذهنتون نمی رسه؟

mohsen_blid
02-12-2010, 20:28
سلام.

خب هر دو روش به یک معادله ختم میشه. در حقیقت این دو تا یک معادله هستند ولی طریقه ی نمایششون فرق میکنه.

تو این روش برای راحتی کار ، خودمون یه کاری کردیم که وقتی لاندا رو صفر بذاریم به مختصات یکی از نقاط برسیم و نقطه ی دیگه هم وقتی بدست بیاریم که لاندا برابر با 1 باشه.



سلام.

از اونجایی که تصمیم گرفتم دیگه برای سوالات اینچنینی (منظورم سوالاتیه که مربوط به دروس دبیرستان و دانشگاه میشه) جواب کامل رو نذارم تا خود کاربران تمرین بیشتری بکنن، بنابراین راه حل و جواب رو محبت نمیکنم :31: و فقط به راهنمایی های لازمه بسنده میکنم. بقیه اش به عنوان تمرین به عهده ی خودتون:31:

در سوال اول از این قانون استفاده کنین:

[ برای مشاهده لینک ، لطفا با نام کاربری خود وارد شوید یا ثبت نام کنید ] %28x%29%7D=x


و در سوال دوم هم از این قانون:

[ برای مشاهده لینک ، لطفا با نام کاربری خود وارد شوید یا ثبت نام کنید ] &plus;%5Clog_%7BA%7D%28Y%29=%5Clog_%7BA%7D%28XY%29


موفق باشین.
89/9/11





ممنون دی وی جونز عزیز
ولی من جواب رو بدست اورده بودم جواب -4 میشه اما تو کتاب جواب رو +2 میدونه
حالا من نمیدونم کتاب اشتباه میکنه یا من
ممنون میشم جواب رو بزارید

قله بلند
02-12-2010, 20:31
ممنون دی وی جونز عزیز
ولی من جواب رو بدست اورده بودم جواب -4 میشه اما تو کتاب جواب رو +2 میدونه
حالا من نمیدونم کتاب اشتباه میکنه یا من
ممنون میشم جواب رو بزارید
سلام. به جوابی هم که من دادم یه نگاهی بکنید

davy jones
02-12-2010, 21:17
من که از این دو قضیه نتونستم به اون logk برسم. شما چیزی به ذهنتون نمی رسه؟

گرچه حس میکنم این حرفم شبیه توجیهه ولب فکر کنم اون logk یه حالت خاص از قضیه ی کلی ای باشه که فرمودین و تو اون مساله از ما خواسته توی اون حالت خاص مساله رو حل کنیم.

بیشتر از این چیزی به ذهنم نمیرسه.:31:


ممنون دی وی جونز عزیز
ولی من جواب رو بدست اورده بودم جواب -4 میشه اما تو کتاب جواب رو +2 میدونه
حالا من نمیدونم کتاب اشتباه میکنه یا من
ممنون میشم جواب رو بزارید

دوست عزیز جناب قله بلند به درستی هر دو مساله رو حل کردند. پست شماره ی 3163

موفق باشین.
89/9/11

lebesgue
03-12-2010, 01:17
من یک مسئله کمکی مطرح می کنم:

فرض کنید من یک عدد بین 1 تا 128 انتخاب کردم و شما قراره با پرسیدن سوال های بله و خیر، این عدد رو پیدا کنید.
پیش پا افتاده ترین راه اینه که شروع کنید از 1، بپرسید آیا جواب 1 است؟ آیا 2 است؟...
اما یک روش هوشمندانه تر (که بهینه ترین الگوریتم هم هست) اینه که بپرسید آیا این عدد بین 1 تا 64 هست، اگر پاسخ مثبت بود، بپرسید آیا بین 1 تا 32 است و... به همین ترتیب بازه ها رو نصف کنید.
با این روش، چون 128 = 7^2 ، پس از 7 مرحله یعنی Log128 در مبنای 2، تعداد گزینه های ممکن به 1 می رسه و جواب پیدا می شه.

در حالت کلی اثبات می شه که برای پیدا کردن عددی بین 1 تا n ، به سقف Log n در مبنای 2، سوال نیاز است.

آیا میتونید شبیه چنین الگوریتمی رو برای مسئله خودتون پیدا کنید؟

mohsen_blid
03-12-2010, 08:30
سلام


[ برای مشاهده لینک ، لطفا با نام کاربری خود وارد شوید یا ثبت نام کنید ] E%7B2%7D+4%5E%7B-1%7D=%2849%29%5Cfrac%7B1%7D%7B4%7D+%5Cfrac%7B1%7D% 7B4%7D=12.5





[ برای مشاهده لینک ، لطفا با نام کاربری خود وارد شوید یا ثبت نام کنید ] B2%7D%5E%7B%28x+2%29%7D=6&space;%5CRightarrow&space;log_%7B2%7 D%5E%7B%28x+14%29%28x+2%29%7D=6&space;%5CRightarrow&space;2%5E %7B6%7D=%28x+14%29%28x+2%29&space;%5CRightarrow&space;x=-18&space;,&space;x=2

میگم، عجب کار سختیه این فرمول نویسی!

---------- Post added at 08:00 PM ---------- Previous post was at 07:58 PM ----------

پس چرا نمایش داده نمی شن؟


حالا نمایش داده شد:20:
من تو قسمت اخر مشکل دارم نمی دونم چطور دو جواب رو بدست اوردید مسئله دوم رو میگم؟
2
-
18

اقا خودم فهمیدم اشتباه کلی من تو حل این بود که 2 به توان 6 رو 32 محاسبه میکردم نه 64
واقعا به خودم امیدوار شدم

davy jones
03-12-2010, 13:06
من تو قسمت اخر مشکل دارم نمی دونم چطور دو جواب رو بدست اوردید مسئله دوم رو میگم؟
2
-
18

اقا خودم فهمیدم اشتباه کلی من تو حل این بود که 2 به توان 6 رو 32 محاسبه میکردم نه 64
واقعا به خودم امیدوار شدم

خب حالا میدونی چرا 18- قبول نیست و فقط 2 میتونه جواب باشه؟

موفق باشین.
89/9/12

mohsen_blid
03-12-2010, 13:36
خب حالا میدونی چرا 18- قبول نیست و فقط 2 میتونه جواب باشه؟

موفق باشین.
89/9/12
بله
1- لوگاریتم هیچ وقت منفی نمیگیره
دوم شرط وجود جواب هست
x>0

akpa
07-12-2010, 22:35
سلام

برای معادله ی =(1/x)+(1/y)+(1/z)+(1/z) 1

با فرض اینکه x,y,z,tدوبه دو متمایزند جواب طبیعی بیابید ونشان دهید بیش از یک دسته جواب وجود دارد
پیشاپیش ممنون وتشکرفراوان

Life24
09-12-2010, 19:38
[ برای مشاهده لینک ، لطفا با نام کاربری خود وارد شوید یا ثبت نام کنید ]

مشتق این از فرمول بدست میارم
اما دامنه اش چطور بدست میارید؟
چون باید در فاصله منفی بینهایت تا مثبت بی نهایت باشه
اینجا arc نوشتم جای اینورس(اگر درست نوشته باشم اینورس رو -منظور تانژانت اینورس بوده):دی)

Mohammad Hosseyn
09-12-2010, 20:09
[ برای مشاهده لینک ، لطفا با نام کاربری خود وارد شوید یا ثبت نام کنید ]

مشتق این از فرمول بدست میارم
اما دامنه اش چطور بدست میارید؟
چون باید در فاصله منفی بینهایت تا مثبت بی نهایت باشه
اینجا arc نوشتم جای اینورس(اگر درست نوشته باشم اینورس رو :دی)
دامنه arctan همون برد tan هست که میشه مجموعه R .

یعنی عبارت داخل تابع آرک (3x+2) باید در R باشه . که یعنی خود x در R هست .

Life24
09-12-2010, 20:15
دامنه arctan همون برد tan هست که میشه مجموعه R .

یعنی عبارت داخل تابع آرک (3x+2) باید در R باشه . که یعنی خود x در R هست .
خوب منفی بینهایت تا مثبت بی نهایت میشه همون R
خوب تا اینجا درست
اما در اینجا دامنه دقیقا چی میشه؟ میشه تایپش کنید.
راستی بردش هم بگید چون از منفی پی دوم تا مثبت پی دوم میشه میخوام ببینم چطور حساب میشه
ممنونم.

قله بلند
10-12-2010, 00:19
سلام.
من هنوز نفهمیدم این logk اینجا چه نقشی داره.
مثلاً فرض کنید عدد 17 هم به لیست آن 4 عددی که زیرشان خط کشیده شده اضافه شود. حالا فرض می کنیم که log5=2.1 که سقفش می شود عدد 3. این عدد 3 چه ارتباطی با تقسیم بندی ما داره؟ چون ما می تونیم بیشتر از 3 دسته تولید کنیم و این عددهای دو به دو اول را در آن دسته ها بریزیم و کارمان هم کاملاً بدون ایراد خواهد بود.

سلام
یه چیزهایی فهمیدم ولی به نظرم اساسی نیست.

ما می خواهیم این چند عدد دو به دو اول را طوری ب.م.م بگیریم که با یک گام، همه مقایسه بشن پس چه بهتره که با گام کمتری این کار صورت بگیره.

1-مثلاً دو عدد صحیح دو به دو اول داریم پس در یک گام می توانیم بگوییم:

برای مشاهده محتوا ، لطفا وارد شوید یا ثبت نام کنید

2-حالا سه عدد داریمف می توانیم در یک گام، ب.م.م همه این سه تا را به دست بیاوریم:

برای مشاهده محتوا ، لطفا وارد شوید یا ثبت نام کنید

3-حالا 4 تا از این اعداد داریم و می خواهیم با یک گام همه را با هم مقایسه کنیم:

برای مشاهده محتوا ، لطفا وارد شوید یا ثبت نام کنید
حالا چرا نمی توانیم بگوییم:

برای مشاهده محتوا ، لطفا وارد شوید یا ثبت نام کنید
در این حالت نیز با یک گام مساله را حل کردیم؟!!


شاید به این خاطر نمی تونیم اینجوری بگیم که نمی تونیم ترکیب خطی با 3 حاصلضرب یا بیشتر بنویسیم:
مثلاً می گوییم:

برای مشاهده محتوا ، لطفا وارد شوید یا ثبت نام کنید

در این مورد چیزی به ذهنتون می رسه؟ اصولاً حرف من درسته؟

Mohammad Hosseyn
10-12-2010, 00:42
خوب منفی بینهایت تا مثبت بی نهایت میشه همون R
خوب تا اینجا درست
اما در اینجا دامنه دقیقا چی میشه؟ میشه تایپش کنید.
راستی بردش هم بگید چون از منفی پی دوم تا مثبت پی دوم میشه میخوام ببینم چطور حساب میشه
ممنونم.
ببین عزیز ، کاری به منفی بینهایت و مثبت بی نهایت نداشته باش. این باعث میشه اشتباه کنی .

وقتی یه تابع خطی (مثل همون 3X+2) بخواد بردش R باشه باید خود x هاش R باشه.
اصلا همین عبارت 3x+2 با چه مقادیری از منفی بینهایت تا مثبت بی نهایت تغییر میکنه ؟؟ خوب معلومه که باید به x همه مقادیر بین دو بینهایت رو بدی تا تابع هم همینطور بین بینهایت تغییر کنه.

برد بری توابع arc ثابت هست. یعنی قبل محاسبه مشخصه که همینه. این دامنه هست که وابسته به عبارتیه که آرک میگیریم ازش.
البته تابع Arcu با arcu تفاوت داره.

برد در Arc همون بین -90 و +90 (درجه) هست.
ولی در acr برد هم بینهایت هست.
در واقع نوعی قرار داد هست. چون تابع arc در واقع تابع نیست. اگه بخوایم تبدیل به تابع بشه باید دامنش رو محدود کنیم تا تناوب عرضی نداشته باشه. (به ازا یک x ، دو y نداشته باشه)

حالا دلیل سادش اینه که این تابع معکوسه تانژانت هست. و تانژانت در بین این دو مقدار از منفی تا مثبت بینهایت تغییر میکنه.


به نظر من نمودار کمک میکنه راحت یاد بگیریو به خاطر داشته باشی (کلا در توابع مثلثاتی نمودار خیلی کلیدیه)

[ برای مشاهده لینک ، لطفا با نام کاربری خود وارد شوید یا ثبت نام کنید ]

AryanShining
10-12-2010, 08:31
سلام فكر مي كنم تاپيك رو درست اومدم
دنبال اثبات آخرين خاصيت جز صحيح هستم موقعي كه دبيرستان بودم بلدش بودم ولي الان يادم رفته
اين زير خاصيت رو مي نويسم

برای مشاهده محتوا ، لطفا وارد شوید یا ثبت نام کنید

davy jones
10-12-2010, 11:09
[ برای مشاهده لینک ، لطفا با نام کاربری خود وارد شوید یا ثبت نام کنید ]

مشتق این از فرمول بدست میارم
اما دامنه اش چطور بدست میارید؟
چون باید در فاصله منفی بینهایت تا مثبت بی نهایت باشه
اینجا arc نوشتم جای اینورس(اگر درست نوشته باشم اینورس رو -منظور تانژانت اینورس بوده):دی)

جواب محمدحسین خان درسته. ولی در تایید صحبتهای ایشون و برای اینکه بهتر متوجه بشین، به خدمتتون عارض بشم که اگر داشته باشیم:

دامنه = domain
برد = range

آنگاه برای تایین دامنه تابع [ برای مشاهده لینک ، لطفا با نام کاربری خود وارد شوید یا ثبت نام کنید ] همواره داریم:



[ برای مشاهده لینک ، لطفا با نام کاربری خود وارد شوید یا ثبت نام کنید ]


[ برای مشاهده لینک ، لطفا با نام کاربری خود وارد شوید یا ثبت نام کنید ] %7B-%5Cinfty-2%7D%7B3%7D%3Cx%3C%5Cfrac%7B&plus;%5Cinfty-2%7D%7B3%7D

[ برای مشاهده لینک ، لطفا با نام کاربری خود وارد شوید یا ثبت نام کنید ] r%7Bred%7D&space;domain%5C;&space;f=%5Cmathbb%7BR%7D%7D


از طرفی چون دامنه ی تابع f هیچ محدودیتی ایجاد نکرد و همچنین میدانیم که تابع ما یک به یک میباشد بنابراین برد تابع f هم اینچنین بدست میآید:


[ برای مشاهده لینک ، لطفا با نام کاربری خود وارد شوید یا ثبت نام کنید ]

[ برای مشاهده لینک ، لطفا با نام کاربری خود وارد شوید یا ثبت نام کنید ] nfty&space;%7Df%28x%29=&plus;%5Cfrac%7B%5Cpi&space;%7D%7B2%7D

[ برای مشاهده لینک ، لطفا با نام کاربری خود وارد شوید یا ثبت نام کنید ] rrow&space;range%5C;&space;%5C;&space;f:&space;%5C;&space;x%5Cin&space;%28-%5Cfrac%7B%5Cpi&space;%7D%7B2%7D,&plus;%5Cfrac%7B%5Cpi&space;%7D%7B 2%7D%29%7D


برای مشتق گرفتن از این تابع هم دقت میکنیم که تابع ما در حقیقت یک تابع مرکب است:

[ برای مشاهده لینک ، لطفا با نام کاربری خود وارد شوید یا ثبت نام کنید ] %5C;&space;%5CRightarrow&space;f%28x%29=g%28u%28x%29%29%5CRigh tarrow&space;%7Bf%7D%27%28x%29=%7Bu%7D%27%28x%29%7Bg%7D% 27%28u%28x%29%29=3%5Ctimes&space;%5Cfrac%7B1%7D%7B1&plus;%283 x&plus;2%29%5E%7B2%7D%7D=%7B%5Ccolor%7Bred%7D&space;%5Cfrac%7 B3%7D%7B9x%5E%7B2%7D&plus;12x&plus;5%7D%7D

موفق باشین.
89/9/19

قله بلند
10-12-2010, 11:29
سلام.

برای مشاهده محتوا ، لطفا وارد شوید یا ثبت نام کنید
به نظر من pairs یعنی تیم دو نفره نه زوج.

lebesgue
10-12-2010, 14:02
من یک مسئله کمکی مطرح می کنم:

فرض کنید من یک عدد بین 1 تا 128 انتخاب کردم و شما قراره با پرسیدن سوال های بله و خیر، این عدد رو پیدا کنید.
پیش پا افتاده ترین راه اینه که شروع کنید از 1، بپرسید آیا جواب 1 است؟ آیا 2 است؟...
اما یک روش هوشمندانه تر (که بهینه ترین الگوریتم هم هست) اینه که بپرسید آیا این عدد بین 1 تا 64 هست، اگر پاسخ مثبت بود، بپرسید آیا بین 1 تا 32 است و... به همین ترتیب بازه ها رو نصف کنید.
با این روش، چون 128 = 7^2 ، پس از 7 مرحله یعنی Log128 در مبنای 2، تعداد گزینه های ممکن به 1 می رسه و جواب پیدا می شه.

در حالت کلی اثبات می شه که برای پیدا کردن عددی بین 1 تا n ، به سقف Log n در مبنای 2، سوال نیاز است.

آیا میتونید شبیه چنین الگوریتمی رو برای مسئله خودتون پیدا کنید؟

لم: اگر [ برای مشاهده لینک ، لطفا با نام کاربری خود وارد شوید یا ثبت نام کنید ](x_1x_2...x_n,y_1y_2...y_ m)=1

آنگاه برای هر [ برای مشاهده لینک ، لطفا با نام کاربری خود وارد شوید یا ثبت نام کنید ] ,%5C:%201%5Cleq%20j%5Cleq%20m

داریم:[ برای مشاهده لینک ، لطفا با نام کاربری خود وارد شوید یا ثبت نام کنید ](x_i,y_j)=1

(gcd همان ب.م.م است.)

فرض کنید میخواین نشون بدید که 16 عدد [ برای مشاهده لینک ، لطفا با نام کاربری خود وارد شوید یا ثبت نام کنید ]{16} دو به دو نسبت به هم اول هستند (gcd اونها 1 هست).
ابتدایی ترین روش اینه که شروع کنید هر جفت ممکن رو بررسی کنید، که این روش به [ برای مشاهده لینک ، لطفا با نام کاربری خود وارد شوید یا ثبت نام کنید ]{16}{2}=120 مرحله نیاز داره.
اما الگوریتمی هست که میتونید با [ برای مشاهده لینک ، لطفا با نام کاربری خود وارد شوید یا ثبت نام کنید ]{2}^{16}=4 مرحله این کار رو انجام بدید.

مرحله1: نشون می دید [ برای مشاهده لینک ، لطفا با نام کاربری خود وارد شوید یا ثبت نام کنید ](n_1...n_8,n_9...n_{16})= 1

از اینجا میتونید نتیجه بگیرید که gcd هر عدد از 8 تای اول، با هر عدد از 8 تای دوم، برابر با 1 هست.
حالا باید در هر کدام از مجموعه های 8 تایی، اول بودن اعداد نسبت به هم رو چک کنید.

مرحله2: نشون میدید [ برای مشاهده لینک ، لطفا با نام کاربری خود وارد شوید یا ثبت نام کنید ](n_1n_2n_3n_4n_9n_{10}n_{ 11}n_{12},n_5n_6n_7n_8n_{13}n_{14}n_{15}n_{16})=1

از اینجا میتونید نتیجه بگیرید که در هر کدوم از مجموعه های 8 تایی، 4 تای اول نسبت به 4 تای دوم، اول هستند.

در مرحله 3، ما 4 مجموعه 4 عضوی داریم که بطور مشابه از هر مجموعه 2 تا رو در سمت چپ gcd و دو تای دیگه رو در سمت راست gcd انتخاب می کنیم.
مرحله 4، 8 مجموعه 2 عضوی داریم که از هر مجموعه یکی را در طرف چپ و یکی را در طرف راست انتخاب می کنیم و تمام.

نکته ای که اینجا باید در نظر بگیرید این هست که شما اگر 1 میلیارد مجموعه 16 عضوی هم داشته باشید که بخواین در مورد هر کدوم نشون بدید دو به دو نسبت به هم اول هستند، با این روشی که خدمتتون عرض کردم، به طور موازی و همزمان اینکار رو میتونید انجام بدید و در کل به 4 مرحله نیاز دارین.

اثبات برای حالت کلی دشوار نیست، از استقرای ریاضی میتونید استفاده کنید.

قله بلند
10-12-2010, 19:47
سلام. ممنونم جناب 1233445566
ای کاش زودتر این راه حل رو می دادید. خیلی گشتم.
آیا این لم دارای قضیه ای است؟ از چه آدرسی یا کتابی می شه به اون رسید؟

استنباط شما از جمله
A set of [logk] pairs of numbers derived from the ni
سقف logk مرحله است؟ چون شما فرمودید که 4 مرحله داریم. من این رو مثلاً 4 جفت از اعداد مشتق شده در نظر می گرفتم.

قله بلند
10-12-2010, 20:33
سلام جناب 1233445566عزیز

لطفاً ببینید درست می گم:
برای چهار عدد
a=6 b=55 c=91 d=19*11


فرض: می دانیم که این 4 عدد دو به دو نسبت به هم اولند و می خواهیم برسیم به اینکه

برای مشاهده محتوا ، لطفا وارد شوید یا ثبت نام کنید


اثبات:

برای مشاهده محتوا ، لطفا وارد شوید یا ثبت نام کنید[/B]

حالا می خواهیم از حکم به فرض برویم چون گفته اگر و تنها اگر.


چون سقف logk برابر عدد 2 است، پس باید در دو مرحله کارها انجا شود.

مرحله اول:

برای مشاهده محتوا ، لطفا وارد شوید یا ثبت نام کنید

مرحله دوم:

برای مشاهده محتوا ، لطفا وارد شوید یا ثبت نام کنید


و دیگر لزومی به چک کردن 4 حالت باقی مانده نیست.

درست گفتم؟

در این جا اگر 3 تا عدد صحیح دو به دو نسبت به هم اول داشتیم نیز 2 مرحله نیاز داشتیم. چون سقف جزء صحیح log3 می شود عدد 2.

lebesgue
10-12-2010, 21:20
ای کاش زودتر این راه حل رو می دادید. خیلی گشتم.
آیا این لم دارای قضیه ای است؟ از چه آدرسی یا کتابی می شه به اون رسید؟

اثباتش خیلی سادست، با برهان خلف.
فرض کنید i و j ای وجود داشته باشه که این قضیه برقرار نباشه، یعنی:

[ برای مشاهده لینک ، لطفا با نام کاربری خود وارد شوید یا ثبت نام کنید ](x_i,y_j)=a%5Cneq%2 01%5C:%20%5C:%20%5Crightarrow%5C:%20%5C:%20a%5Cmid %20x_i%20%5C:%5C:%20and%5C:%5C:%20a%5Cmid%20y_j%5C %5C%20%5Crightarrow%20a%5Cmid%20x_1...x_n%20%5C:%5 C:%20and%5C:%5C:%20a%5Cmid%20y_1...y_m%5C:%20%5C:% 20%5Crightarrow%20a%5Cmid%20gcd(x_1...x_n,y_1...y_ m)
که نتیجه بدست اومده با فرض در تناقضه.



استنباط شما از جمله
A set of [logk] pairs of numbers derived from the ni
سقف logk مرحله است؟ چون شما فرمودید که 4 مرحله داریم. من این رو مثلاً 4 جفت از اعداد مشتق شده در نظر می گرفتم.
فرقی نمی کنه، چون بحث (اگر اشتباه نکنم) مربوط به بهینه سازی الگوریتم ها بود، من از مرحله استفاده کردم.
یعنی محاسبه هر gcd رو یک مرحله فرض کردم.
میشه اینطور گفت که شما برای تشخیص اینکه آیا یک مجموعه 16 عضوی، Pairwise coprime هست یا نه (اعضای اون دو به دو نسبت به هم اول هستند یا نه)، کافیه 4 تا gcd خاص رو بدونید (و البته نه اینکه هر 4 تا gcd این خاصیت را داشته باشند).

در مورد پست بالایی، بله درست گفتید.

قله بلند
11-12-2010, 11:13
سلام
شما فرمودید که در بخش بهینه سازی الگوریتم ها این لم آمده است.
من می گم چرا این کار رو نمی کنن:
Gcd(abc,d)=1

وقتی که می دانیم که این 4 عدد صحیح، دو به دو نسبت به هم اولند. با یک گام همه چیز حله.

قله بلند
12-12-2010, 15:42
سلام
ببخشید که دوباره سوال می کنم چون دچار مشکل شدم. خواهش می کنم پاسخ بدید چون واقعاً گیر کردم و اعصابم دیگه داره خط خطی می شه.
1-

برای مشاهده محتوا ، لطفا وارد شوید یا ثبت نام کنید

2-

برای مشاهده محتوا ، لطفا وارد شوید یا ثبت نام کنید

3-


برای مشاهده محتوا ، لطفا وارد شوید یا ثبت نام کنید


5-حالا فرض کنید می خواهم عدد پنجم را وارد کنم تا همگی دو به دو اول باشند. حالا نمی دانم چه کار کنم؟
در این فرمول ها، همیشه از اطلاعات گام قبل استفاده می شود و قبل از ورود عدد پنجم ما می دانیم که 4 عدد قبلی دو به دو اول هستند ولی چه طوری عدد پنجم را با این خصوصیات وارد کنم؟



برای مشاهده محتوا ، لطفا وارد شوید یا ثبت نام کنید

در اینجا عدد 6 نسبت به 11 اول است ولی نمی شود حکم داده که نسبت به 3 عدد دیگر نیز اول باشد. پس چه طوری گام سوم رو وارد کنم؟

lebesgue
12-12-2010, 18:24
وقتی که می دانیم که این 4 عدد صحیح، دو به دو نسبت به هم اولند. با یک گام همه چیز حله.

خب اگر بدونید دو به دو نسبت به هم اولند که دیگه چیزی برای حل نمی مونه!
مسئله اینجاست که به شما یک مجموعه مثلا 10 عضوی میدن و شما قراره با صرف کمترین وقت و محاسبات، تعیین کنید که آیا این مجموعه Pairwise coprime هست یا نه.
من روندی که شما میخواین طی کنید رو متوجه نمی شم، اگر هنوز ابهامی هست، صورت مسئله ای که باهاش درگیر هستید رو بطور دقیق اینجا بنویسید.





در این فرمول ها، همیشه از اطلاعات گام قبل استفاده می شود و قبل از ورود عدد پنجم ما می دانیم که 4 عدد قبلی دو به دو اول هستند ولی چه طوری عدد پنجم را با این خصوصیات وارد کنم؟

همه مسئله مربوط به همون لم* که نوشتم میشه، پیشنهاد می کنم یه مقدار روش فکر کنید.
شما کافیه بدست بیارید gcd(e , abcd) =1 تا بتونید نتیجه بگیرید e نسبت به هر 4 عدد a , b , c , d اول هست.

*لم معمولاً به قضیه ای گفته می شه که ازش برای اثبات قضایای مهمتر استفاده میشه، یعنی خودش یک قضیه هست.

lebesgue
12-12-2010, 19:31
سلام فكر مي كنم تاپيك رو درست اومدم
دنبال اثبات آخرين خاصيت جز صحيح هستم موقعي كه دبيرستان بودم بلدش بودم ولي الان يادم رفته
اين زير خاصيت رو مي نويسم

برای مشاهده محتوا ، لطفا وارد شوید یا ثبت نام کنید
این چیزیه که الان به ذهنم میرسه، نمیدونم چقدر بر راه حل مورد نظر شما منطبق هست.
کافیه برای x در بازه (1 , 0] این مطلب رو نشون بدید، چون در حالت کلی قسمت های صحیح از جزء صحیح بیرون میان و از طرفین خط می خورن.


[ برای مشاهده لینک ، لطفا با نام کاربری خود وارد شوید یا ثبت نام کنید ] nx%20%5Cright%20%5Crfloor=k%5C:%20%5C:%20%5Crighta rrow%5C:%20%5C:%20k%5Cleq%20nx%3C%20k+1%5C:%20%5C: %20%5Crightarrow%20%5C:%20%5C:%20%5Cfrac{k}{n}%5Cl eq%20x%3C%20%5Cfrac{k+1}{n}%5C%5C%5C%5C%200%5Cleq% 20i%5Cleq%20n-k-1:%5C:%20%5C:%20x+%5Cfrac{i}{n}%3C%5Cfrac{k+1}{n}+ %5Cfrac{i}{n}%5Cleq%20%5Cfrac{n}{n}=1%5C:%20%5C:%2 0%5Crightarrow%20%5Cleft%20%5Clfloor%20x+%5Cfrac{i }{n}%20%5Cright%20%5Crfloor=0%5C%5C%5C%5C%20n-k%5Cleq%20i%5Cleq%20n-1:%5C:%20%5C:%20x+%5Cfrac{i}{n}%5Cgeq%20%5Cfrac{k} {n}+%5Cfrac{i}{n}%5Cgeq%20%5Cfrac{n}{n}=1%5C:%20%5 C:%20%5Crightarrow%20%5Cleft%20%5Clfloor%20x+%5Cfr ac{i}{n}%20%5Cright%20%5Crfloor=1%5C%5C%5C%5C%20%5 Csum_{i=0}^{n-1}%5Cleft%20%5Clfloor%20x+%5Cfrac{i}{n}%20%5Cright %20%5Crfloor=%5Csum_{i=n-k}^{n-1}1=k

(n عدد صحیح مثبت است)

قله بلند
12-12-2010, 20:20
سلام
جناب 1233445566 عزیز
می تونم خواهش کنم یک مثال برای

برای مشاهده محتوا ، لطفا وارد شوید یا ثبت نام کنید
و یک مثال برای

برای مشاهده محتوا ، لطفا وارد شوید یا ثبت نام کنید
بزنید. من هر چی تلاش می کنم نمی شه. می خوام ببینم شما چطوری این تقسیم بر دو رو انجام می دید. طبق همون لم.

lebesgue
12-12-2010, 20:53
سلام
جناب 1233445566 عزیز
می تونم خواهش کنم یک مثال برای

برای مشاهده محتوا ، لطفا وارد شوید یا ثبت نام کنید
و یک مثال برای

برای مشاهده محتوا ، لطفا وارد شوید یا ثبت نام کنید
بزنید. من هر چی تلاش می کنم نمی شه. می خوام ببینم شما چطوری این تقسیم بر دو رو انجام می دید. طبق همون لم.

برای اولی: میتونید خودتون n6 , n7 , n8 رو به مجموعه اضافه کنید (البته باید هر سه، نسبت به 5 عدد دیگه اول باشند).
بلدید در سه مرحله نشون بدید که این مجموعه 8 عضوی Pairwise coprime هست. خب از اینجا نتیجه می گیرید که هر زیر مجموعه اونهم Pairwise coprime هست!:31:
میتونید برای سادگی هرسه n6 , n7 , n8 رو برابر 1 انتخاب کنید.
دومی هم بطور مشابه!

AryanShining
12-12-2010, 21:27
این چیزیه که الان به ذهنم میرسه، نمیدونم چقدر بر راه حل مورد نظر شما منطبق هست.
کافیه برای x در بازه (1 , 0] این مطلب رو نشون بدید، چون در حالت کلی قسمت های صحیح از جزء صحیح بیرون میان و از طرفین خط می خورن.


[ برای مشاهده لینک ، لطفا با نام کاربری خود وارد شوید یا ثبت نام کنید ] nx%20%5Cright%20%5Crfloor=k%5C:%20%5C:%20%5Crighta rrow%5C:%20%5C:%20k%5Cleq%20nx%3C%20k+1%5C:%20%5C: %20%5Crightarrow%20%5C:%20%5C:%20%5Cfrac{k}{n}%5Cl eq%20x%3C%20%5Cfrac{k+1}{n}%5C%5C%5C%5C%200%5Cleq% 20i%5Cleq%20n-k-1:%5C:%20%5C:%20x+%5Cfrac{i}{n}%3C%5Cfrac{k+1}{n}+ %5Cfrac{i}{n}%5Cleq%20%5Cfrac{n}{n}=1%5C:%20%5C:%2 0%5Crightarrow%20%5Cleft%20%5Clfloor%20x+%5Cfrac{i }{n}%20%5Cright%20%5Crfloor=0%5C%5C%5C%5C%20n-k%5Cleq%20i%5Cleq%20n-1:%5C:%20%5C:%20x+%5Cfrac{i}{n}%5Cgeq%20%5Cfrac{k} {n}+%5Cfrac{i}{n}%5Cgeq%20%5Cfrac{n}{n}=1%5C:%20%5 C:%20%5Crightarrow%20%5Cleft%20%5Clfloor%20x+%5Cfr ac{i}{n}%20%5Cright%20%5Crfloor=1%5C%5C%5C%5C%20%5 Csum_{i=0}^{n-1}%5Cleft%20%5Clfloor%20x+%5Cfrac{i}{n}%20%5Cright %20%5Crfloor=%5Csum_{i=n-k}^{n-1}1=k

(n عدد صحیح مثبت است)
دستت درد نکنه
این چیزی که شما نوشتی تو ذهنم نبود دنباله یه چیز دیگم احتمالا تا چند روز دیگه تو کتاب ها پیداش می کنم اگه پیدا شد حتما اینجا می ذارمش
ضمنا در مورد مطلبی که نوشتی بگم خیلی جالب و به کارم خورد و حتما این رو به استاد نشون می دم واقعا ممنون
:10:

قله بلند
12-12-2010, 22:11
سلام
ببینید، من این کار رو انجام می دم:

برای مشاهده محتوا ، لطفا وارد شوید یا ثبت نام کنید

حس می کنم درست نیست.
اینهمه خواهش کردم که راه حلش رو بنویسید.

lebesgue
12-12-2010, 22:39
سلام
ببینید، من این کار رو انجام می دم:

برای مشاهده محتوا ، لطفا وارد شوید یا ثبت نام کنید

حس می کنم درست نیست.
اینهمه خواهش کردم که راه حلش رو بنویسید.

الان اینها چی هست؟
بطور واضح بفرمایید صورت مسئلتون چیه که "این کار رو" براش انجام می دید!

قله بلند
13-12-2010, 00:29
سلام


سوال: فرض کنید n1 و n2 و n3 و n4 دو به دو نسبت به هم اولند اگر و فقط اگر
gcd(n1n2,n3n4)=gcd(n1n3,n2n4)=1

یعنی با دونستن همین دو تا ب.م.م، میشه ثابت کرد که این 4 تا، دو به دو نسبت به هم اولند.

حالا در قسمت دوم سوال یه جورایی می خواد که با دونستن این مطلب، این قضیه رو تعمیم بدیم که:


در حالت کلی تر نشان دهید که n1,n2,….,nk دو به دو نسبت به هم اولند اگر و فقط اگر مجموعه ای از سقف logk جفت از اعداد مشتق شده از ni، نسبت به هم اول باشند.


یعنی منظورش از سقف log4 همون دو تا ب.م.م ای هست که در خط سوم آوردم.

1-مثلاً سقف log2=1 یعنی با gcd(n1,n2)=1 مساله حله.
2-مثلاً سقف log3=2 یعنی با gcd(n1n2,n3)=1 و gcd(n1n3,n2)=1مساله حله.
3-مثلاً سقف log4=2 و ....

حالا اگر بخواهیم برای k=5 این قضیه رو اثبات کنیم، باید 3 تا از این gcd ها داشته باشیم و برای k=6 و 7 و 8.
اگر من بتونم حداقل برای 8 تای اول این 3 تا gcd رو بنویسم خوبه.

یه جورایی موندم چه طوریه n1n2n3 رو به دو قسمت تقسیم کنم و با دو نیمه n4n5n6 مقایسه کنم؟ و همچنین برای n5 و n7 و n8 که بتونم ازشون 3 تا gcd بیرون بیارم.

مثلاً با حساب و کتاب که وقت گیر هم هست برای k=5 به این سه تا gcd رسیدم:

برای مشاهده محتوا ، لطفا وارد شوید یا ثبت نام کنید

و برای k=6

برای مشاهده محتوا ، لطفا وارد شوید یا ثبت نام کنید

ولی با اون لم نمی دونم چه طوری می شه به این سه تا gcd رسید؟

msm43njn
13-12-2010, 10:17
جواب این سوال رو بطور مفصل می‌خوام:
تابع مولد معمولی مساله یافتن تعداد طرق انتخاب یک زیر مجموعه n>=11 عضوی {n,...3,2,1} به طوری که این مجموعه‌ها شامل هیچ دو عدد صحیح متوالی نباشد.
مطلب مربوط به ریاضیات گسسته بخش توابع مولد هست.

farzaneh*f
14-12-2010, 16:29
بچه ها منم دو تا سوال دارم که تا شنبه باید جوابشو ببرم سر کلاس ممنون می شم اگه یکی لطف کنه کمکم کنه :

1. lim 1/x(cot x - 1/x) , x-->0
حد 1 روی x در (کتانژانت x منهای 1 روی x ) و x میل می کنه به صفر

2. f(x) = 5(X^2) + a/(X^5) , X,a>0
کوچکترین a را پیدا کنید که به ازای هر x ،مقدار این تابع (5 در xبه توان 2 + a روی xبه توان 5) بزرگتر یامساوی 24 باشه

پیشاپیش ممنون:11: :31:

قله بلند
14-12-2010, 20:12
سلام
من برای n1 و n2 و n3 و n4 و n5 و n6 به این سه تا gcd رسیدم:

برای مشاهده محتوا ، لطفا وارد شوید یا ثبت نام کنید
که وقتی تجزیه یک عنصری روش انجام می شه:

برای مشاهده محتوا ، لطفا وارد شوید یا ثبت نام کنید[/SIZE]
به جای 15 ب.م.م دو عضوی.

حالا اگر برای 5 عضوی بخواهیم، n6 را از سه تا gcd بالا، حذف می کنیم و سه تا ب.م.م پایین هم حذف می شوند.
در ضمن، گام هایی که بر می دارند نیز به سقف logk ارتباط دارد.

درست گفتم؟


---------- Post added at 08:12 PM ---------- Previous post was at 08:11 PM ----------


بچه ها منم دو تا سوال دارم که تا شنبه باید جوابشو ببرم سر کلاس ممنون می شم اگه یکی لطف کنه کمکم کنه :

1. lim 1/x(cot x - 1/x) , x-->0
حد 1 روی x در (کتانژانت x منهای 1 روی x ) و x میل می کنه به صفر

2. f(x) = 5(X^2) + a/(X^5) , X,a>0
کوچکترین a را پیدا کنید که به ازای هر x ،مقدار این تابع (5 در xبه توان 2 + a روی xبه توان 5) بزرگتر یامساوی 24 باشه

پیشاپیش ممنون:11: :31:
سلام
آیا تقسیم عدد 1- بر عدد صفرف احتیاج به رفع ابهام داره؟ من این رو یادم رفته. اگر احتیاج داره، که جواب می شه عدد صفر و اگر احتیاج نداره می تونه، منهای بی نهایت و مثبت بی نهایت باشه.

farzaneh*f
14-12-2010, 20:45
سلام
آیا تقسیم عدد 1- بر عدد صفرف احتیاج به رفع ابهام داره؟ من این رو یادم رفته. اگر احتیاج داره، که جواب می شه عدد صفر و اگر احتیاج نداره می تونه، منهای بی نهایت و مثبت بی نهایت باشه.


می شه راه حل تونم توضیح بدین :11:

davy jones
14-12-2010, 21:15
بچه ها منم دو تا سوال دارم که تا شنبه باید جوابشو ببرم سر کلاس ممنون می شم اگه یکی لطف کنه کمکم کنه :

1. lim 1/x(cot x - 1/x) , x-->0
حد 1 روی x در (کتانژانت x منهای 1 روی x ) و x میل می کنه به صفر

2. f(x) = 5(X^2) + a/(X^5) , X,a>0
کوچکترین a را پیدا کنید که به ازای هر x ،مقدار این تابع (5 در xبه توان 2 + a روی xبه توان 5) بزرگتر یامساوی 24 باشه

پیشاپیش ممنون:11: :31:

1- :

[ برای مشاهده لینک ، لطفا با نام کاربری خود وارد شوید یا ثبت نام کنید ] %28%5Ccot&space;%28x%29-%5Cfrac%7B1%7D%7Bx%7D%29%7D=%5Clim_%7Bx&space;%5Cto&space;0%7D %5Cfrac%7B1%7D%7B%5Cfrac%7Bx%7D%7B%5Ctan&space;%28x%29%7 D-1%7D=%5Clim_%7Bx&space;%5Cto&space;0%7D%5Cfrac%7B%5Ctan&space;%28x%2 9%7D%7Bx-%5Ctan&space;%28x%29%7D&space;%5Cto&space;%5Cinfty&space;%5CRightarrow&space;%7B %5Ccolor%7Bred%7D&space;limit%5C;&space;is%5C;&space;exist%7D


این حد به سمت بینهایت میره و در حقیقت حد نداره. (درستش این بود که بنویسم: limit is not exist :31:)


-------------------

2- :

[ برای مشاهده لینک ، لطفا با نام کاربری خود وارد شوید یا ثبت نام کنید ] %5E%7B5%7D%7D%5CRightarrow&space;%7Bf%7D%27%28x%29=10x-%5Cfrac%7B5a%7D%7Bx%5E%7B4%7D%7D=0%5CRightarrow&space;x_ %7Bmin%7D=%5Csqrt[5]%7B%5Cfrac%7Ba%7D%7B2%7D%7D%5CRightarrow&space;y_%7Bmin% 7D=f%28x_%7Bmin%7D%29=5%5Csqrt[5]%7B%5Cfrac%7Ba%5E%7B2%7D%7D%7B4%7D%7D&plus;2=24%5CRight arrow&space;%7B%5Ccolor%7Bred%7D&space;%5Cmathbf%7Ba%7D%7D=%5C sqrt%7B4%5Ctimes&space;%28%5Cfrac%7B22%7D%7B5%7D%29%5E%7 B5%7D%7D%5C;&space;%5C;&space;%7B%5Ccolor%7Bred%7D&space;%5Capprox&space;8 1.22%7D


موفق باشین.
89/9/23

lebesgue
14-12-2010, 21:15
بچه ها منم دو تا سوال دارم که تا شنبه باید جوابشو ببرم سر کلاس ممنون می شم اگه یکی لطف کنه کمکم کنه :

1. lim 1/x(cot x - 1/x) , x-->0
حد 1 روی x در (کتانژانت x منهای 1 روی x ) و x میل می کنه به صفر

2. f(x) = 5(X^2) + a/(X^5) , X,a>0
کوچکترین a را پیدا کنید که به ازای هر x ،مقدار این تابع (5 در xبه توان 2 + a روی xبه توان 5) بزرگتر یامساوی 24 باشه

پیشاپیش ممنون:11: :31:
راهنمایی:
برای اولی، صورت و مخرج رو در xsinx ضرب کنید و با دو بار استفاده از قاعده هوپیتال، جواب بدست میاد 1/3-
برای دومی، مینیمم تابع رو با مشتق گیری بدست بیارید (بر حسب a) و برابر با 24 قرار بدید.

ویرایش:
البته من فرض کردم (cot x - 1/x) در صورت کسر هست وگرنه پاسخ davy jones عزیز درسته.

lebesgue
14-12-2010, 21:47
دوست عزیز قله بلند،
من در پست #3189 مسئله رو برای حالت خاصی که k=16 هست حل کردم.
با استقرای ریاضی میتونید قضیه:

نشان دهید که n1,n2,….,nk دو به دو نسبت به هم اولند اگر و فقط اگر مجموعه ای از سقف logk جفت از اعداد مشتق شده از ni، نسبت به هم اول باشند.
برای حالت خاصی که k توانی از 2 هست (یعنی k=2^t) اثبات کنید.

حالتی که k توانی از 2 نباشه رو هم میتونید به حالتی که باشه تبدیل کنید، کافیه تعدادی 1 به مجموعه تون اضافه کنید تا تعداد اعضای مجموعه برابر با توانی از 2 بشه.

به عنوان مثال، قضیه برای حالت k=8 به این صورت هست:

n1,n2,….,n8 دو به دو نسبت به هم اولند اگر و فقط اگر مجموعه ای از 3 جفت از اعداد مشتق شده از ni، نسبت به هم اول باشند.

اون سه جفت میتونن اینها باشن:

[ برای مشاهده لینک ، لطفا با نام کاربری خود وارد شوید یا ثبت نام کنید ](n_1n_2n_3n_4,n_5n_ 6n_7n_8)%5C%5Cgcd(n_1n_2n_5n_6,n_3n_4n_7n_8)%5C%5C gcd(n_1n_3n_5n_7,n_2n_4n_6n_8)

---------- Post added at 09:47 PM ---------- Previous post was at 09:44 PM ----------

برای حالت k=6 ، میتونید فرض کنید که n7 و n8 برابر 1 هستن، در اینصورت همون روابط بالا تبدیل میشن به:

[ برای مشاهده لینک ، لطفا با نام کاربری خود وارد شوید یا ثبت نام کنید ](n_1n_2n_3n_4,n_5n_ 6)%5C%5Cgcd(n_1n_2n_5n_6,n_3n_4)%5C%5Cgcd(n_1n_3n_ 5,n_2n_4n_6)

farzaneh*f
14-12-2010, 23:19
راهنمایی:
برای اولی، صورت و مخرج رو در xsinx ضرب کنید و با دو بار استفاده از قاعده هوپیتال، جواب بدست میاد 1/3-
برای دومی، مینیمم تابع رو با مشتق گیری بدست بیارید (بر حسب a) و برابر با 24 قرار بدید.

ویرایش:
البته من فرض کردم (cot x - 1/x) در صورت کسر هست وگرنه پاسخ davy jones عزیز درسته.

بله درسته تو صورت کسر بود من نتونستم خوب تایپ کنم :31:

خیییییییییییلی ممنون بچه ها ؛ قله ی بلند ، davy jones ، و 1233445566 از هر سه نفرتون بی نهایت ممنونم :11::11::11:






2- :

[ برای مشاهده لینک ، لطفا با نام کاربری خود وارد شوید یا ثبت نام کنید ] %5E%7B5%7D%7D%5CRightarrow&space;%7Bf%7D%27%28x%29=10x-%5Cfrac%7B5a%7D%7Bx%5E%7B4%7D%7D=0%5CRightarrow&space;x_ %7Bmin%7D=%5Csqrt[5]%7B%5Cfrac%7Ba%7D%7B2%7D%7D%5CRightarrow&space;y_%7Bmin% 7D=f%28x_%7Bmin%7D%29=5%5Csqrt[5]%7B%5Cfrac%7Ba%5E%7B2%7D%7D%7B4%7D%7D&plus;2=24%5CRight arrow&space;%7B%5Ccolor%7Bred%7D&space;%5Cmathbf%7Ba%7D%7D=%5C sqrt%7B4%5Ctimes&space;%28%5Cfrac%7B22%7D%7B5%7D%29%5E%7 B5%7D%7D%5C;&space;%5C;&space;%7B%5Ccolor%7Bred%7D&space;%5Capprox&space;8 1.22%7D


موفق باشین.
89/9/23






فقط ببخشید یه اشکال دیگه برام پیش اومده ، من متشق این عبارت رو من خودم به صورت [ برای مشاهده لینک ، لطفا با نام کاربری خود وارد شوید یا ثبت نام کنید ] =%2010x%20-%20%5Cfrac%7B5a%7D%7Bx%5E%7B6%7D%7D به دست میارم که نهایتش جوابم می شه 149.25 (به طور تقریبی):41: می شه بگین اشکال کارم کجاست؟ ممنون[ برای مشاهده لینک ، لطفا با نام کاربری خود وارد شوید یا ثبت نام کنید ]

قله بلند
15-12-2010, 13:31
سلام
جناب 1233445566 عزیز، خدا خیرتون بده. اگه بدونید، دیگه بریده بودم. قبل از اینکه به این تاپیک سر بزنم، با خودم گفتم که خدا کنه پاسخی اومده باشه.
وقتی پاسخ رو دیدم و فهمیدم چه اشتباهی می کردم، از ته دل دعاتون کردم.

می دونید چه اشتباهی می کردم؟ من گمان می کردم از اون ب.م.م های دو تایی با این روش کم می شه یعنی مثلاً برای اثبات اینکه 16 عدد صحیح، دو به دو اول باشند، به صورت ساده می تونی 120 تا ب.م.م. بنویسیم که من گمان می کردم باید تعداد اینها کم بشه تا بهینه بشن که اشتباه فکر می کردم .
درست این بود که با چهار تا ب.م.م کاری کنیم که اون 120 تا پوشش داده بشن نه اینکه از اون 120 تا کم بشن.

قله بلند
15-12-2010, 13:55
سلام
امیدوارم اینبار دیگه اذییتون نکنم.
وقتی دو تا جمله زیر رو در هم ضرب می کنم بر اساس قانون هورنر، n به توان 2 تا ضرب انجام می دم.

برای مشاهده محتوا ، لطفا وارد شوید یا ثبت نام کنید
یعنی ما نهایتاً به 4 تا ضرب احتیاج داریم.
حالا یه حالت دیگه ای برای نمایش چند جمله ای هست که نمایش نقطه ای به جای نمایش چند جمله ای است که به وسیله ماتریس نمایش داده می شه. یعنی در ماتریس A، x ها نمایش داده می شه و در ماتریس بعدی، ضرایب و در ماتریش آخر، مقادیر(طبق نمایش نقطه ای زیر)

برای مشاهده محتوا ، لطفا وارد شوید یا ثبت نام کنید
حالا به جای اینکه اون مارتیس ها رو ضرب کنیم و معکوس کنیم که به زمان n به قوه 3 احتیاج داره می یان اون ماتریس رو با فرمول لاکرانژ محاسبه می کنن.
طرز نوشتن این فرمول رو خوب نفهمیدم. مثلاً می گه که

برای مشاهده محتوا ، لطفا وارد شوید یا ثبت نام کنید
و یک حاصل ضربی رو در صورت و یک حاصل ضربی رو در مخرج انجام می ده و اونها رو در yk ها ضرب می کنه و همه رو با هم جمع می کنه.
می شه این عملیت رو مثلاً برای سه نقطه

برای مشاهده محتوا ، لطفا وارد شوید یا ثبت نام کنید
انجام بدید تا ببینم چه طوری به j و k مقدرا می دید؟

Paradise_human
15-12-2010, 22:56
سلام ...
میخوام سری فوریه ی این تابع رو بنویسم ولی نیمیدونم (f(x ش چی میشه !
قسمت قرمز تابع مورد نظر ماست.

[ برای مشاهده لینک ، لطفا با نام کاربری خود وارد شوید یا ثبت نام کنید ]

lebesgue
16-12-2010, 18:39
سلام
امیدوارم اینبار دیگه اذییتون نکنم.
...

اینها رو بلد نیستم!:31:

---------- Post added at 06:39 PM ---------- Previous post was at 06:39 PM ----------


سلام ...
میخوام سری فوریه ی این تابع رو بنویسم ولی نیمیدونم (f(x ش چی میشه !
قسمت قرمز تابع مورد نظر ماست.

[ برای مشاهده لینک ، لطفا با نام کاربری خود وارد شوید یا ثبت نام کنید ]

تابع دو ضابطه ای هست، در هر بازه یک معادله خط که باید بدست بیارید.
در محاسبه ضرایب سری هم باید انتگرال رو در هر بازه جدا محاسبه کنید.

پیشنهاد می کنم اول سری فوریه f(x) -1 رو که ضابطه اش در هر بازه به ترتیب x و x- هست،
محاسبه کنید، در انتهای جواب رو +1 کنید.
همچنین دقت کنید چون تابع زوج هست، ضرایب سینوسها صفر هستن.

davy jones
16-12-2010, 21:07
پیشنهاد می کنم اول سری فوریه f(x) -1 رو که ضابطه اش در هر بازه به ترتیب x و x- هست،
محاسبه کنید، در انتهای جواب رو +1 کنید.
همچنین دقت کنید چون تابع زوج هست، ضرایب سینوسها صفر هستن.

البته منظور دوست عزیزمون احتمالا این بود که در هر بازه [ برای مشاهده لینک ، لطفا با نام کاربری خود وارد شوید یا ثبت نام کنید ] i&space;%7D%5C;&space;%5C;&space;%5Cmathbf%7B,%7D%5C;&space;%5C;&space;%5Cfrac%7 B-x%7D%7B%5Cpi&space;%7D هستش

موفق باشین
89/9/25

lebesgue
16-12-2010, 21:19
بله، فکر کنم [ برای مشاهده لینک ، لطفا با نام کاربری خود وارد شوید یا ثبت نام کنید ] pi&space;%7D%5C;&space;%5C;&space;%5Cmathbf%7B,%7D%5C;&space;%5C;&space;%5Cfrac% 7B-2x%7D%7B%5Cpi&space;%7D


4,294,967,295 کاربر از davy jones بخاطر این مطلب مفید تشکر کرده اند
چه تحویلی گرفتن ما رو!:27:

Paradise_human
17-12-2010, 13:49
بله، فکر کنم [ برای مشاهده لینک ، لطفا با نام کاربری خود وارد شوید یا ثبت نام کنید ] pi&space;%7D%5C;&space;%5C;&space;%5Cmathbf%7B,%7D%5C;&space;%5C;&space;%5Cfrac% 7B-2x%7D%7B%5Cpi&space;%7D


چه تحویلی گرفتن ما رو!:27:
ممنون از توجهتون دوستان ولی اگه ما نقاط رو (منظورم x و y هستش ) در معادله قرار بدیم روی شکل صدق نمیکنه ...
مثلا اگه ما به جای x صفر قرار بدیم باید طبق شکل مثلا در ضابطه ی دوم y ما 1 بشه وای صفر میشه ...
یا مثلا اگه ما به جای X قرار بدیم پی دوم ، y ما میشه 1 که صحیح نیست و باید صفر بشه.
من هر کاری کردم که این نقاط رو به هم ربط بدم نشد و فقط تنها تابعی که این نقاط توش صدق میکنه تابع y=cosx هستش که فکر نیمکنم اینم صحیح باشه.

lebesgue
17-12-2010, 18:51
خب این ضابطه f(x) - 1 هست دیگه، یعنی:

[ برای مشاهده لینک ، لطفا با نام کاربری خود وارد شوید یا ثبت نام کنید ](x)-1=%5Cleft%5C{%5Cbegin{matrix}%5Cfrac{2x}{%5Cpi%20} %20&%20-%5Cpi%20%5Cleq%20x%3C%200%5C%5C%20&%20%5C%5C%20%5Cfrac{-2x}{%5Cpi%20}%20&%200%20%5Cleq%20x%3C%20%5Cpi%20%5Cend{matrix}%5Cri ght.%5C:%20%5C:%5C:%20%5C:%20%5C:%20%5C:,%20%5C:%2 0%5C:%20%5C:%20%5C:%20%5C:%20f(x+2%5Cpi)=f(x)


4,294,967,295 کاربر از davy jones بخاطر این مطلب مفید تشکر کرده اند
32 بیتش پر شده!
این رو که دیدم، یاد این خبر فالس نیوز افتادم:

برای مشاهده محتوا ، لطفا وارد شوید یا ثبت نام کنید

Paradise_human
17-12-2010, 20:52
خب این ضابطه f(x) - 1 هست دیگه، یعنی:

[ برای مشاهده لینک ، لطفا با نام کاربری خود وارد شوید یا ثبت نام کنید ] %7B%5Cpi%20%7D%20&%20-%5Cpi%20%5Cleq%20x%3C%200%5C%5C%20&%20%5C%5C%20%5Cfrac%7B-2x%7D%7B%5Cpi%20%7D%20&%200%20%5Cleq%20x%3C%20%5Cpi%20%5Cend%7Bmatrix%7D% 5Cright.%5C:%20%5C:%5C:%20%5C:%20%5C:%20%5C:,%20%5 C:%20%5C:%20%5C:%20%5C:%20%5C:%20f%28x+2%5Cpi%29=f %28x%29


32 بیتش پر شده!
این رو که دیدم، یاد این خبر فالس نیوز افتادم:

برای مشاهده محتوا ، لطفا وارد شوید یا ثبت نام کنید
تا حالا یه همچین ضابطه ای ندیده بودم !!!
میشه یکم بیشتر راجع بهش توضیح بدین ...
ممنون.

davy jones
17-12-2010, 21:27
تا حالا یه همچین ضابطه ای ندیده بودم !!!
میشه یکم بیشتر راجع بهش توضیح بدین ...
ممنون.
چرا مساله رو اینقدر برای خودت سخت میکنی؟ این ضابطه ای که دوستمون نوشتند رو اگه رسم کنی میبینی که دقیقا مشابه قسمت قرمز رنگ شکل شماست. طریقه ی بدست آوردنش هم تابلوه دیگه. یه خط راست که از دو نقطه با مختصات معلوم عبور کرده. به همین راحتی. هیچ نکته ی ویژه ای نداره.:20:

موفق باشین.
89/9/25

mohsen_blid
17-12-2010, 21:42
سلام دوستان یکی میشه اینجا اموزش بدست اوردن زوایه ای در تانژانت و بالعکس بدست اوردن رادیان یک تانژانت رو بده
می خوام یاد بگیرم چجوری تانژانت یک رقم میشه زوایه چند و بالعکس
ممنون میشم

davy jones
18-12-2010, 07:37
سلام دوستان یکی میشه اینجا اموزش بدست اوردن زوایه ای در تانژانت و بالعکس بدست اوردن رادیان یک تانژانت رو بده
می خوام یاد بگیرم چجوری تانژانت یک رقم میشه زوایه چند و بالعکس
ممنون میشم
سلام.
شکل تابع y= tan x به صورت زیر هستش:


برای مشاهده محتوا ، لطفا وارد شوید یا ثبت نام کنیدهمونطور که در لینک بالا میتونین مشاهده کنین، اگه یه خط افقی موازی با محور x ها رسم کنیم، بینهایت بار با نمودار تابع y=tan x برخورد میکنه که فاصله ی هر دو نقطه ی متوالی این نقاط از همدیگه روی محور x ها، برابر با [ برای مشاهده لینک ، لطفا با نام کاربری خود وارد شوید یا ثبت نام کنید ] هستش. و این یعنی تعداد نامتناهی نقطه وجود دارند که تانژانت اونها با همدیگه برابره. حالا برعکس جمله ی بولد شده یعنی به ازای هر مقدار حقیقی، بینهایت زاویه وجود دارد که برابر با آرک تانژانت (تابع وارون تانژانت) اون عدد حقیقی باشه.

موفق باشین.
89/9/27

mohsen_blid
18-12-2010, 11:35
ممنون دیوی جونز اما من می خوام راه بدست اوردن مثلا pi/4 رو بدونم در تانژانت چند میشه؟ یا مثلا تانژانت 43 درجه چند میشه ؟یا اینکه مثلا تانژانت برابر 4/3- بود یعنی چه زاویه ای داره تانژانت ؟
ممنون میشم راهنمایی کنید در این رابطه
بازم ممنون

davy jones
19-12-2010, 14:20
ممنون دیوی جونز اما من می خوام راه بدست اوردن مثلا pi/4 رو بدونم در تانژانت چند میشه؟ یا مثلا تانژانت 43 درجه چند میشه ؟یا اینکه مثلا تانژانت برابر 4/3- بود یعنی چه زاویه ای داره تانژانت ؟
ممنون میشم راهنمایی کنید در این رابطه
بازم ممنون

اصولی ترین روش، استفاده از مثلث قائم الزاویه (طبق اصل تعریف تانژانت) هستش. مثلا میخوایم ببینیم تانژانت زاویه ی x چقدره؟ (زاویه ی x فرضا بین صفر و 90 درجه است) میایم و یک مثلث قائم الزاویه میکشیم که علاوه بر یک زاویه ی 90 درجه که داره، یکی دیگه از زاویه هاش x باشه. حالا تانژانت x بنا بر تعریف تابع تانژانت برابر با نسبت طول ضلع مقابل زاویه ی x تقسیم بر طول ضلع مجاور زاویه ی x هستش. (البته ضلع مجاوری که وتر مثلث نیست)

به شکل زیر دقت کن:

[ برای مشاهده لینک ، لطفا با نام کاربری خود وارد شوید یا ثبت نام کنید ]

tan(theta) = sin(theta) / cos(theta) = a / b



برای زوایای بزرگتر از 90 درجه و کوچکتر از 180 درجه هم میتوانیم از رابطه ی زیر کمک بگیریم:


[ برای مشاهده لینک ، لطفا با نام کاربری خود وارد شوید یا ثبت نام کنید ]


برای زوایای بین 180 درجه تا 360 درجه هم از رابطه ی زیر میشه استفاده کرد تا باز دوباره بتونیم از مثلث استفاده کنیم:


[ برای مشاهده لینک ، لطفا با نام کاربری خود وارد شوید یا ثبت نام کنید ]


برای زوایای بزرگتر از 360 درجه هم واضحه که میتونیم از اونها 360 تا کم کنیم بدون اینکه تغییری در نتیجه حاصل بشه.





پس ...

با این اوصاف و با روش رسم مثلث قائم الزاویه میشه تانژانت هر زاویه ای رو حساب کرد و برعکس. یعنی میشه با دونستن تانژانت، زاویه ی متناظر با اون رو حساب کرد.

-----------

ولی یه راه ساده تر هم وجود داره که اونم استفاده از سری مک لورن تابع تانژانت هستش. اینکه این سری از کجا اومده و چطوری اثبات میشه رو اگه نمیدونین، خودتون بعدا برین مطالعه کنین ولی این سری به ما میگه که تابعی مثل y= tan x رو میشه اینطور تقریب زد:


[ برای مشاهده لینک ، لطفا با نام کاربری خود وارد شوید یا ثبت نام کنید ]



[ برای مشاهده لینک ، لطفا با نام کاربری خود وارد شوید یا ثبت نام کنید ]


و تابع وارون تانژانت رو هم به این صورت:


[ برای مشاهده لینک ، لطفا با نام کاربری خود وارد شوید یا ثبت نام کنید ]



[ برای مشاهده لینک ، لطفا با نام کاربری خود وارد شوید یا ثبت نام کنید ]

[ برای مشاهده لینک ، لطفا با نام کاربری خود وارد شوید یا ثبت نام کنید ]



این سری ها تا بینهایت باید ادامه داشته باشند تا بتوانیم با قاطعیت بگیم که دو طرف تساوی با هم برابرند. ولی میشه با تقریب خوبی مثلا تا جمله ی 7 ام رو از سمت راست تساوی حساب کرد و مقدار بدست اموده رو معادل تانژانت و یا آرک تانژانت مقدار مورد نظرمون فرض کرد.

موفق باشین.
89/9/28

ALt3rnA
27-12-2010, 01:13
کتاب درسی حسابان جدید
صفحه ی 108
سوال 27
یه سوال داده گفته تابع وارون اینو بیابید :
fx = x^3 - 2x
من با اون فرمول عظیم معادلات درجه 3 حلش کردم ! :دی (نمیدونم میدونید کدومو میگم)
ولی خوب کسی راه بهتری داره ؟

davy jones
27-12-2010, 09:58
کتاب درسی حسابان جدید
صفحه ی 108
سوال 27
یه سوال داده گفته تابع وارون اینو بیابید :
fx = x^3 - 2x
من با اون فرمول عظیم معادلات درجه 3 حلش کردم ! :دی (نمیدونم میدونید کدومو میگم)
ولی خوب کسی راه بهتری داره ؟
البته چون تابع f یک به یک نیست لذا باید در یک بازه ای که تابع یک به یک میشه ازمون بخواد که وارونش رو حساب کنیم وگرنه وارون این تابع به صورت تابع نخواهد بود.
اون فرمول عظیم رو هم اگه میشه بذارین چون مشتاق شدم بدونم چیه.
اینم جوابی که والفرام درآورد:


[ برای مشاهده لینک ، لطفا با نام کاربری خود وارد شوید یا ثبت نام کنید ] Type=image/gif&s=7&w=432&h=97


:18:


موفق باشین.
89/10/6

ALt3rnA
27-12-2010, 11:52
البته چون تابع f یک به یک نیست لذا باید در یک بازه ای که تابع یک به یک میشه ازمون بخواد که وارونش رو حساب کنیم وگرنه وارون این تابع به صورت تابع نخواهد بود.
اون فرمول عظیم رو هم اگه میشه بذارین چون مشتاق شدم بدونم چیه.
اینم جوابی که والفرام درآورد:

[ برای مشاهده لینک ، لطفا با نام کاربری خود وارد شوید یا ثبت نام کنید ] Type=image/gif&s=7&w=432&h=97

[ برای مشاهده لینک ، لطفا با نام کاربری خود وارد شوید یا ثبت نام کنید ]

موفق باشین.

آخ .... :دی
درسته ! بازه اش یادم رفت !
1 بسته تا مثبت بینهایت بود

اون فرمول هم اینه البته فکر کنم بدونید:

[ برای مشاهده لینک ، لطفا با نام کاربری خود وارد شوید یا ثبت نام کنید ]

farzaneh*f
27-12-2010, 13:53
سلام:20:
می تونین حل کامل این انتگرال رو برام بذارین : [ برای مشاهده لینک ، لطفا با نام کاربری خود وارد شوید یا ثبت نام کنید ] \frac{dx}{x^{4}+1}

خیلی ممنون :11:

davy jones
27-12-2010, 14:31
سلام:20:
می تونین حل کامل این انتگرال رو برام بذارین : [ برای مشاهده لینک ، لطفا با نام کاربری خود وارد شوید یا ثبت نام کنید ] %7D

خیلی ممنون :11:
سلام.
خودتون میتونین از اینجا ([ برای مشاهده لینک ، لطفا با نام کاربری خود وارد شوید یا ثبت نام کنید ]) ببینین.

موفق باشین.
89/10/6

eh_mn
27-12-2010, 18:39
سلام.
خودتون میتونین از اینجا ([ برای مشاهده لینک ، لطفا با نام کاربری خود وارد شوید یا ثبت نام کنید ]) ببینین.

موفق باشین.
89/10/6

اگه اين كه گفتن درست نبود اين رو امتحان كنيد


برای مشاهده محتوا ، لطفا وارد شوید یا ثبت نام کنید

farzaneh*f
27-12-2010, 20:15
بی نهایت ممنون بابت معرفی سایت:11:
همون چیزیه که شدیــــــــــــــــــدا بهش نیاز داشتم این روزا :26:

فقط یه سوال اینجام مث بعضی از ماشین حسابا منظور از tan^ -1 همون arctan هست؟!:31:

davy jones
27-12-2010, 20:46
بی نهایت ممنون بابت معرفی سایت:11:
همون چیزیه که شدیــــــــــــــــــدا بهش نیاز داشتم این روزا :26:

فقط یه سوال اینجام مث بعضی از ماشین حسابا منظور از tan^ -1 همون arctan هست؟!:31:
آره. فکر کنم تو خود همونجا هم برای اینکه ممکنه سوال برانگیز باشه، اینو توضیح داده بود و تاکید کرده بود که منظورش همونه. به عکس زیر دقت کنین:

[ برای مشاهده لینک ، لطفا با نام کاربری خود وارد شوید یا ثبت نام کنید ]

موفق باشین.
89/10/6

ALt3rnA
27-12-2010, 23:48
این ولفرام تابع وارون نمیگیره ؟ :دی
من کارم گیره :(

davy jones
28-12-2010, 08:38
این ولفرام تابع وارون نمیگیره ؟ :دی
من کارم گیره :(

چرا میگیره. کافیه عبارت inverse of f رو وارد کنین (البته به جای f تابع مورد نظرتون رو بنویسین. یعنی مثلا ننویسین ...=f بلکه فقط ضابطه ی تابع رو بنویسین)
به طور مثال من ضابطه ی تابع f(x)=3x^2 رو وارد کردم و وارونش رو برام محاسبه کرد که میتونین از اینجا ([ برای مشاهده لینک ، لطفا با نام کاربری خود وارد شوید یا ثبت نام کنید ]) ملاحظه کنین.

موفق باشین.
89/10/7

kasra_khan2003
28-12-2010, 17:20
یه سوال ساده داشتم:

باقیمانده تقسیم "197 بتوان 48" بر "2" چی میشه؟ ( جواب باید 1 بیاد )

ALt3rnA
28-12-2010, 18:58
یه سوال ساده داشتم:

باقیمانده تقسیم "197 بتوان 48" بر "2" چی میشه؟ ( جواب باید 1 بیاد )

معمولا تو این سوالا اینقدر با پایه ور میریم تا باقیمانده اش بر اون عدد یا صفر یا یک شه
اما اینجا کلا 197 تقسیم بر 2 باقیمانده اش یکه !
در نتیجه باقیمانده ی کل همون 1 میشه

mmmmohsenea
28-12-2010, 19:01
im (x-->0) (x-tan(x)) /x^3
وقتی این سوال رو با هم ارزی یا برنامه های ریاضی یا بسط تیلور یا هوپیتال حل میکنیم جواب 1/3 هست ولی با روش حل مستقیم و با روابط مثلثاتی و فقط هم ارزی sin(x)/x=1 جواب 1/2 میاد.
حالا اگه کسی میتونه این سوالو با شرایط فوق ( بدون همارزی و ...)حل کنه و 1/3 بیاره ممنون میشم.

kasra_khan2003
29-12-2010, 03:31
معمولا تو این سوالا اینقدر با پایه ور میریم تا باقیمانده اش بر اون عدد یا صفر یا یک شه
اما اینجا کلا 197 تقسیم بر 2 باقیمانده اش یکه !
در نتیجه باقیمانده ی کل همون 1 میشه

خوب من اینو چجوری برای یه اول راهنمایی توضیح بدم؟ میشه یکم بیشتر توضیح بدین؟

mohsen_blid
29-12-2010, 08:59
میشه در باره لگاریتم در پایه های مختلف یه اموزش کوچولو بزارید؟

davy jones
29-12-2010, 11:19
یه سوال ساده داشتم:

باقیمانده تقسیم "197 بتوان 48" بر "2" چی میشه؟ ( جواب باید 1 بیاد )

جواب این سوال خیلی خیلی آسونه. همونطور که خود شما هم میدونین حاصل ضرب دو عدد طبیعی مثل a و b تنها در صورتی یک عدد فرد خواهد بود که هر دوی a و b همزمان فرد باشند. در صورتی که هر کدوم از اینها (یا جفتشون) زوج باشه حاصلضرب حتما زوج میشه. عدد 197 یه عدد فرد هستش. به توان رسوندن هم یعنی در خود اون عدد ضرب کردن. پس بنابراین اگه 48 تا عدد فرد رو در هم ضرب کنیم باز هم حاصل عددی فرد هستش. پس باقیمانده ی تقسیم 197 به توان 48 بر عدد 2 برابر با یک خواهد بود. به همین راحتی:31:


im (x-->0) (x-tan(x)) /x^3
وقتی این سوال رو با هم ارزی یا برنامه های ریاضی یا بسط تیلور یا هوپیتال حل میکنیم جواب 1/3 هست ولی با روش حل مستقیم و با روابط مثلثاتی و فقط هم ارزی sin(x)/x=1 جواب 1/2 میاد.
حالا اگه کسی میتونه این سوالو با شرایط فوق ( بدون همارزی و ...)حل کنه و 1/3 بیاره ممنون میشم.

سلام.
البته جواب منفی یک سوم در میاد. بفرمایین:


[ برای مشاهده لینک ، لطفا با نام کاربری خود وارد شوید یا ثبت نام کنید ] 5Cfrac%7Bx-%5Ctan&space;x%7D%7Bx%5E%7B3%7D%7D%5C;&space;%5Coverset%7BHopi tal%7D%7B%5Crightarrow%7D%5C;&space;%5Clim_%7Bx%5Crighta rrow&space;0%7D%5Cfrac%7B1-%281&plus;%5Ctan%5E%7B2%7D&space;x%29%7D%7B3x%5E%7B2%7D%7D=%5 Clim_%7Bx%5Crightarrow&space;0%7D%5Cfrac%7B-%5Ctan%5E%7B2%7D&space;x%7D%7B3x%5E%7B2%7D%7D%5Coverset% 7BH%7D%7B%5Crightarrow%7D%5Clim_%7Bx%5Crightarrow&space; 0%7D%5Cfrac%7B-2%5Ctan&space;x%281&plus;%5Ctan&space;%5E%7B2%7Dx%29%7D%7B6x%7D=%5C lim_%7Bx%5Crightarrow&space;0%7D%5Cfrac%7B-%5Ctan&space;x-%5Ctan%5E%7B3%7D&space;x%7D%7B3x%7D%5Coverset%7BH%7D%7B% 5Crightarrow%7D%5Clim_%7Bx%5Crightarrow&space;0%7D%5Cfra c%7B-%281&plus;%5Ctan&space;%5E%7B2%7Dx%29-3%5Ctan&space;%5E%7B2%7Dx%281&plus;%5Ctan&space;%5E%7B2%7Dx%29%7D%7 B3%7D=%5Cfrac%7B-%281&plus;0%29-3%5Ctimes&space;0%281&plus;0%5E%7B2%7D%29%7D%7B3%7D=%5Cfrac%7 B-1%7D%7B3%7D



میشه در باره لگاریتم در پایه های مختلف یه اموزش کوچولو بزارید؟

لگاریتم طبق تعریف عکس تابع نمایی هستش. پایه ی لگاریتم طبق تعریف نباید منفی باشه و همچنین نباید عدد 1 باشه. چرا که در غیر این صورت حاصل لگاریتم به صورت حقیقی قابل تعریف نیست.اگر پایه ی لگاریتم در بازه ی باز بین صفر و یک قرار بگیره، این تابع اکیدا نزولی خواهد بود ولی اگه پایه ی لگاریتم در بازی باز بین 1 تا مثبت بینهایت باشه این تابع اکیدا صعودی خواهد بود.

برای مطالعه ی بیشتر به اینجا ([ برای مشاهده لینک ، لطفا با نام کاربری خود وارد شوید یا ثبت نام کنید ])مراجعه کنین.



موفق باشین.
89/10/8

mohsen_blid
29-12-2010, 12:58
با سلام
می خوام بدونم این عبارت ها چجوری بدست میاد یکم توضیح در موردش ممنون میشم مخصوصا در بخش قرمز شده
[ برای مشاهده لینک ، لطفا با نام کاربری خود وارد شوید یا ثبت نام کنید ]

و

[ برای مشاهده لینک ، لطفا با نام کاربری خود وارد شوید یا ثبت نام کنید ] 7Bn%28n&plus;1%29%7D%7B2%7D%7D

مخصوصا که 2 از کجا میاد

davy jones
29-12-2010, 16:21
[ برای مشاهده لینک ، لطفا با نام کاربری خود وارد شوید یا ثبت نام کنید ] 7Bn%28n&plus;1%29%7D%7B2%7D%7D

مخصوصا که 2 از کجا میاد
سلام.

بفرمایین:

[ برای مشاهده لینک ، لطفا با نام کاربری خود وارد شوید یا ثبت نام کنید ] =1%5C;&space;%5C;%5C;&space;%5C;&space;&plus;%5C;&space;%5C;&space;%5C;&space;2%5C;&space;%5C;%5C ;&space;%5C;&space;%5C;&space;&plus;%5C;&space;%5C;&space;%5C;&space;%5C;&space;3%5C;&space;%5C;%5C;&space;%5 C;&space;%5C;&space;&plus;%5C;&space;%5C;&space;%5C;&space;...%5C;&space;%5C;&space;%5C;&space;&plus;%5C;&space;%2 8n-2%29&plus;%28n-1%29&plus;n%5C%5C&space;S=n%5C;&space;&plus;%28n-1%29&plus;%28n-2%29&plus;%5C;&space;%5C;&space;%5C;&space;...%5C;&space;%5C;&space;%5C;&space;&plus;%5C;&space;%5C;&space;% 5C;&space;%5C;&space;3%5C;&space;%5C;&space;%5C;&space;%5C;&space;&plus;%5C;&space;%5C;&space;%5C;&space;%5C; &space;%5C;&space;2%5C;&space;%5C;&space;%5C;&space;&plus;%5C;&space;%5C;&space;%5C;&space;%5C;&space;%5C;&space;1&space; %5Cend%7Bmatrix%7D%5Cright%5C%7D%5C;&space;%5C;&space;%5C;&space;%5C ;&space;%5C;&space;%5C;&space;%5CRightarrow&space;%5C;&space;%5C;&space;%5C;&space;%5C;&space;%5C; &space;%5C;&space;2S=%281&plus;n%29&plus;%282&plus;n-1%29&plus;%283&plus;n-2%29&plus;...&plus;%28n-2&plus;3%29&plus;%28n-1&plus;2%29&plus;%28n&plus;1%29=n%28n&plus;1%29%5CRightarrow&space;%7B%5Ccol or%7Bred%7D&space;S=%5Cfrac%7Bn%28n&plus;1%29%7D%7B2%7D%7D


عبارت اولی هم که نوشته بودین در حقیقت همینه که در اون n=i-1 فرض شده.

موفق باشین.
89/10/8

mmmmohsenea
29-12-2010, 17:12
جواب این سوال خیلی خیلی آسونه. همونطور که خود شما هم میدونین حاصل ضرب دو عدد طبیعی مثل a و b تنها در صورتی یک عدد فرد خواهد بود که هر دوی a و b همزمان فرد باشند. در صورتی که هر کدوم از اینها (یا جفتشون) زوج باشه حاصلضرب حتما زوج میشه. عدد 197 یه عدد فرد هستش. به توان رسوندن هم یعنی در خود اون عدد ضرب کردن. پس بنابراین اگه 48 تا عدد فرد رو در هم ضرب کنیم باز هم حاصل عددی فرد هستش. پس باقیمانده ی تقسیم 197 به توان 48 بر عدد 2 برابر با یک خواهد بود. به همین راحتی:31:



سلام.
البته جواب منفی یک سوم در میاد. بفرمایین:


[ برای مشاهده لینک ، لطفا با نام کاربری خود وارد شوید یا ثبت نام کنید ] 5Cfrac%7Bx-%5Ctan&space;x%7D%7Bx%5E%7B3%7D%7D%5C;&space;%5Coverset%7BHopi tal%7D%7B%5Crightarrow%7D%5C;&space;%5Clim_%7Bx%5Crighta rrow&space;0%7D%5Cfrac%7B1-%281&plus;%5Ctan%5E%7B2%7D&space;x%29%7D%7B3x%5E%7B2%7D%7D=%5 Clim_%7Bx%5Crightarrow&space;0%7D%5Cfrac%7B-%5Ctan%5E%7B2%7D&space;x%7D%7B3x%5E%7B2%7D%7D%5Coverset% 7BH%7D%7B%5Crightarrow%7D%5Clim_%7Bx%5Crightarrow&space; 0%7D%5Cfrac%7B-2%5Ctan&space;x%281&plus;%5Ctan&space;%5E%7B2%7Dx%29%7D%7B6x%7D=%5C lim_%7Bx%5Crightarrow&space;0%7D%5Cfrac%7B-%5Ctan&space;x-%5Ctan%5E%7B3%7D&space;x%7D%7B3x%7D%5Coverset%7BH%7D%7B% 5Crightarrow%7D%5Clim_%7Bx%5Crightarrow&space;0%7D%5Cfra c%7B-%281&plus;%5Ctan&space;%5E%7B2%7Dx%29-3%5Ctan&space;%5E%7B2%7Dx%281&plus;%5Ctan&space;%5E%7B2%7Dx%29%7D%7 B3%7D=%5Cfrac%7B-%281&plus;0%29-3%5Ctimes&space;0%281&plus;0%5E%7B2%7D%29%7D%7B3%7D=%5Cfrac%7 B-1%7D%7B3%7D




لگاریتم طبق تعریف عکس تابع نمایی هستش. پایه ی لگاریتم طبق تعریف نباید منفی باشه و همچنین نباید عدد 1 باشه. چرا که در غیر این صورت حاصل لگاریتم به صورت حقیقی قابل تعریف نیست.اگر پایه ی لگاریتم در بازه ی باز بین صفر و یک قرار بگیره، این تابع اکیدا نزولی خواهد بود ولی اگه پایه ی لگاریتم در بازی باز بین 1 تا مثبت بینهایت باشه این تابع اکیدا صعودی خواهد بود.

برای مطالعه ی بیشتر به اینجا ([ برای مشاهده لینک ، لطفا با نام کاربری خود وارد شوید یا ثبت نام کنید ])مراجعه کنین.



موفق باشین.
89/10/8
شرمنده با هوپیتال که خودم هم بلد بودم بیزحمت بدون هوپیتال بست تیلور هم ارزی و ..
فقط با مثلثات و sin(x)/x=1

mohsen_blid
30-12-2010, 11:49
سلام.

بفرمایین:

[ برای مشاهده لینک ، لطفا با نام کاربری خود وارد شوید یا ثبت نام کنید ] =1%5C;&space;%5C;%5C;&space;%5C;&space;&plus;%5C;&space;%5C;&space;%5C;&space;2%5C;&space;%5C;%5C ;&space;%5C;&space;%5C;&space;&plus;%5C;&space;%5C;&space;%5C;&space;%5C;&space;3%5C;&space;%5C;%5C;&space;%5 C;&space;%5C;&space;&plus;%5C;&space;%5C;&space;%5C;&space;...%5C;&space;%5C;&space;%5C;&space;&plus;%5C;&space;%2 8n-2%29&plus;%28n-1%29&plus;n%5C%5C&space;S=n%5C;&space;&plus;%28n-1%29&plus;%28n-2%29&plus;%5C;&space;%5C;&space;%5C;&space;...%5C;&space;%5C;&space;%5C;&space;&plus;%5C;&space;%5C;&space;% 5C;&space;%5C;&space;3%5C;&space;%5C;&space;%5C;&space;%5C;&space;&plus;%5C;&space;%5C;&space;%5C;&space;%5C; &space;%5C;&space;2%5C;&space;%5C;&space;%5C;&space;&plus;%5C;&space;%5C;&space;%5C;&space;%5C;&space;%5C;&space;1&space; %5Cend%7Bmatrix%7D%5Cright%5C%7D%5C;&space;%5C;&space;%5C;&space;%5C ;&space;%5C;&space;%5C;&space;%5CRightarrow&space;%5C;&space;%5C;&space;%5C;&space;%5C;&space;%5C; &space;%5C;&space;2S=%281&plus;n%29&plus;%282&plus;n-1%29&plus;%283&plus;n-2%29&plus;...&plus;%28n-2&plus;3%29&plus;%28n-1&plus;2%29&plus;%28n&plus;1%29=n%28n&plus;1%29%5CRightarrow&space;%7B%5Ccol or%7Bred%7D&space;S=%5Cfrac%7Bn%28n&plus;1%29%7D%7B2%7D%7D


عبارت اولی هم که نوشته بودین در حقیقت همینه که در اون n=i-1 فرض شده.

موفق باشین.
89/10/8




ممنون از پاسخت
این مبحث رو می تونم کجا بیشتر در موردش یاد بگیرم
ممنون میشم راهنمایی کنید

Mohammad Hosseyn
30-12-2010, 13:23
دوستان اثبات مشتق ln رو کسی می تونه بزاره (با توجه به تعریف مشتق)

davy jones
30-12-2010, 18:08
ممنون از پاسخت
این مبحث رو می تونم کجا بیشتر در موردش یاد بگیرم
ممنون میشم راهنمایی کنید

خواهش میکنم. قابلی نداشت. این مبحث رو تو کتابها و جزوات مربوط به آنالیز ترکیبی و ریاضیات گسسته بیشتر میتونین در موردش مطالعه کنین.

------------


دوستان اثبات مشتق ln رو کسی می تونه بزاره (با توجه به تعریف مشتق)

سلام. البته تو هر سایت و ماشین حساب مهندسی هم که بزنین میبینین که مشتق ln رو طبق تعریف و به عنوان یک اصل پذیرفته شده برابر با یک تقسیم بر x محاسبه میکنه و هیچ توضیحی هم نمیده. ولی بنده یه اثبات من درآوری براش نوشتم. امیدوارم که بتونه نظرتون رو جلب کنه:


[ برای مشاهده لینک ، لطفا با نام کاربری خود وارد شوید یا ثبت نام کنید ] lta&space;x%5Crightarrow&space;0%7D%5Cfrac%7B%5Cln&space;%28x&plus;%5CDel ta&space;x%29-%5Cln&space;x%7D%7B%5CDelta&space;x%7D=%5Clim_%7B%5CDelta&space;x%5C rightarrow&space;0%7D%5Cfrac%7B1%7D%7B%5CDelta&space;x%7D%5Cln &space;%28%5Cfrac%7Bx&plus;%5CDelta&space;x%7D%7Bx%7D%29=%5Clim_%7B %5CDelta&space;x%5Crightarrow&space;0%7D%5Cfrac%7B1%7D%7B%5CDe lta&space;x%7D%5Cln&space;%281&plus;%5Cfrac%7B%5CDelta&space;x%7D%7Bx%7D% 29=%5Clim_%7B%5CDelta&space;x%5Crightarrow&space;0%7D%5Cln&space;[%281&plus;%5Cfrac%7B%5CDelta&space;x%7D%7Bx%7D%29%5E%7B%5Cfra c%7B1%7D%7B%5CDelta&space;x%7D%7D]%5CRightarrow&space;e%5E%7BI%7D=e%5E%7B%5Clim_%7B%5CDelt a&space;x%5Crightarrow&space;0%7D%5Cln&space;[%281&plus;%5Cfrac%7B%5CDelta&space;x%7D%7Bx%7D%29%5E%7B%5Cfra c%7B1%7D%7B%5CDelta&space;x%7D%7D]%7D=%5Clim_%7B%5CDelta&space;x%5Crightarrow&space;0%7De%5E%7B% 5Cln&space;[%281&plus;%5Cfrac%7B%5CDelta&space;x%7D%7Bx%7D%29%5E%7B%5Cfra c%7B1%7D%7B%5CDelta&space;x%7D%7D]%7D=%5Clim_%7B%5CDelta&space;x%5Crightarrow&space;0%7D%281&plus;%5C frac%7B%5CDelta&space;x%7D%7Bx%7D%29%5E%7B%5Cfrac%7B1%7D %7B%5CDelta&space;x%7D%7D=e%5E%7B%5Cfrac%7B1%7D%7Bx%7D%7 D%5CRightarrow&space;%7B%5Ccolor%7Bred%7D&space;I=%5Cfrac%7B1% 7D%7Bx%7D%7D

موفق باشین.
نهم دیماه 1389

lebesgue
30-12-2010, 19:19
نشان دهید برای هر x حقیقی داریم:


[ برای مشاهده لینک ، لطفا با نام کاربری خود وارد شوید یا ثبت نام کنید ]{%5Csin%20(x)}{1!}+%5 Cfrac{%5Csin%20(2x)}{2!}+%5Cfrac{%5Csin%20(3x)}{3! }+%5Ccdots%20=e^{%5Ccos%20(x)}%5Csin%20(%5Csin%20( x))

lebesgue
31-12-2010, 11:13
im (x-->0) (x-tan(x)) /x^3
وقتی این سوال رو با هم ارزی یا برنامه های ریاضی یا بسط تیلور یا هوپیتال حل میکنیم جواب 1/3 هست ولی با روش حل مستقیم و با روابط مثلثاتی و فقط هم ارزی sin(x)/x=1 جواب 1/2 میاد.
حالا اگه کسی میتونه این سوالو با شرایط فوق ( بدون همارزی و ...)حل کنه و 1/3 بیاره ممنون میشم.


[ برای مشاهده لینک ، لطفا با نام کاربری خود وارد شوید یا ثبت نام کنید ]{x%5Crightarr ow%200}%5Cfrac{x-%5Ctan%20x}{x^3}=%5Clim_{3t%5Crightarrow%200}%5Cfr ac{3t-%5Ctan%203t}{(3t)^3}=%5Clim_{t%5Crightarrow%200}%5 Cfrac{3t-%5Cfrac{3%5Ctan%20t-%5Ctan%20^3t}{1-3%5Ctan%20^2t}}{27t^3}%5C%5C%5C%5C%5C%5C%20=%5Clim _{t%5Crightarrow%200}%5Cfrac{3t(1-3%5Ctan%20^2t)-(3%5Ctan%20t-%5Ctan%20^3t)}{27t^3(1-3%5Ctan%20^2t)}%5C%5C%5C%5C%5C%5C%20=%5Clim_{t%5Cr ightarrow%200}%5Cfrac{3t-9t%5Ctan%20^2t-3%5Ctan%20t+%5Ctan%20^3t}{27t^3(1-3%5Ctan%20^2t)}%5C%5C%5C%5C%5C%5C%20=%5Clim_{t%5Cr ightarrow%200}%5Cfrac{3t-9t%5Ctan%20^2t-3%5Ctan%20t+%5Ctan%20^3t}{27t^3}%5C%5C%5C%5C%5C%5C %20=%5Clim_{t%5Crightarrow%200}%5Cfrac{3t-3%5Ctan%20t}{27t^3}+%5Clim_{t%5Crightarrow%200}%5C frac{-9t%5Ctan%20^2t}{27t^3}+%5Clim_{t%5Crightarrow%200} %5Cfrac{%5Ctan%20^3t}{27t^3}%5C%5C%5C%5C%5C%5C%20= %5Cfrac{1}{9}%5Clim_{t%5Crightarrow%200}%5Cfrac{t-%5Ctan%20t}{t^3}-%5Cfrac{1}{3}%5Clim_{t%5Crightarrow%200}(%5Cfrac{% 5Ctan%20t}{t})^2+%5Cfrac{1}{27}%5Clim_{t%5Crightar row%200}(%5Cfrac{%5Ctan%20t}{t})^3=%5Cfrac{1}{9}L-%5Cfrac{1}{3}+%5Cfrac{1}{27}%5C%5C%5C%5C%5C%5C%20L =%5Cfrac{1}{9}L-%5Cfrac{1}{3}+%5Cfrac{1}{27}%5Crightarrow%20L=-%5Cfrac{1}{3}

البته این استدلال تنها با فرض وجود حد L معتبر است.

ayyob22
31-12-2010, 16:28
سلام دوستان اینو حل کنید.

y'=e^xy

:13:

noia
01-01-2011, 00:09
درود،
دوتا سئوال دارم از محاسبات عددی.
1. در تابع جدولی زیر، مطلوب است محاسبه f'(0.73 {خود f در مقدار 0.73 رو می‌تونم حساب کنم، اما واسه پرایم f نمی‌دونم باید چیکار کرد؛ به فارسی هم اگه توضیح بدید، کافیه. ممنون می‌شم.}
تابع جدولی‌رو دلخواه در نظر بگیرید، فقط توی سئوال نقاط متساوی‌الفاصله‌اند.

2. تقریبی از ریشه معادله x+lnx=0 در بازه [1 , 0.5] به روش تکرار ساده. (5 تکرار)
اون دو شرطی که مخصوص روش تکرار ساده هست جور در نمی‌آد.

davy jones
01-01-2011, 14:25
درود،
دوتا سئوال دارم از محاسبات عددی.
1. در تابع جدولی زیر، مطلوب است محاسبه f'(0.73 {خود f در مقدار 0.73 رو می‌تونم حساب کنم، اما واسه پرایم f نمی‌دونم باید چیکار کرد؛ به فارسی هم اگه توضیح بدید، کافیه. ممنون می‌شم.}
تابع جدولی‌رو دلخواه در نظر بگیرید، فقط توی سئوال نقاط متساوی‌الفاصله‌اند.

2. تقریبی از ریشه معادله x+lnx=0 در بازه [1 , 0.5] به روش تکرار ساده. (5 تکرار)
اون دو شرطی که مخصوص روش تکرار ساده هست جور در نمی‌آد.
درود!

تو سوال اولتون، تابع جدولی رو نمیتونم ببینم. اگه به صورت عکس هستش لطفا یه جای دیگه آپلود کنین. (از آپلود سنترهایی که شیلتر هستند استفاده نکنین)

موفق باشین.
89/10/11